Anda di halaman 1dari 239

i

 
 
 
 
 
 
 
 

FÍSICA III
 
 
ii

 
iii

 
 
 
 
 
 
 
 
 
 
 

FÍSICA III

Gustavo Mauricio Bastién Montoya


Hugo Sergio Becerril Hernández
Nicolás Falcón Hernández
Juan Domingo Pérez López
Alejandro Raymundo Pérez Ricárdez
Abelardo Luis Rodríguez Soria

UNIVERSIDAD
AUTÓNOMA
METROPOLITANA
Casa abierta al tiempo
Azcapotzalco

División de Ciencias Básicas e Ingeniería


Departamento de Ciencias Básicas
iv

 
v

 
CONTENIDO 
 
PREFACIO…………………………………………………………………………………….…………..……….ix 
 
REFERENCIAS BIBLIOGRÁFICAS ………………………………………………………..…………..….xi 
 
CAPÍTULO 1. CAMPO ELÉCTRICO DE CARGAS PUNTUALES 
1.1. CARGA ELÉCTRICA. CONDUCTORES Y AISLADORES. …………………………………………………………..1.1
1.2. LA LEY DE COULOMB…………………………………………………………………………………………...1.5
1.3. DEFINICIÓN DEL CAMPO ELÉCTRICO……………………………………………………………………………1.12
1.4. CAMPO ELÉCTRICO DE UNA CARGA PUNTUAL………………………………………………………………….1.15
1.5. CAMPO ELÉCTRICO DE UN SISTEMA DE CARGAS PUNTUALES……………………………………………...1.18 
1.6. EL DIPOLO ELÉCTRICO………………………………………………………………………………………….1.25 
1.7. TORCA SOBRE UN DIPOLO ELÉCTRICO EN EL SENO DE UN CAMPO ELÉCTRICO UNIFORME…………………...1.32
1.8. PROBLEMAS……………………………………………………………………………………………………..1.34
 
CAPÍTULO 2. CAMPO ELÉCTRICO DE DISTRIBUCIONES CONTINUAS DE CARGA 
2.1. DENSIDAD DE CARGA ELÉCTRICA…………………………………………………………………………….….2.1
2.2. DENSIDAD LINEAL DE CARGA ELÉCTRICA…………………………………………………………………….….2.1
2.3. DENSIDADES SUPERFICIAL Y VOLUMÉTRICA DE CARGA ELÉCTRICA……………………………………………2.4
2.4. PROCEDIMIENTO GENERAL PARA CALCULAR EL CAMPO ELÉCTRICO PRODUCIDO
POR UNA DISTRIBUCIÓN CONTINUA DE CARGA………………………………………………………………..2.8
2.5. CAMPO ELÉCTRICO GENERADO POR UN TROZO DE ALAMBRE RECTO DE LONGITUD L Y CARGA
ELÉCTRICA TOTAL Q DISTRIBUÍDA UNIFORMEMENTE……………………………………………………….2.11
2.5. CAMPO ELÉCTRICO DE UNA ESPIRA CIRCULAR DE RADIO “a”, CARGADA UNIFORMEMENTE CON CARGA
TOTAL “Q”, EN UN PUNTO CUALQUIERA SOBRE SU EJE DE SIMETRÍA PERPENDICULAR……………………2.19
2.6. CAMPO ELÉCTRICO PRODUCIDO POR UNA LÁMINA DELGADA PLANA CIRCULAR DE RADIO “a”
Y CARGA ELÉCTRICA TOTAL Q DISTRIBUÍDA UNIFORMEMENTE ………………………………………….2.25
2.7. SUPERPOSICIÓN VECTORIAL DE CAMPOS ELÉCTRICOS………………………………………………………….2.29
2.7. ROBLEMAS…………………………………………….………………………………………………………...2.32
 
CAPÍTULO 3. LA LEY DE GAUSS 
3.1. FLUJO DE UN CAMPO VECTORIAL A TRAVÉS DE UNA SUPERFICIE………………………………………………3.1
3.2. FLUJO DE UN CAMPO ELÉCTRICO CONSTANTE A TRAVÉS DE UNA
SUPERFICIE PLANA (RECTÁNGULO)……………………………………………………………………………3.1
3.3. INTERPRETACIÓN DEL FLUJO DEL CAMPO DE VELOCIDADES DE UN FLUIDO…………………………………..3.2
3.4. DEFINICIÓN DEL FLUJO DEL CAMPO ELÉCTRICO EN EL CASO GENERAL………………………………………..3.5
3.5. FLUJO DEL CAMPO ELÉCTRICO DE UNA CARGA PUNTUAL A TRAVÉS DE
UNA SUPERFICIE ESFÉRICA CENTRADA EN LA CARGA………………………………………………………..3.6
3.6. FLUJO DEL CAMPO ELÉCTRICO DE UNA CARGA PUNTUAL A TRAVÉS
DE UNA SUPERFICIE ARBITRARIA S. …………………………………………………………………………..3.7

3.7. LEY DE GAUSS PARA UN CONJUNTO DE CARGAS PUNTUALES…………………………………………………..3.8


3.8. LEY DE GAUSS EN GENERAL, VÁLIDA PARA CUALQUIER DISTRIBUCIÓN DE CARGA. …………………………….3.9
vi

3.9. CONDUCTORES Y LEY DE GAUSS………………………………………………………………………………..3.10


3.10. APLICACIÓN DE LA LEY DE GAUSS AL CÁLCULO DE CAMPOS ELÉCTRICOS
CON SIMETRÍA ESFÉRICA.…………………………………………………………………………………….3.12
3.11. APLICACIÓN DE LA LEY DE GAUSS AL CÁLCULO DE CAMPOS ELÉCTRICOS
CON SIMETRÍA CILÍNDRICA…………………………………………………………………………………..3.18
3.12. CÁLCULO DE CAMPOS CON SIMETRÍA PLANA…………………………………………………………………3.20
3.13. PROOBLEMAS………………………………………………………………………………………………….3.22
 
CAPÍTULO 4. POTENCIAL ELÉCTRICO, ENERGÍA Y VOLTAJE 
4.1. INTRODUCCIÓN…………………………………………………………………………………………………...4.1
4.2. POTENCIAL ELÉCTRICO DEL CAPACITOR DE PLACAS PARALELAS………………………………………………4.1
4.3. VOLTAJE O TENSIÓN ELÉCTRICA A TRAVÉS DEL CAPACITOR…………………………………………………..4.4
4.4. MOVIMIENTO DE CARGAS ELÉCTRICAS DENTRO DEL CAMPO DE UN CAPACITOR
DE PLACAS PARALELAS……………………………………………………………………….………..………4.5
4.5. DEFINICIÓN GENERAL DEL POTENCIAL ELÉCTRICO……………………………………………….…………….4.8
4.6. VOLTAJE O TENSIÓN ELÉCTRICA………………………………………………………………………………..4.11
4.6. INTEGRAL DE LÍNEA DEL CAMPO ELÉCTRICO A LO LARGO DE UNA CURVA CERRADA………………………4.13
4.7. POTENCIAL ELÉCTRICO DE UNA CARGA PUNTUAL. 4.14
4.8. POTENCIAL ELÉCTRICO DE UN SISTEMA DE CARGAS PUNTUALES……………………………………………..4.16
4.9. UNA CARGA PUNTUAL MÓVIL EN EL CAMPO DE VARIAS CARGAS PUNTUALES FIJAS…………………………4.21
4.10. SOBRE EL CÁLCULO DE POTENCIALES ELÉCTRICOS. …………………………………………………………..4.23
4.11. CÁLCULO DEL POTENCIAL ELÉCTRICO A PARTIR DE LA CARGA.  …………………………………………...4.28 
4.12. SUPERFICIES EQUIPOTENCIALES…………………………………………………………………………….4.30 
4.13. RELACIÓN ENTRE EL POTENCIAL (O VOLTAJE) Y EL CAMPO ELÉCTRICO. ……………………………………4.33
4.14. AUTOENERGÍA DE UNA DISTRIBUCIÓN DE CARGA……………………………………………………………...4.37
4.15. AUTOENERGÍA DE UN CAPACITOR DE PLACAS PARALELAS…………………………………………………..4.42
4.16. CONDUCTORES Y POTENCIAL ELÉCTRICO…………………………………………………………………….4.44
4.17. FUENTES DE VOLTAJE Y BATERÍAS……………………………………………………………………………4.45
4.18. PROBLEMAS……………………………………………………………………………………………………4.47
 
CAPÍTULO 5. CAPACITORES Y DIELÉCTRICOS. 
5.1. INTRODUCCIÓN…………………………………………………………………………………………………..5.1
5.2. DEFINICIÓN DE CAPACITOR Y CAPACITANCIA. …………………………………………………………………..5.1
5.3. EL CAPACITOR DE PLACAS PARALELAS………………………………………………………………………….5.3
5.4. EL CAPACITOR ESFÉRICO…………………………………………………………………………………………5.5
5.5. EL CAPACITOR CILÍNDRICO……………………………………………………………………………………….5.6
5.6. CAPACITORES EN SERIE O EN PARALELO…………………………………………………………………………5.7
5.7. CAPACITOR CON DIELÉCTRICO. …………………………………………………………………………………5.14
5.8. PROBLEMAS……………………………………………………………………………………………………..5.21
 
CAPÍTULO 6. CORRIENTE ELÉCTRICA Y RESISTIVIDAD. 
6.1. DEFINICIÓN DE CORRIENTE ELÉCTRICA……………………………………………………………………….6.1 
6.2. CORRIENTE ELÉCTRICA EN UN CONDUCTOR………………………………………………………………….6.1 
6.3. VISIÓN MICROSCÓPICA DE LA CORRIENTE EN UN ALAMBRE CONDUCTOR…………………………..……6.8 
vii

 
6.4. CORRIENTE Y RESISTENCIA EN UN CIRCUITO SIMPLE……………………………………………………….6.10 
6.5. RESISTENCIA EQUIVALENTE DE VARIOS RESISTORES. LEYES DE KIRCHHOFF………………………….…6.12 
6.6. CONSIDERACIONES DE ENERGÍA EN CIRCUITOS CON FUENTES DE FEM Y RESISTORES……………….….6.17 
6.7. FUENTES DE FEM REALES………………………………………………………………………………………6.19 
6.8. PROBLEMAS………………………………………………………………………………………………….6.20 
 
CAPÍTULO 7. CAMPO MAGNÉTICO. 
7.1. FENÓMENOS MAGNÉTICOS………………………………………………………………………………………7.1 
7.2. INTERACCIÓN ENTRE CARGAS ELÉCTRICAS PUNTUALES………………………………………………….….7.2 
7.3. DEFINICIÓN DEL CAMPO MAGNÉTICO………………………………………………………………………….7.4 
7.4. MOVIMIENTO DE CARGAS DENTRO DE UN CAMPO MAGNÉTICO. FUERZA DE LORENTZ………………….7.8 
7.5. EL EFECTO HALL…………………………………………………………………………………………………7.9 
7.6. LA LEY DE BIOT‐SAVART……………………………………………………………………………………….7.11 
7.7. CAMPO MAGNÉTICO GENERADO POR UN HILO RECTO INFINITO DE CORRIENTE…………………….….7.13 
7.8. CAMPO MAGNÉTICO DEBIDO A UNA ESPIRA CIRCULAR……………………………………………………7.17 
7.9. CAMPO MAGNÉTICO DE UN SOLENOIDE………………………………………………………………..……7.20 
7.10. CAMPO MAGNÉTICO DE UN SOLENOIDE INFINITO………………………………………………….…….7.22 
7.11. FUERZA MAGNÉTICA SOBRE UN ALAMBRE RECTO QUE TRANSPORTA CORRIENTE ELÉCTRICA 
  DENTRO DE UN CAMPO MAGNÉTICO CONSTANTE……………………………………………………….7.24 
7.12. DEFINICIÓN DEL AMPERIO, UNIDAD ELECTROMAGNÉTICA BÁSICA 
  EN EL SISTEMA INTERNACIONAL DE UNIDADES…………………………………………………………7.28 
7.13. ESPIRA RECTANGULAR DE CORRIENTE DENTRO DE UN CAMPO MAGNÉTICO UNIFORME……….…..…7.30 
7.14. LA LEY DE AMPERE……………………………………………………………………………………………7.32 
7.15. PROBLEMAS……………………………………………………………………………………..…………….7.35 
 
viii

 
ix

 
PREFACIO 
  El  primer  curso  sobre  electricidad  y  magnetismo,  correspondiente  al  Tronco  General  de 
Asignaturas  de  las  carreras  de  ingeniería  en  la  Universidad  Autónoma  Metropolitana  en  Azcapotzalco, 
denominado “FÍSICA III”, comprende los temas esenciales de electrostática y magnétostática, es decir, el 
estudio  de  fenómenos  eléctricos  y  magnéticos  que  no  dependen  del  tiempo.  Este  libro  está  dedicado  a 
dicha materia. 
  La  materia  de  FÍSICA  III  requiere  gran  dedicación,  dada  la  extensión  del  programa  y  las 
dificultades matemáticas que algunos alumnos encuentran en el camino. 
  Es por ello que el presente texto tiene como objetivos principales los siguientes: 
• Presentar la teoría en una forma lo más resumida posible, consistente con el logro de una adecuada 
comprensión de los fenómenos eléctricos y magnéticos. 
• Presentar  un  número  suficiente  de  ejemplos  resueltos  con  todo  detalle,  así  como  problemas 
ajustados  al  nivel  académico  de  este  curso  introductorio.  Se  han  incluído  las  respuestas  a  la 
mayoría de los problemas. 
• Explicar con detalle el significado y aplicación de los principales cálculos matemáticos propios de la 
materia. 
  Los prerrequisitos matemáticos que la materia exige del estudiante son los siguientes: 
i) Conocimiento del álgebra vectorial: suma y resta de vectores; productos escalar y vectorial de vectores; 
expresión de vectores en la base vectorial cartesiana  {i, j, k} y en la base polar en el plano; representación 
paramétrica de curvas en el plano. 
ii)  Conocimiento  del  cálculo  diferencial  e  integral:  derivada;  integral  de  una  sóla  variable;  integral 
definida. 
  En la materia de FÍSICA III trataremos con varios tipos de integrales vectoriales. Si bien este tipo de 
integrales  son  una  novedad  para  el  estudiante,  para  la  clase  de  campos  eléctricos  y  magnéticos  que 
consideraremos  en  este  curso  introductorio,  y  además  para  las  clases  simples  de  geometría  que 
estudiaremos  (rectas,  planos,  círculos,  esferas,  cilindros  regulares),  las  integrales  se  pueden  reducir 
fácilmente a las familiares integrales escalares de una sola variable (integral de Riemann). Para ello, claro 
está,  el  estudiante  debe  entender  perfectamente  el  concepto  de  integral  mismo  y  la  deducción  de  la 
expresión integral, así como también.el significado de cada símbolo que figura en la integral. 
  A  tal  fin  hemos  adoptado  una  notación  consistente  a  lo  largo  de  todo  el  texto.  Para  evaluar  las 
integrales hemos tratado de evitar trucos particulares (muy populares en otros libros de texto), partiendo 
directamente de la forma del integrando. 
  El  libro  presupone  del  estudiante  conocimientos  elementales  sobre  estructura  de  la  materia,  así 
como de las propiedades fundamentales del átomo, como son la masa y la carga eléctrica. 
x

 
xi

 
REFERENCIAS BIBLIOGRÁFICAS 
 
FÍSICA UNIVERSITARIA con FÍSICA MODERNA 
Volumen 2, Undécima edición 
SEARS, ZEMANSKY, YOUNG, FREEDMAN 
Editorial PEARSON/ADDISON WESLEY 
 
FÍSICA 
Volumen 2, Versión Ampliada, Cuarta edición 
HALLIDAY, RESNICK, KRANE 
Editorial CECSA 
 
FÍSICA Para ciencias e ingeniería 
Volumen II, Sexta edición 
SERWAY, JEWETT JR. 
Editorial THOMSON 
 
 
 
 
 
1‐1 
 
  CAPÍTULO 1 
   
  CAMPO ELÉCTRICO DE CARGAS PUNTUALES 
 
 
 
1.1. CARGA ELÉCTRICA. CONDUCTORES Y AISLANTES. 
  La  gran  diversidad  de  fenómenos  electromagnéticos  a  nuestro  alrededor  están  esencialmente 
relacionados con unas propiedades fundamentales de la materia, denominadas la carga eléctrica y el spin.  
A  nivel  atómico,  la  materia  física  está  constituída  por  protones,  neutrones  y  electrones.  Los  protones  y 
neutrones forman el núcleo del átomo, y los electrones circundan el núcleo en capas. 
  El protón y el electrón poseen carga eléctrica, no así el neutrón. La carga eléctrica está cuantizada, es 
decir,  se  observa  siempre  en  múltiplos  de  un  valor  fundamental  cuyo  valor  es  e  =  1.60  (10–19)  C.  El 
protón posee carga positiva igual a “e”, y el electrón carga negativa “–  e”. La siguiente tabla da valores 
aproximados de la masa y carga de ambas partículas.  
 
  Masa  Carga eléctrica 
Protón  1.67 (10–27) kg  1.60 (10–19) C 
Electrón  9.11 (10–31) kg  –1.60 (10–19) C 
 
  La  unidad  S.I.  de  carga  eléctrica  es  el  coulombio  (abreviado  “C”).  Ésta  se  define  en  términos  de  la 
unidad de corriente, el amperio “A”, en la forma  1 C = 1 A ⋅ s. En el Capítulo VII definiremos el amperio. 
Por lo pronto, podemos definir el coulombio grosso modo como la carga total de 6.25 (1018) electrones. 
  La  teoría  moderna  de  las  partículas  elementales  sostiene  que  el  protón,  neutrón  y  electrón  están 
constituídos por otras subpartículas llamadas “quarks”, cuyas cargas eléctricas son fracciones de “e”. Así, 
2 1
el  protón  está  formado  por  tres  quarks;  dos  de  ellos  con  carga  e ,  y  el  tercero  con  carga  − e .  El 
3 3
1 2
neutrón,  cuya  carga  es  nula,  está  formado  por  dos  quarks  de  carga  − e   y un quark  de  carga  e .  Sin 
3 3
embargo, aunque la existencia de los quarks posee evidencia experimental sólida, no ha sido posible hasta 
la fecha observar quarks libres, quizás porque los aceleradores modernos no alcanzan la energía necesaria 
para vencer la energía de ligadura de los quarks. 
  En su estado natural, el átomo es neutro: el número de protones en el núcleo iguala el número de 
electrones circundantes, de tal manera que la carga total del átomo es nula. Un átomo ionizado, al que le 
falta  uno  o  más  electrones,  posee  una  carga  neta  positiva  igual  a    “Ne”,  donde  N  es  el  número  de 
electrones  faltantes.  En  las  interacciones  atómicas  (colisiones,  reacciones  químicas,  etc.)  siempre  se 
observa  que  la  carga  eléctrica  se  conserva.  Esto  constituye  una  ley  fundamental  de  la  naturaleza  en 
cualquier circunstancia. 
  La existencia de carga eléctrica de un trozo de material se explica mediante una deficiencia o exceso 
de  carga,  usualmente  negativa.  Una  deficiencia  de  electrones  significa  una  carga  neta  positiva  del 
material, un exceso una carga negativa. Tal deficiencia o exceso es regularmente una fracción mucho muy 
pequeña de la carga positiva o negativa total contenida en el trozo. 
1‐2 
 
 
EJEMPLO 1.1. ¿Cuánta carga positiva (o negativa) posee un trozo de cobre neutro cuya masa es de 1 g? 
‐‐‐‐‐‐‐‐‐‐‐‐‐‐‐‐‐‐‐‐‐‐‐‐‐‐‐‐‐‐‐‐‐‐‐‐‐‐ 
  El átomo de cobre contiene Z protones y Z electrones en su estado neutro, donde Z es el número 
atómico del cobre, igual a 29. Entonces la carga positiva de cada átomo es “Ze”. 
  Debemos  calcular  entonces  cuántos  átomos  de  cobre  constituyen  1  g  de  cobre.  Para  ello 
primeramente calcularemos el número “N” de moles de cobre equivalentes a 1 g. La masa molar “M” del 
cobre (también nombrada “peso atómico” o “peso molecular”) es  
 
g
    M = 63.5  
mol
 
de tal manera que un gramo de cobre equivale a los siguientes moles de cobre: 
 
m 1g
    N= = =  0.01575 mol 
M 63.5 g
mol
 
Cada  mol  de  cobre  contiene  un  número  de  átomos  igual  al  número  de  Avogadro,  NA,  de  modo  que  el 
número de átomos contenidos en 1 g de cobre es 
 
    número de átomos = número de moles × número de Avogadro = 
 
1
    = N ⋅ NA = 0.01575 mol⋅ 6.02 (1023)   = 0.09482 (1023) 
mol
 
  Ahora multiplicamos el número de átomos por la carga positiva de cada uno, igual a 
 
    Z e = 29 ⋅ 1.6 (10–19) C = 46.4 (10–19) C 
 
Obtenemos 
 

    Carga = 46.4 (10–19) ⋅ 0.09482 (1023) = 4.4 (104) C 
 
Este  valor  de  la  carga  positiva  contenida  en  un  gramo  de  cobre  es  enorme.  Como  podemos  demostrar 
después  de  estudiar  la  siguiente  sección,  la  fuerza  con  que  se  atraerían  dos  cargas  eléctricas  tales, 
separadas un metro, sería de 
 
    1.74 (1019) newton 
 
la cual equivale a un peso de ¡más de 1018 toneladas! 
  Esto  indica  que  la  carga  en  exceso  contenida  en  un  material  es  regularmente  una  fracción 
muchísimo muy pequeña de la carga total del material. Si bien un material en su estado neutro posee una 
gran cantidad de carga positiva y negativa, existe un balance muy preciso de ambos tipos de carga. 
‐‐‐‐‐‐‐‐‐‐‐‐‐‐‐‐‐‐‐‐‐‐‐‐‐‐‐‐‐‐‐‐‐‐‐‐ 
1‐3 
 
 
  La  deficiencia  o  exceso  de  carga  eléctrica  dentro  en  un  material  se  comporta  de  distinta  forma 
dependiendo de si el material es lo que se llama un aislante o un conductor. 
  El vidrio es un material aislante. Si se frota una barra de vidrio con un pañuelo de seda, la barra se 
carga eléctricamente (en este proceso la barra adquiere típicamente una carga del orden de 10–18  C). Esta 
carga así adquirida permanece en la superficie donde se crea. Si se toca con un dedo dicha superficie cargada, 
la carga alojada en la porción de superficie tocada fluye por el dedo a través del cuerpo hasta tierra, y el 
lugar  que  ocupaba  esta  carga  ya  no  queda  cargado.  En  los  materiales  aislantes,  la  carga  eléctrica  no 
balanceada no puede moverse libremente a través del material, sino que permanece en el lugar donde ha 
sido creada. 
  En  cambio,  en  los  materiales  conductores  cualquier  carga  eléctrica  en  exceso  (típicamente 
electrones)  inmediatamente  se  disemina  a  través  del  conductor,  hasta  llegar  a  la  superficie  del  mismo, 
donde  eventualmente  llega  al  reposo.  Las  cargas  eléctricas  en  exceso  son  libres  de  moverse  por  todo  el 
conductor.  Si  se  deposita  cierta  carga  eléctrica  (digamos  electrones)  en  el  interior  de  un  conductor,  la 
repulsión mutua entre estas cargas motiva que se alejen mutuamente, lo cual pueden hacer dado que nada 
les  impide  moverse  a  través  del  material.  Las  cargas  se  siguen  moviendo  hasta  llegar  a  la  superficie  y 
desarrollarse allí una situación de equilibrio. 
  Consideremos por ejemplo un trozo de cobre, el cual es un conductor excelente. El número atómico 
del cobre es Z = 29 (posee 29 protones en el núcleo, y 29 electrones en su estado neutro). El átomo de cobre 
posee  2  electrones  en  su  primera  capa,  8  en  la  segunda  y  18  en  la  tercera.  En  su  última  capa  posee 
solamente 1 electrón. Este electrón se denomina electrón libre o electrón de conducción, debido a que está 
muy  débilmente  ligado  al  resto  del  átomo  (prácticamente  libre)  y  bajo  la  influencia  de  algún  campo 
eléctrico en el interior del conductor, incluso muy débil, es capaz de desligarse del átomo y viajar por todo 
el conductor. 
  Mediante  un  mango  aislante  sostengamos  una  barra  de  cobre  cargada  eléctricamente.  Al  tocar  la 
barra de cobre con un dedo, la barra pierde inmediatamente toda su carga. 
  Una propiedad importante de los conductores es la denominada densidad de electrones libres que, 
como  su  nombre  lo  indica,  es  el  número  de  electrones  libres  por  unidad  de  volumen  que  contiene  el 
material. El aluminio, cuyo número atómico es Z = 13, posee tres electrones libres en su última capa, de tal 
manera  que  la  densidad  de  electrones  libres  del  aluminio  es  (muy  aproximadamente)  tres  veces  la 
densidad de átomos del mismo. La densidad de electrones libres de una sustancia se puede medir en el 
laboratorio usando el Efecto Hall (El cual discutiremos en el Capítulo VII). 
 
1‐4 
 
 
EJEMPLO 1.2.  La densidad de electrones libres del cobre es  n = 8.5 (1028) / m3. Calcular la masa de un 
trozo de cobre que posee 18 (1029) electrones libres. 
‐‐‐‐‐‐‐‐‐‐‐‐‐‐‐‐‐‐‐‐‐‐‐ 
  Dado que cada átomo de cobre contribuye un electrón libre, tenemos que el trozo posee un número 
de átomos de cobre igual al número de electrones libres. 
  Calculemos  primeramente  el  número  de  moles  de  cobre  que  tenemos,  dividiendo  el  número  de 
átomos de cobre por el número de Avogadro: 
 
18(10 24 )
    = 30 mol  
6(10 23 ) / mol
 
  Ahora bien, la masa molar del cobre es 
 
kg
    M = 63.5(10 −3 )  
mol
 
Multiplicando el número de moles por la masa molar M obtenemos la masa que representan estos moles: 
 
kg
    m = 30 mol ⋅ M = 30 mol ⋅ 63.5(10−3 )  = 1.9 kg 
mol
 
EJEMPLO  1.3.    Se  tiene  un  trozo  de  material  con  las  siguientes  propiedades:  densidad  de  masa  ρm, 
volumen “V” y masa molar “M”. Calcular el número de átomos (o entidades) contenidos en el trozo. 
‐‐‐‐‐‐‐‐‐‐‐‐‐‐‐‐‐‐‐‐‐‐‐‐‐‐‐‐‐‐‐‐‐‐‐‐‐‐‐‐ 
  Sabemos  que  un  mol  del  material  contiene  6.02  (1023)  átomos  o  entidades  (este  es  el  número  de 
Avogadro, NA, cuyas unidades son “1/mol”). Calculemos entonces cuántos moles “N” representa el trozo. 
Primeramente obtenemos la masa del trozo, “m”, usando la densidad de masa ρm y el volumen V: 
 
    m = ρm V 
 
Ahora obtenemos el número de moles N dividiendo la masa m por la masa molar M: 
 
m ρm V kg
    N= =       (Balance de unidades:     mol = = mol ) 
M M kg / mol
 
Multiplicando el número de moles por el número de Avogadro obtenemos el número de átomos): 
 
ρm V
    n átomos = N ⋅ N A = ⋅ NA  
M
 

Note aquí el balance de unidades:   1 =
( kg / m 3 ) ⋅ m 3

1

kg / mol mol
1‐5 
 
 
1.2. LA LEY DE COULOMB. 
  Sean q1 y q2 dos cargas eléctricas puntuales de cualquier signo. 
  Sean:  R el vector separación entre las cargas, (Fig. 1),  R = |R| su magnitud (es decir, la distancia 
entre las cargas), y  R̂  el vector unitario en la dirección de R. 
  Sea F la fuerza eléctrica que ejerce q1 sobre q2. 

 
Fig. 1 
 
  La Ley de Coulomb establece que esta fuerza viene dada por la expresión 
 
  Ley de Coulomb (Forma vectorial) 
1 q1q 2 R Fuerza que ejerce la carga q1 sobre la carga q2. 
(3)    F=  
4πε0 R 3 El vector separación R va desde la carga q1 hasta 
  la carga q2. 
 
  Si ambas cargas son del mismo signo, la fuerza eléctrica entre ellas tiene la misma dirección que R y 
por tanto es de repulsión (caso representado en la Fig. 1). Las cargas se atraen si son de signos opuestos. 
  La fuerza eléctrica (3) obedece la tercera ley de Newton (al intercambiar en ella los indices 1 y 2, y 
cambiar R por –R, la fuerza cambia de signo). 
  La magnitud de la fuerza es 
 
1 q 1q 2 Ley de Coulomb. 
(4)    F=      
4 πε0 R 2 Magnitud de la fuerza entre dos cargas q1 y q2. 
 
R R R 1
(Esta expresión se saca de (3) tomando magnitudes. Note que  3
= 3 = 3 = 2 ). 
R R R R
Como  vemos  en  (3)  o  (4),  la  fuerza  eléctrica  entre  dos  cargas  puntuales  es  proporcional  a  las  cargas  e 
inversamente proporcional al cuadrado de la distancia entre ellas. 
  El factor de proporcionalidad, “1/4πε0”, se suele abreviar así: 
1 N ⋅ m2
(Valor experimental:  k = 8.988(10 )
9
(5)    k=            ) 
4 πε0 C2
 

En los cálculos usaremos el valor aproximado 
 

N ⋅ m2
(6)    k = 9(109 )
C2  
1‐6 
 
 
  Así pues, las expresiones (3) y (4) se pueden escribir también: 
 
q 1q 2 R q1q 2
(7)    F=k 3
F=k  
R R2
 
  La fórmula (3) es una expresión vectorial, es decir, válida en cualquier sistema de coordenadas. Para 
efectuar un cálculo es necesario ya definir un sistema de coordenadas. 
 
EJEMPLO  1.4.  ¿Qué  cargas  (iguales)  deberían  tener  la  Tierra  y  la  Luna  para  que  la  fuerza  eléctrica  de 
atracción entre ellas fuese igual a su fuerza gravitatoria? 
‐‐‐‐‐‐‐‐‐‐‐‐‐‐‐‐‐‐‐‐‐‐‐‐‐‐‐‐‐‐‐‐‐‐‐‐‐ 
  Sea  Q  la  carga  de  la  Tierra  y  –Q  la  de  la  Luna.  La  fuerza  eléctrica  entre  ellas,  consideradas  como 
cargas puntuales dado la gran distancia que las separa, es 
 
Q2
    Fe = −k  
d2
 
donde “d” es la distancia Tierra‐Luna. 
  Por otra parte, la fuerza gravitatoria es 
 
MT m L
    Fg = −G  
d2
 
donde MT y mL son las masas de la Tierra y la Luna, respectivamente. 
  Igualando ambas fuerzas, 
 
Q2 M m
    −k 2
= −G T 2 L  
d d
 
  De aquí sacamos 
 
GMT m L
    Q=  
k
 
Sustituyendo valores: 
 
    G = 6.67 (10–11) N‐m2/kg2     k = 8.988 (109) N‐m2/C2 
    MT = 5.98 (1024) kg       mL = 7.36 (1022) kg 
 
Se encuentra   Q = 5.71 (1013) C 
1‐7 
 
 
EJEMPLO  1.5.  Dos  esferitas  metálicas  que  portan  la  misma  carga  eléctrica  negativa,  y  cuya  distancia 
mutua  d = 2 m es mucho mayor que sus radios, se repelen con una fuerza  F = 12 mN (Véase la Fig. 2). 
¿Cuántos electrones en exceso tiene cada esferita? 
  Sea  “q”  la  carga  de  cada  esferita.  Según  la   
Ley de Coulomb, la fuerza de repulsión entre ellas 
tiene magnitud 
   
kq 2  
    F=  
Fig. 2 
d2
 
donde 
 
1 N ⋅ m2
    k= ≈ 9(10 9 )  
4 πε0 C2
 
Despejando la carga “q” tenemos 
 
F 12(10 −3 )N
    q=d = 2m = 2.31(10 −6 )C  
k N⋅m 2
9(109 )
C2
 
Dividiendo  “q”  por  la  carga  del  electrón,  e  =  1.6  ×  10–19  C,  obtenemos  el  número  N  de  electrones  en 
exceso que posee cada esferita: 
 
q 2.31(10 −6 )C
    N= = −19
= 1.44(1013 ) = 14.4(1012 )  
e 1.6(10 )C
 
 
1‐8 
 
 
EJEMPLO 1.6. Una carga positiva de 8 μC y una negativa de – 5 μC se hallan en los puntos indicados en 
la cuadrícula mostrada en la Fig. 3. Suponer que el   
lado de la cuadrícula mide  1  m y calcular la fuerza 
eléctrica  F  que  ejerce  la  carga  positiva  sobre  la 
negativa. 
  Emplearemos la expresión (3), o sea 
 
1 q1q 2
    F= R 
4πε0 R 3
 
Debemos tomar  
 
    q1 = 8 μC 
    q2 = – 5 μC   
Fig. 3 
 
  Recordemos que el vector  R se dirige hacia la carga que sufre la fuerza a calcular,  F. Obtengamos 
los vectores de posición de ambas cargas: 
 

    r’1 = (11, 5) m    r’2 = (2, 10) m 
 

  Ahora obtengamos el vector separación R: 
 

    R = r’2 – r’1 = (2, 10) m – (11, 5) m = (–9, 5) m 
 

(Estas componentes podrían haberse obtenido también gráficamente de la Fig. 3, contando cuadritos). 
  El cubo de su magnitud es (en metros) 
 

( )
3
    R3 = 9 2 + 52 =  1091.33 
 

  Sustituyendo valores (en unidades S.I.): 
 
−6 −6
1 q 1q 2 9 8(10 ) ⋅ ( −5(10 ))
    F= R = 9(10 ) ( −9,5) = 0.33(10 −3 ) ⋅ ( −9,5)
4 πε0 R 3 1091.33  
 
⇒     F = (2.97 ⋅ 10– 3   , 1.65 ⋅ 10– 3) N = (2.97 , 1.65) mN 
1‐9 
 
 
EJEMPLO 1.7. Dado el arreglo de cargas puntuales mostrado en la Fig. 4, ¿Dónde debe estar situada la 
carga “q” para que la fuerza eléctrica sobre ella debida a las otras dos cargas sea nula? 
 

 
 
Fig. 4 
‐‐‐‐‐‐‐‐‐‐‐‐‐‐‐‐‐‐‐‐‐‐‐‐‐‐‐‐‐‐‐‐‐‐ 
  Tomemos un Eje X con su origen en la carga  4q y su dirección hacia la derecha. Está claro que la 
carga  “q”  debe  encontrarse  a  la  derecha  de  la  carga  “–q”,  donde  las  fuerzas  eléctricas  sobre  ella  tienen 
sentidos opuestos y pueden anularse. 
  Para resolver el problema obtendremos las magnitudes de las fuerzas (opuestas) sobre “q” debidas 
a las otras dos cargas y luego las igualaremos. La magnitud de la fuerza que ejerce la carga “4q” es 
 

4q ⋅ q 4q 2
    F1 = k =k  
x2 x2
 

y la magnitud de la fuerza que ejerce la carga “–q” es 
 

q ⋅ ( −q) q2
    F2 = k =k  
(x − a)2 (x − a)2
 

Igualando ambas fuerzas tenemos 
 

4q 2 q2
    k =k  
x2 (x − a)2
 

Cancelando factores comunes se tiene 
 
4 1
    =  
x 2
(x − a)2
 

Quitando denominadores, 
 
    4(x − a)2 − x2 = 0  
 
    3 x2 – 8 a x + 4 a2 = 0 
 
Las raíces de esta ecuación son 
 
2
    x = 2a y x= a 
3
 
La solución correcta es la primera raíz: “x = 2a”. Para la segunda raíz, x = (2/3) a, también hay igualdad de 
las magnitudes de las fuerzas, pero no son opuestas y no se cancelan. 
1‐10 
 
 
EJEMPLO 1.8. En el modelo de Bohr del átomo de hidrógeno, el electrón en su estado base describe una 

órbita circular de radio 0.529 (10–10) m alrededor del protón. Calcular la velocidad del electrón. 
 

 
 

Fig. 5 
‐‐‐‐‐‐‐‐‐‐‐‐‐‐‐‐‐‐‐‐‐‐‐‐‐‐‐‐‐‐‐‐‐‐‐‐‐‐‐‐‐‐‐‐ 
  El electrón realiza un movimiento circular uniforme con rapidez constante v. La fuerza Fr que sufre 
es la atracción eléctrica debida al protón, la cual es una fuerza centrípeta cuya componente radial (única) 
es 
 

1 e2
    Fr = −  
4 πε0 r 2
 

  De acuerdo con la 2ª. Ley de Newton, esta fuerza debe ser igual al producto de la masa del electrón 
y su aceleración centrípeta, o sea 
 
    Fr = m ar 
 
Usando la expresión para la aceleración centrípeta, o sea 
 
v2
    ar = −  
r
 
tenemos 
e2 ⎛ v2 ⎞
    −k = m ⎜⎜ − ⎟⎟  
r2 ⎝ r ⎠
 
Despejando la velocidad, 
 
ke 2
    v=  
mr
 
Sustituyendo valores en unidades S.I., 
 
9(109 ) ⋅ (1.6 ⋅ 10 −19 )2 m
    v= −31 −10
=  2.18 (106)        ( ) 
9.11(10 ) ⋅ 0.529(10 ) s
1‐11 
 
 
EJEMPLO  1.9.  Dos  esferitas  cargadas,  de  la  misma  masa  m  =  40  g,  están  suspendidas  mediante  hilos 
iguales  y  aislantes  de  un  punto  común,  como  muestra  la  Fig.  6  izquierda.  Se  observa  que  cuando  el 
sistema  está  en  equilibrio,  el  ángulo  θ1  que  forma  el  hilo  izquierdo  con  la  vertical  es  θ1  =  8°.  Dada  la 
longitud  de  cada  hilo,  L = 20 cm,  y  la  carga  de  la  esferita  izquierda,  q1 = 6 μC,  calcular  la  carga  de  la 
esferita derecha. 
 

 
Fig. 6 
 
  Para simplificar el problema,  propondremos la siguiente hipótesis: en el equilibrio, los ángulos θ1 y 
θ2 que forman los hilos con la vertical son iguales. ¿Por qué nuestra hipótesis? Tenemos que las masas de 
las  esferitas,  y  por  ende  sus  pesos,  son  iguales;  por  otra  parte,  las  fuerzas  eléctricas  de  repulsión  entre 
ambas también son iguales (y opuestas), ya que estas fuerzas obedecen la tercera ley de Newton. 
  La Fig. 6 derecha muestra el diagrama de cuerpo libre (DCL) de las esferitas. Sobre cada una actúan 
las siguientes fuerzas: la eléctrica Fe (fuerza de repulsión); la tensión del hilo, T, y el peso mg. Note que si 
las masas de las esferitas fuesen distintas, se destruiría la simetría de fuerzas. 
  Tomando unos Ejes X y Y horizontal y vertical, respectivamente, tenemos las siguientes ecuaciones 
de equilibrio para la esferita izquierda: 
 

   
∑ Fx = 0 : − Fe + T sen θ1 = 0
 
∑ Fy = 0 : − mg + T cos θ1 = 0
 
  En unidades S.I., la distancia entre las esferitas es  d = 2 L sen θ1 = 0.05567  y la fuerza eléctrica es 
 
q 1q 2 6(10 −6 )q 2
    Fe = k 2
= 9(10 9 ) −2 2
= 1.74(107 )q 2  
d (5.56(10 ))
 
De las ecuaciones de equilibrio sacamos  Fe = mg tan θ1, de modo que 
 
    1.74(107 )q 2 = 0.04 ⋅ tan(8°) = 0.00562 = 5.62(10 −3 )  
 
5.62(10 −3 )
    q2 = 7
= 3.23(10 −10 )      o bien     q2 = 0.000323 μC 
1.74(10 )
1‐12 
 
 
1.3. DEFINICIÓN DEL CAMPO ELÉCTRICO. 
  Sea  D    una  distribución  arbitraria  de  carga  eléctrica.  Puede  ser  un  sistema  de  cargas  puntuales,  o 
carga repartida por una región espacial tridimensional, o sobre una superficie, o a lo largo de un alambre 
o  filamento;  es  completamente  arbitraria.  Lo  único  que  exigimos  es  que  el  cuerpo  material  que  aloja  la 
carga  esté  fijo,  y  que  además  su  carga  eléctrica  no  se  mueva  dentro  de  él,  esto  es,  que  no  existan 
influencias externas importantes que “redistribuyan” su carga por su interior. 
  Sea “q” una carga puntual, pequeña y positiva. 
 

 
 

Fig. 7 
 
  Se define el vector de intensidad eléctrica E en el punto P, producido por la distribución D , como la 
fuerza eléctrica por unidad de carga que actúa sobre la carga puntual en P. En símbolos, 
 
  El  vector  de  intensidad  eléctrica  producido  por  una  distribución  de 
F carga en un punto P es la fuerza eléctrica por unidad de carga que ejerce 
(8)    E=  
q la distribución sobre una pequeña carga puntual positiva colocada en P. 
 
 
Comentarios. 
• La carga “q” que interviene en la definición del campo eléctrico se denomina para este efecto carga 
testigo. 
• Es común llamar simplemente campo eléctrico  E al vector de intensidad eléctrica. Así lo haremos en 
lo sucesivo. 
• El campo eléctrico E es un vector que tiene la misma dirección que la fuerza F (Fig. 7). 
• Las unidades físicas de E son, de (8), 
 
newton N
    =  
coulombio C
 
• La carga testigo debe ser pequeña para que no afecte la distribución de carga  D, cuyo campo deseamos 
medir.  Existe  un  fenómeno  llamado  “inducción  electrostática”,  que  consiste  en  una    separación  o 
reacomodo de cargas en un cuerpo (metálico o dieléctrico), provocada por la cercanía de otra carga. 
1‐13 
 
 
  En  la  Fig.  8  se  muestra  una  esfera  metálica   
apoyada  sobre  una  varilla  aislante.  Inicialmente  la   
esfera  estaba  lejos  de  cualquier  carga  eléctrica,  y 
sus cargas eléctricas atómicas positivas y negativas 
estaban  uniformemente  distribuidas.  Al  acercar 
una  carga  negativa  Q  a  la  esfera  se  producen 
repulsiones  entre  Q  y  los  electrones  libres  de  la 
esfera.  El  efecto  consiste  en  una  separación  o   
“polarización”  de  las  cargas,  como  se  ve  en  esta   
figura. Suponemos la carga testigo tan pequeña que   
no provoque una inducción apreciable.  Fig. 8 
   
• La definición dada de E sirve para medir el campo eléctrico fuera de la distribución D (estrictamente en 
el vacío), pues no podemos meter una carga testigo en el interior de  D  sin alterar su distribución de 
carga.  Existen  otras  definiciones  operacionales  (esto  es,  basadas  en  un  procedimiento  de  medición) 
aplicables al interior de la materia, pero no las daremos ahora. 
 
• La ecuación definitoria (8) se usa mucho en la forma 
 
  Fuerza eléctrica sobre una carga puntual q. 
(9)    F = q E  La  fuerza  eléctrica  está  en  la  dirección  del  campo,  u  opuesta  a 
  ésta,  según  que  la  carga  “q”  sea  positiva  o  negativa,  respecti‐
vamente. 
 
En (9), “q” ya no es una carga “testigo”, sino cualquier carga situada en la vecindad de la distribución 
D. Esta  q ya puede ser positiva o negativa. El movimiento de “q” dentro del campo eléctrico E se rige 
por la segunda ley de Newton en la forma “masa x aceleración = fuerza” o sea 
 
  Expresión matemática de la segunda ley de Newton 
 
para una partícula de masa “m” y carga eléctrica “q” 
(10)    m a = q E 
en el seno de un campo eléctrico E. 
 
“a” es el vector aceleración de la partícula. 
 
La carga “q” experimenta dentro del campo una aceleración dada por 
 
F qE
    a= =  
m m
 
  Las  ecuaciones  (9)  y  (10)  gobiernan  el  movimiento  de  partículas  cargadas  dentro  de  un  campo 
eléctrico. 
1‐14 
 
EJEMPLO 1.10. Una distribución de carga eléctrica fija produce un campo eléctrico  E. (a) Se observa que 
una partícula de carga q = 15 μC y masa m = 4 (10–8) kg sufre en cierto punto una aceleración, debida al 
campo  eléctrico,  de  magnitud  a  =  2.5  (104)  m/s2.  ¿Cuánto  vale  el  campo  eléctrico  en  dicho  punto?  (b) 
¿Qué fuerza experimenta otra carga  q = 9 μC cuando se encuentra en un punto donde el campo eléctrico 
tiene una magnitud E = 105 N/C? 
 

 
Fig. 9 
 
(a)  Dada la masa y la aceleración de la partícula, podemos calcular la fuerza sobre ella empleando la 
segunda Ley de Newton: 
 
m
    F = ma ⇒ F = 4(10−8 )kg ⋅ 2.5(10 4 ) = 10 −3 N  
s2
 
Ahora podemos calcular el campo eléctrico usando su definición (8) en la forma 
 
F 10 −3 N N
    E= = −6
= 66.6  
q 15(10 )C C
 
La dirección de este campo es la misma que la de la fuerza o la aceleración en el punto considerado. 
 
(b)  La magnitud de la fuerza se calcula de la expresión (9) (tomando magnitudes): 
 
    F = q E 
 
Obtenemos 
 
N
    F = 9μC ⋅ 105 = 945 ⋅ 10 −6 N  
C
 
La dirección de esta fuerza es la misma que la del campo eléctrico E en el punto considerado. 
1‐15 
 
 
1.4. CAMPO ELÉCTRICO DE UNA CARGA PUNTUAL. 
  Observe  la  Fig.  10.  Deseamos  obtener  el  campo  eléctrico  producido  por  la  carga  puntual  Q  en  el 
punto arbitrario  P. Recuerde nuestra convención: el punto donde se localiza la carga es el punto fuente, y 
el punto P es el punto campo. El vector separación desde Q hasta P es R. 
  De  acuerdo  con  la  definición  dada  del  campo  eléctrico,  debemos  obtener  la  fuerza  eléctrica  por 
unidad de carga que ejercería Q sobre una pequeña carga puntual positiva q situada en P. 
  Según la Ley de Coulomb, tal fuerza F sería   
 
Qq ˆ = Q q R   
    F= R
4 πε0 R2
4 πε0 R 3
 
De  acuerdo  con  la  definición  (8),  la  fuerza  por  unidad 
de carga, es decir, el campo eléctrico E, es   
 
Fig. 10 
 
 
Campo  eléctrico  de  una  carga  puntual  Q  en  un 
F QRˆ QR
(11)    E= = =   punto cuyo vector separación desde la carga es R. 
q 4 πε0 R 2
4 πε0 R 3
 
 
Observaciones: 
• Si  Q  es  positiva,  el  campo  E  está  dirigido  en  la  dirección  radial  hacia  fuera  desde  Q.  Sus  líneas  de 
fuerza son líneas radiales que emanan de Q (“huyendo” de la misma), como vemos en la Fig. 11. 
Si Q es negativa, el campo E apunta hacia ella: las líneas de fuerza se dirigen hacia Q.  
Esto es válido en todo punto P del espacio. 

 
Fig. 11 
 
1‐16 
 
EJEMPLO 1.11.  Una carga puntual Q = 5 (10 – 5) C está situada en el punto de coordenadas cartesianas 
(6, 1) (coordenadas en metros). Calcular el campo de esta carga en el punto P(–3, 8) (Véase la Fig. 12; cada 
lado de la cuadrícula representa un metro). 
 

 
Fig. 12 
 
  El campo eléctrico es la expresión (11): 
 
QR |Q|
[(11)]      E=     (Magnitud:     E= E = ) 
4 πε0 R 3
4πε0 R 2
 
  Hay  dos  métodos  equivalentes  de  calcular  el  campo  E  en  P.  En  el  primer  método  calculamos 
primeramente la magnitud del vector  E, y luego sacamos sus componentes X y Y, expresando el campo en 
la forma  E = (Ex, Ey) o en la forma  E = Ex i + Ey j. En la segunda manera procedemos desde el principio 
vectorialmente, obteniendo el vector R y haciendo la operación vectorial indicada en (11). Lo haremos con 
los dos métodos en este ejemplo. 
 
Primer método. 
  Como vemos en la Fig. 12, la distancia desde Q hasta el punto P es, en metros, 
 
    R = 9 2 + 7 2 = 130 = 11.4   (m) 
 
de modo que la magnitud del campo eléctrico en P es 
 
Q 5 ⋅ 10 −5
    E= E = = 9 ⋅ 109 ⋅ =  3462.60      (N/C) 
4 πε0 R 2 11.4 2
 
Ahora bien, el ángulo agudo “α” que forma E con el Eje X, mostrado en la Fig. 12, es 
1‐17 
 
⎛ 9 ⎞
    α = ang cos ⎜ ⎟ = 37.86°  
⎝ 11.4 ⎠
 
de tal manera que las componentes del campo E, con los signos correctos, son 
 
    Ex = – E cos 37.86° = – 3462.60 cos 37.86 = – 2733.77    (N/C) 
 
    Ey = E sen 37.86° = 3462.60 sen 37.86 = 2125.11      (N/C) 
 
N N
⇒     E = (– 2733.77, 2125.11)   = (– 2733.77 i + 2125.11 j)   
C C
 
Segundo método. 
  Las componentes del vector separación son  R = (– 9, 7) y su magnitud es  R = 11.4. Sustituyendo 
esto en (11) tenemos 
 
QR 5 ⋅ 10 −5 ( −9,7)
    E= = 9 ⋅ 109 ⋅    (N/C) 
4 πε0 R 3 (11.4)3
 
Efectuemos las operaciones indicadas en esta expresión vectorial. 
Usando 
 
5 ⋅ 10 −5
    9 ⋅ 109 ⋅ =  303.74 
(11.4)3
 
obtenemos 
 
    E = 303.74 ( −9,7) = (303.74 × −9, 303.74 × 7) =  (– 2733.66,  2126.18)     (N/C) 
 
Excepto por errores de redondeo, este es el mismo resultado que obtuvimos con el primer método. 
  Se recomienda usar preferentemente el segundo método (vectorial). 
 
1‐18 
 
 
1.5. CAMPO ELÉCTRICO DE UN SISTEMA DE CARGAS PUNTUALES. 
  En  la  sección  1.4  obtuvimos  la  expresión  para  el  campo  eléctrico  de  una  carga  puntual  Q  en  un 
punto arbitrario P cuyo vector separación relativo a Q es R (Fig. 5), a saber, 
   
QR
[(11)]          E=  
4 πε0 R 3
 
  Existe  en  electromagnetismo  un  principio 
fundamental  denominado  principio  de 
superposición de los campos eléctricos, que enuncia   
lo siguiente:   
[Fig. 5] 
 
Principio de superposición 
  El campo eléctrico producido por un conjunto de cargas es igual a la suma vectorial de los campos 
eléctricos individuales producidos por cada carga. 
 
  Apliquemos este principio para calcular el campo eléctrico de un conjunto de cargas puntuales  Q1, 
Q2, …, QN en un punto P arbitrario. 
  El  campo  eléctrico  Ei  producido  por  cada   
carga Qi (i = 1, 2, …, N) tiene la forma dada en (11), 
esto es, 
 
k Qi R i
    Ei =   (i = 1, 2, …, N) 
R i3
 
donde hemos abreviado 
   
1 Fig. 13 
    k=  
4 πε0
 
y Ri es el vector separación desde la carga Qi hasta el punto dado P. 
  Obtenemos así para el campo de las N cargas la expresión 
 
 
N Campo eléctrico de un sistema de 
k Q1 R1 k Q2 R 2 k QN R N k Qi R i
(12)    E=
R 13
+
R 32
+ ... + 3
RN
= ∑ R i3
  cargas puntuales Q1, Q2, …, QN. 
i =1

 
 
 
1‐19 
 
EJEMPLO 1.12.  Cuatro cargas de valores 5 μC, 2 μC, −7μC y 3 μC están colocadas tal como se muestra en 
la  Fig.  14.  La  cuadrícula  consta  de  cuadrados  de  lado  1  m.  Calcular  el  campo  eléctrico  E  debido  a  las 
cuatro cargas en el punto P. 
  Este  es  de  hecho  un  ejercicio  de   
álgebra  vectorial.  Debemos  efectuar  el 
cálculo indicado en la Ec. (12). 
  Numeremos  las  cargas  desde  1 
hasta 4 en un orden arbitrario, y tracemos 
los  vectores  separación  desde  cada  carga 
hasta el punto campo P. 
  Habrá  que  obtener  los  vectores  R1, 
R2,  R3  y  R4  junto  con  sus  magnitudes, 
sustituir  en  la  Ec.  (12),  y  hacer  las 
operaciones vectoriales indicadas.   
  Conviene  hacer  el  cálculo  en  coor‐ Fig. 14 
denadas cartesianas. Independientemente 
de la ubicación del origen de coordenadas, tenemos (en metros): 
 
    R1 = (7, −2)         R2 = (0, 6)           R3 = (4, 3)           R4 = (−7, −1) 
 

Además, 
 
3
    ( )
R13 =  7 2 + ( −2) 2 2 = 385.8,     R23 = 216,      R33 = 125,    R43 =  353.5 
 

Entonces, 
 
4
2(7, −2) 5(0,6) −7(4,3) 3( −7, −1) ⎞
∑ = 9(10)9  ⋅10 −6 ⋅ ⎛⎜
Qi R i
    E=k + + + ⎟
R
i =1 i
3 ⎝ 385.8 216 125 353.6 ⎠  

 
⎛ (126, −36) (0,270) ( −252,189) ( −189, −27) ⎞
        = 10 3 ⎜ + + + ⎟ 
⎝ 385.8 216 125 353.6 ⎠
 
        = (326.5, −93.3) + (0,1250) + ( −2016, −1512) + ( −534.6, −76.7)  
 
        = ( −2224, −432)  
Tenemos así 
 
N N
    E = (−2224, −432)   , o bien   E = (−2224 i − 432 j)    
C C
 
1‐20 
 
EJEMPLO  1.13.  Esquematizar  las  líneas  de  fuerza  del  campo  eléctrico  producido  por  (a)  Una  carga 
positiva  “4q” y otra carga positiva “2q”; (b) Una carga positiva “4q” y una carga negativa “–2q”. 
‐‐‐‐‐‐‐‐‐‐‐‐‐‐‐‐‐‐‐‐‐‐‐‐‐‐‐‐‐‐‐‐‐‐‐‐‐‐ 
  Las  líneas  de  fuerza  se  muestran  en  las  Figs.  15  y  16,  respectivamente.  En  la  Fig.  15  note  que, 
comparada con la correspondiente a la carga “4q”, la densidad de líneas de fuerza en la vecindad de la 
carga “2q” es menor. Esto indica gráficamente que el campo es de menor intensidad en la vecindad de la 
carga “2q”. En la Fig. 16, no todas las líneas que salen de la carga “4q” terminan en la carga “–2q”; algunas 
líneas se desvían hacia el infinito. 
 

 
 
Fig. 15 
 

 
 
Fig. 16 
 
1‐21 
 
EJEMPLO  1.14.  Se  tienen  tres  cargas  puntuales  situadas  sobre  una  línea  recta  horizontal.  El  campo 
eléctrico de las tres cargas tiene las líneas de fuerza que se muestran en la Fig. 17. ¿Qué puede decir sobre 
los signos y magnitudes de las cargas? 
 

 
 
Fig. 17 
 
  Sabemos que las líneas de fuerza del campo eléctrico se originan en las cargas positivas y mueren 
en las negativas. En la Fig. 17, las líneas dadas no tienen dirección, así que tenemos dos posibilidades: 
(i) Las cargas izquierda y central son positivas y la carga derecha es negativa 
(ii) Las cargas izquierda y central son negativas y la carga derecha es positiva. 
  Supongamos cierta la primera posibilidad. Observemos que no todas las líneas que se originan en la 
carga central (positiva) terminan en la carga derecha (negativa); esto significa que la carga central es (en 
valor absoluto) mayor que la carga derecha. Por otra parte, las cargas izquierda y central parecen ser del 
mismo valor. 
  La  densidad  de  líneas  de  fuerza  da  una  idea  gráfica  de  la  magnitud  del  campo  eléctrico.  Así,  el 
valor relativo del campo es pequeño en el punto medio entre las cargas izquierda y central, y también a la 
derecha de la carga derecha. El campo es intenso en la cercanía de cada carga. 
1‐22 
 
 
EJEMPLO  1.15.  Dos  cargas  puntuales  positivas 
“Q” están separadas una distancia “2a”. Un punto 
P se halla sobre la recta perpendicular a la línea de 
unión  de  las cargas  (Eje  Y),  como  se  muestra  en  la 
Fig.  18.  A  qué  distancia  “y”  debe  encontrarse  tal 
punto  P para que el campo eléctrico allí, producido 
por ambas cargas, tenga un valor máximo? 
‐‐‐‐‐‐‐‐‐‐‐‐‐‐‐‐‐‐‐‐‐‐‐‐‐‐‐‐ 
  Las distancias de las cargas al punto P valen 
 
ambas 
Fig. 18 
 
    a2 + y2  
 
de tal manera que los campos eléctricos de las cargas poseen una magnitud común dada por 
 
kQ ⎛ 1 ⎞
    E1 = E2 =     ⎜k = ⎟ 
a + y2
2
⎝ 4 πε0 ⎠
 
Las  componentes  horizontales  de  E1  y  E2  se  anulan,  y  las  verticales,  a  lo  largo  del  Eje  Y,  se  suman.  El 
campo E resultante en P tiene componente Y igual a 
 
    E y = E1y + E2y = 2E1y = 2E1 sen θ  
 
donde “θ” es el ángulo que forma el campo E1 con la horizontal. Usando 
 

y
    sen θ =  
a2 + y2
 

obtenemos 
 

kQ y 2kQy
    Ey = 2 ⋅ ⋅ =  
a +y
2 2 3
a2 + y2
(a 2
+ )
y2 2
 

Los valores extremos de Ey se encuentran con la condición 
 

dE y
    =0 
dy
 

de donde se encuentra un valor mínimo del campo Ey en y = 0, y un valor máximo en  
 
    y = 2a /2 
1‐23 
 
EJEMPLO  1.16.  Una  carga  de  2  μC  se  encuentra  en  el  origen  de  coordenadas  de  un  sistema  XY.  Otra 
carga  desconocida  se  encuentra  en  x  = 1.6  m.  ¿Cuánto  debe  valer  esta  última  carga  para  que  el  campo 
eléctrico total en el punto x = 0.4 m sea nulo? 
  El  campo  eléctrico  de  la  carga  2  μC  en  el 
punto x = 0.4 m es (en unidades S.I.) 
 

k ⋅ 2(10 −6 )
    E1 =  
(0.4)2
 

y el de la carga desconocida (negativa), “–Q”, es   
Fig. 19 
k⋅Q
    E2 = −  
(1.2)2
 

Imponiendo la condición de que el campo total sea cero en el punto considerado tenemos 
 

k ⋅ 2(10 −6 ) k ⋅ Q
    E = E1 + E2 = − =0 
0.16 1.44
 

de donde sacamos 
 

2(10 −6 )
    Q = 1.44 = 18(10 −6 )       Q = 18 μC 
0.16
 
EJEMPLO  1.17.  Se  tiene  un  arreglo  de  3  cargas  puntuales    –q,  –q  y  Q,  como  se  muestra  en  la  Fig.  20. 
Exprese la carga Q en términos de q para que el campo eléctrico total en el punto P sea nulo. 
  Empleando  la  fórmula  (11)  tenemos  que  el   
campo eléctrico de la carga Q en P es 
 
kQ
  E1 = ⋅ (3s,0)  
(3s)3
 
Los  campos  de  las  cargas  negativas  superior  e 
inferior son respectivamente 
 
k( −q) k( −q)
E2 = 3
(2s, −s)     y      E2 = (2s,s)    
( 5s) ( 5s)3 Fig. 20 
 
  El campo total tiene componente Y igual a cero. La componente X del mismo es 
 
kQ 2kq 2kq kQ 4kq
    Ex = E1x + E2x + E3x = 2
− 3/2 2 − 3/2 2 = 2
− 3/ 2 2  
(3s) 5 s 5 s (3s) 5 s
 

4 ⋅ 32
Esta se anula cuando    Q = q = 3.22q  
53 / 2
1‐24 
 
 
EJEMPLO 1.18. Cuatro cargas puntuales están situadas en los vértices de un rectángulo, como se muestra 
en la Fig. 21. Calcular el campo eléctrico en el vértice superior derecho del rectángulo, debido a las cargas 
en los demás vértices. ¿Qué fuerza experimentaría una carga puntual de –2 μC colocada en dicho vértice? 
Use los valores q = 1.2 mC y  s = 0.8 m. 
 

 
Fig. 21 
 
  Usaremos la expresión (12‐p14), que para 3 cargas se escribiría: 
 
k Q1 R1 k Q2 R 2 k Q3 R 3
E= + +
R 13 R 32 R 33
     
 

R1,  R2 y  R3 son los vectores separación que van desde cada carga hasta el punto campo (vértice superior 


derecho).  Adoptando  un  sistema  XY  con  sus  ejes  paralelos  a  los  lados  del  rectángulo,  tendremos 
(numerando las cargas arbitrariamente): 
 

    Q1 = 5q    R1 =2s i      R1 = 2s 


    Q2 =  –3q    R2 = 2s i + s j         R 2 = (2s)2 + s 2 = 5 s  
    Q3 = 2q    R3 = s j      R3 = s 
 

k ⋅ 5q(2s i) k ⋅ ( −3q)(2si + sj) k ⋅ 2q(sj)


    E= 3
+ 3
+  
(2s) ( 5s) s3
 

5kq 3kq(2i + j) 2kq


    E= 2
i− + j 
(2s) 3 2
5 s s2
 

kq ⎛ 5 6 ⎞ kq kq ⎛ 3 ⎞ kq
    Ex = ⎜
2 ⎜
− ⎟⎟ = 0.71 2 Ey = ⎜
2 ⎜
− + 2 ⎟⎟ = 1.73 2  
s ⎝4 3
5 ⎠ s s ⎝ 5 3
⎠ s
 

9(109 ) ⋅ 1.2(10 −6 ) 9(109 ) ⋅ 1.2(10 −6 )


    Ex = ⋅ 0.71 = 11981 Ey = ⋅ 1.73 = 29193  
0.82 0.8 2
 

  La fuerza sobre una carga “–2 μC”  colocada en el vértice superior izquierdo sería 
 

  Fuerza = Carga × Campo eléctrico =  
 
    = –2 (10–6) (11 981, 29 193) = (0.023962, 0.058386)           (newton) 
1‐25 
 
 
1.6. EL DIPOLO ELÉCTRICO. 
  El  dipolo  eléctrico  es  un  sistema  de  dos  cargas  puntuales  −Q  y  Q,  separadas  por  cierto  vector 
separación a, como vemos en la Fig. 22. 
 
 
 

 
 

 
 
Fig. 22  Fig. 23 
 

  El  dipolo  eléctrico  es  importante.  Se  usa  en  la  descripción  de  los  campos  eléctricos  en  la  materia, 
pues las moléculas se pueden modelar en primera aproximación como dipolos eléctricos. 
  En  la  Fig.  23  hemos  introducido  unos  símbolos  y  vectores  para  calcular  el  campo  del  dipolo 
eléctrico.  No  perdemos  generalidad  si  suponemos  que  las  cargas  y  el  punto  campo  P  son  coplanarios. 
Calcularemos E en un punto P arbitrario del plano. Apliquemos la fórmula (12), 
 
k( −Q)R1 kQR 2
    E= +  
R 13 R 32
 
Ahora bien, expresemos R1 y R2 en la base cartesiana, suponiendo P localizado en (x, y): 
 
a a
    R1 = (x, y + ) , R 2 = (x, y − )  
2 2
 
1 1
⎛ ⎛ a ⎞ ⎞2
2 ⎛ ⎛ a ⎞ ⎞2
2
    R1 = ⎜ x2 + ⎜ y + ⎟ ⎟ , R 2 = ⎜ x2 + ⎜ y − ⎟ ⎟  
⎜ ⎝ 2 ⎠ ⎟⎠ ⎜ ⎝ 2 ⎠ ⎟⎠
⎝ ⎝
 

de tal manera que las componentes X y Y del campo eléctrico son: 
 

(13a)  (13b) 
⎛ ⎞ ⎛ ⎞
⎜ ⎟ ⎜ ⎟
⎜ ⎟ ⎜ a a ⎟
y+ y−
E x = kQ ⎜ − ⎟  ⎜ ⎟ 
x x 2 2
+ E y = kQ − +
⎜ 3 3 ⎟ ⎜ 3 3 ⎟
⎜ ⎛ 2 ⎛ a⎞
2 ⎞2 ⎛ 2 ⎛ a⎞
2 ⎞2 ⎟ ⎜ ⎛ 2 ⎛ a⎞
2 ⎞2 ⎛ 2 ⎛ a⎞
2 ⎞2 ⎟
⎜⎜ ⎜⎜ x + ⎜ y + ⎟ ⎟⎟ ⎜⎜ x + ⎜ y − ⎟ ⎟⎟ ⎟⎟ ⎜⎜ ⎜⎜ x + ⎜ y + ⎟ ⎟⎟ ⎜⎜ x + ⎜ y − ⎟ ⎟⎟ ⎟⎟
⎝ ⎝ ⎝ 2 ⎠ ⎠ ⎝ ⎝ 2⎠ ⎠ ⎠ ⎝ ⎝ ⎝ 2 ⎠ ⎠ ⎝ ⎝ 2⎠ ⎠ ⎠
 
  A  continuación  simplificaremos  estas  expresiones  para  el  caso  en  que  el  punto  campo  está  muy 
alejado del dipolo (es decir, a << x, a << y). 
1‐26 
 
  Definamos las funciones 
 

x y
    f(x, y) = 3
, g(x, y) = 3
 
( )
x2 + y2 2 (x 2
+ y2 2 )
 

En términos de ellas, el campo se puede escribir en forma más compacta, así: 
 

⎡ a a ⎤ ⎡ a a ⎤
(E1)    E x = − kQ ⎢ f(x, y + ) − f(x, y − ) ⎥ , E y = − kQ ⎢g(x, y + ) − g(x, y − ) ⎥  
⎣ 2 2 ⎦ ⎣ 2 2 ⎦
 

  Supongamos  que  el  punto  P  está  muy  alejado  del  dipolo.  Entonces  serán  muy  pequeños  los 
cocientes 
 
a a a
    << 1, << 1, << 1  
x y r
 

  Para  obtener  los  valores  aproximados  de  Ex  y  Ey  usaremos  la  llamada  aproximación  lineal  del 
cálculo diferencial. A primer orden en ʺhʺ y ʺkʺ tenemos para cualquier función continua y de derivadas 
continuas f(x, y): 
 
∂f ∂f
    f(x + Δx, y + Δy) = f(x, y) + Δx + Δy + ...     |(Δx, Δy)| << |(x, y)| 
∂x ∂y
 

donde Δx y Δy son pequeños incrementos de las variables x y y, respectivamente. 
Aplicando este desarrollo a las expresiones (E1), con Δy = a/2, tenemos para empezar: 
 
a a ∂f a a ∂f
    f(x, y + ) ≈ f(x, y) + ⋅ , f(x, y − ) ≈ f(x, y) − ⋅
2 2 ∂y 2 2 ∂y  
 
de modo que 
 
a a ∂f ∂f
    f(x, y + ) − f(x, y − ) ≈ a ⇒ E x = − kQa  
2 2 ∂y ∂y
 

y análogamente, 
 

a a ∂g ∂g
    g(x, y + ) − g(x, y − ) ≈ a ⇒ E y = −kQa  
2 2 ∂y ∂y
 

Calculemos las derivadas de las funciones f y g con respecto a ʺyʺ: 
 

⎛ ⎞ ⎛ ⎞
∂f ∂ ⎜ x ⎟ −3xy ∂g ∂ ⎜ y ⎟ x 2 − 2y 2
    = ⎜ ⎟= , = ⎜ ⎟=  
∂y ∂y ⎜ 2 3 5
⎟ (x 2 + y 2 ) 2 ∂y ∂y ⎜ 2 3 5
⎟ (x 2 + y 2 ) 2
⎝ (x + y ) ⎝ (x + y )
2 2 2 2
⎠ ⎠
 

  Pero  x 2 + y 2 = r   es  la  distancia  del  punto  P  al  origen  (centro  del  dipolo),  misma  cosa  que  la 
magnitud ʺrʺ del vector r. Entonces, 
1‐27 
 
 
 
 
  Campo eléctrico del dipolo, lejos 
3xy ⎛ − x + 2y
2 2 ⎞ del  mismo.  “p”  es  el  momento  dipolar 
(16)    E x = kp , E y = kp ⎜ ⎟⎟  
r 5 ⎜ r5 del dipolo, definido por  p = Q a. 
⎝ ⎠
 
 

  En la Fig. 24 se esquematiza el campo del dipolo visto desde gran lejanía (compare con el campo 
1
eléctrico producido por dos cargas iguales y opuestas). Note que este campo es de orden  3  porque la ʺxʺ 
r
y la ʺyʺ en (16) son de orden ʺrʺ. 
 
El  dipolo  es  una  configuración  de  carga  que  se 
emplea  para  describir  los  campos  eléctricos  en  la 
materia, tema que abordaremos más adelante. 
  Una distribución de carga acotada es aquella 
que  se  puede  encerrar  dentro  de  una  esfera,  por 
grande  que  sea  su  diámetro  (en  contraposición  a 
una  distribución  infinita,  en  que  la  carga  se 
extiende  hasta  el  infinito).  Toda  distribución  de 
carga acotada parece una carga puntual vista desde 
muy  lejos.  Conforme  nos  acercamos  a  la 
distribución  se  empieza  a  apreciar  la  separación 
entre  las  cargas  positivas  y  negativas,  y  la 
distribución  se  asemeja  a  una  carga  puntual  junto 
con  un  dipolo.  Todavía  más  cerca  podemos  ver  la 
distribución  como  una  carga  puntual,  un  dipolo  y 
  un cuadrupolo. 
Fig. 24 
 

Casos particulares del campo del dipolo. 
  En el caso particular en que el punto campo P se halle sobre el Eje X, donde sus coordenadas son 
(x, 0), las fórmulas (16) nos dan que la componente X del campo es cero, y la componente Y es 
 
kp
(17a)    Ey = −  
x3
 
Si ahora el punto campo está sobre el Eje Y, con coordenadas (0, y), las fórmulas dan: componente X igual 
a cero, y componente Y igual a 
 
2kp
(17b)    Ey = −  
y3
 
Esto se muestra en la Fig. 25 en la siguiente página. 
1‐28 
 
 
 

 
Fig. 25 
 
  En  la  aplicación  de  las  fórmulas  (17a)  y  (17b),  recuerde  que  sobre  los  ejes  tomados  a  lo  largo  del 
dipolo y perpendicularmente al mismo, el campo en P tiene componente solamente a lo largo del eje del 
dipolo (o sea paralelo o antiparalelo al vector  a), y que varía como el inverso del cubo de la distancia al 
centro  del  dipolo.  Para  recordar la  dirección  de estos  campos,  imagínese las líneas de  fuerza  del  campo 
dipolar, dadas en la Fig. 24 de la página anterior. 
1‐29 
 
EJEMPLO 1.19. Tres cargas “2Q”, “–Q” y “–Q” están situadas como se muestra en la Fig. 26. Calcular el 
campo eléctrico en un punto P sobre la línea que une las cargas, a una distancia grande de la carga central. 
  Dado  que  la  distancia  de  P  a  las  cargas  es   
mucho  mayor  que  “a”,  calcularemos  el  campo 
considerando  el  arreglo  de  cargas  dado  como  dos 
dipolos,  tal  como  se  muestra  en  la  Fig.  27.  Los   
 
momentos dipolares de estos dipolos son opuestos. 
Fig. 26 
 
 
  Tomando un Eje Y de coordenadas a   
lo  largo  de  los  ejes  longitudinales  de  los 
dipolos,  con  su  origen  en  la  carga  central, 
tenemos del Ejemplo 1.12 para el campo de 
cada dipolo en el punto P la expresión   
Fig. 27 
 
Campo del dipolo izquierdo: 
 
2kp
    y =
E(1) 3
 
⎛y+ a ⎞
⎜ ⎟
⎝ 2⎠
 
Campo del dipolo derecho: 
 
2kp
    y =−
E(2) 3
 
⎛y − a ⎞
⎜ ⎟
⎝ 2⎠
 
El campo total es 
 
2kp 2kp
    E y = E(1)
y + Ey =
(2)
3
− 3
 
⎛y+ a ⎞ ⎛y− a ⎞
⎜ ⎟ ⎜ ⎟
⎝ 2⎠ ⎝ 2⎠
 
Usando ahora la aproximación lineal del cálculo diferencial, 
 
df
    f(y + ε) − f(y − ε) ≈ 2ε      (ε << r) 
dy
 
con f(y) = 1/y3 y ε = a/2, obtenemos 
 
⎛ a⎛ 3 ⎞⎞ 6kpa
    E y = 2kp ⎜ 2 ⎜ − 4 ⎟⎟ ⎟⎟ = − 4  
⎜ 2⎜ y y
⎝ ⎝ ⎠⎠
1‐30 
 
EJEMPLO  1.20.  Cuatro  cargas  puntuales  están  en  los  vértices  de  un  cuadrado  de  lado  “a”,  como  se 
muestra en la Fig. 28. Calcular el campo eléctrico producido por estas cargas en un punto P muy alejado 
de las mismas. Aproximar la distribución de cargas considerando que forman dos dipolos eléctricos. 
  Las  4  cargas  dadas  las  podemos  considerar   
como  dos  dipolos  de  dos  distintas  maneras,  ya  sea 
como dipolos verticales o como dipolos horizontales. 
  Recordando  las  fórmulas  (17a)  y  (17b)  de  la 
página  1‐22,  resultará  más  conveniente  considerar 
dos dipolos horizontales, formados  respectivamente 
por  las  cargas  en  los  vértices  superiores  por  una 
parte, y en los vértices inferiores por la otra. 
  Las distancias del punto campo P a los centros 
de estos dipolos serán entonces 
 
a a
    x− y x+  
2 2
 
al dipolo superior y al inferior, respectivamente.   
 
  Los  campos  eléctricos  producidos  por  los 
Fig. 28 
dipolos superior e inferior en el punto P apuntan en 
 
las direcciones  X y –X, respectivamente, y valen 
 
kp kp
    y =
E(1) 3
    y    y =−
E(2) 3
 
⎛x − a ⎞ ⎛x + a ⎞
⎜ ⎟ ⎜ ⎟
⎝ 2⎠ ⎝ 2⎠
 

de modo que el campo total de las 4 cargas, muy lejos de ellas, es 
 

⎛ ⎞
⎜ ⎟
    E y = E(1)
y + Ey
(2)
= − kp ⎜
1

1 ⎟ = −kp ⎜⎛ f(x + a ) − f(x − a ) ⎟⎞  
⎜⎛ a⎞
3
⎛x − a ⎞
3 ⎟ ⎝ 2 2 ⎠
⎜ ⎜x + ⎟ ⎜ ⎟ ⎟
⎝⎝ 2⎠ ⎝ 2⎠ ⎠
 

donde  hemos  definido  f(x)  =  1/x3.  Usando  la  aproximación  lineal  del  cálculo  diferencial  e  integral,  a 
saber, 
 
df
    f(x + ε) − f(x − ε) ≈ 2ε  
dx
 

obtenemos 
 

⎛ a⎛ 3 ⎞ ⎞ 3kpa
    E y = − kp ⎜ 2 ⎜ − 4 ⎟⎟ = 4  
⎝ 2⎝ x ⎠⎠ x
 
1‐31 
 
 
EJEMPLO 21. Un campo eléctrico posee líneas de fuerza rectas. La magnitud del campo depende de una 
variable de posición medida a lo largo de la línea de fuerza. Obtener una expresión para la fuerza eléctrica 
sobre un dipolo orientado a lo largo de una línea de fuerza. 
 

 
Fig. 30 
 
  Observe la Fig. 30: estamos suponiendo que la línea de fuerza del campo va hacia la derecha, y que 
“x” es la variable de posición medida a lo largo de la línea de fuerza. 
  La fuerza F sobre el dipolo es la suma de las fuerzas sobre las cargas –q y q, o sea (Fig. 31): 
 
    F = – q E(x) + q E(x + a) 
 

 
 
Fig. 31 
 
  Usando la aproximación lineal del cálculo diferencial, 
 
dE
    E(x + Δx) − E(x) = Δx  
dx
 
con Δx = a, tenemos 
 
dE dE
    F = q ( E(x + a) − E(x) ) = qa =p  
dx dx
 
donde p = q a es el momento dipolar del dipolo. 
  Ejemplos: 
  Dipolo en el campo inhomogéneo de una carga puntual (K constante): 
 

K ⎛ −2K ⎞
    E(x) = , F = p⎜ 3 ⎟
x2 ⎝ x ⎠
 
  Dipolo en un campo lineal (K constante): 

 
E(x) = Kx, F = pK  
   
1‐32 
 
 
1.7. TORCA SOBRE UN DIPOLO ELÉCTRICO EN EL SENO DE UN CAMPO ELÉCTRICO UNIFORME. 
  Un dipolo situado dentro de un campo eléctrico uniforme no experimenta fuerza neta alguna, pero 
sí una torca. 
  Veamos la Fig. 32. La fuerza eléctrica debida al campo uniforme E sobre el dipolo es la suma de las 
fuerzas sobre las cargas “–q” y “q” que constituyen el dipolo. Tales fuerzas son iguales y opuestas, de tal 
manera que la fuerza total es cero. Existe un par de fuerzas {F, –F} actuando sobre el dipolo. 
 

 
Fig. 32 
 
  Por  otra  parte,  el  momento  o  torca  producido  por  el  par  {F,  –F}  es  independiente  del  punto  de 
referencia de momentos, y es igual al producto de la fuerza  F y la distancia entre las líneas de acción de 
las fuerzas del par, la cual es igual a “a sen θ”. La torca es entonces igual a 
 
    τ = F a sen θ 
 
  En la Fig. 32, la dirección de esta torca es perpendicular al papel, hacia dentro. 
  Existe la relación vectorial 
 
    τ = a × F 
 
Finalmente, poniendo  
 
    F = q E    y    p = q a  (Momento dipolar) 
 
obtenemos 
 
 
Momento o torca sobre un dipolo en el seno 
(14)    τ = p × E 
de un campo eléctrico uniforme. 
 
 
  Note que si el vector  a apunta hacia el semiplano inferior en la Fig. 32 (o sea que la carga “–q” se 
halla a mayor altura que la carga “q”), entonces la torca cambia de sentido, de tal manera que el dipolo 
tiende a alinearse con el campo eléctrico (El vector a y el campo E tienden a ser paralelos). 
 
1‐33 
 
 
1.8. PROBLEMAS. 
1. Se sabe que cada átomo de aluminio aporta 3 electrones libres. La masa molar del aluminio (Z = 13) es 
26.982  g/mol.  ¿Qué  masa de  aluminio es  necesaria para que  el  total  de  electrones  libres aportado  por  la 
misma sea de 15 (1030)? 
Resp. 
 
2. ¿Cuántos átomos contiene una muestra de plata pura cuyo volumen es de 1 cm3?. La densidad de masa 
de la plata es  ρm = 10.5 g/cm3 , y su masa molar es de 107.9 g/mol. 
Resp. 5.86 (1019) 
 
3. Se deposita un exceso de electrones sobre una moneda pequeña de oro cuya masa es de  15 g, de modo 
que su carga neta es –6.2 nC. (a) Calcular el número de electrones en exceso de la moneda; (b) ¿Cuántos 
electrones en exceso hay en cada átomo de oro? El número atómico del oro es 79, y su masa molar es de 
197 g/mol. 
Resp.  (a) 3.875(1010 ) (b) 8.45(10 −11 )  
 
4. Dos esferas pequeñas separadas por una distancia de 0.8 m poseen cargas iguales. ¿Cuántos electrones 
en exceso hay en cada esfera si se repelen con una fuerza de magnitud 3.6 (10–20) N? 
Resp. 1000 000. 
 
5. El número atómico del plomo es Z = 82 y su masa molar es 207 g/mol. ¿Cuántos protones contiene un 
pedazo de plomo de 10 g de masa? Si a dos pedazos tales se les colocara a una distancia mutua de 1 m y se 
les quitara el 0.01 % de su carga eléctrica negativa total, ¿Con qué fuerza se repelerían? 
Resp. 
 
6.  La  figura  muestra  las  líneas  del  campo  eléctrico  de  un  sistema  de  5  cargas  puntuales.  Se  sabe  que  la 
carga superior izquierda es positiva. ¿Cuál es el signo de cada una de las restantes cargas? 
 

 
Fig. Probl. 1 
 
1‐34 
 
 
 
7. Se tienen dos cargas puntuales  q1 y  q2 con   
los valores q1 = – 8 μC y q2 = – 2 μC. 
Están situadas como se muestra en la figura. 
El lado de la cuadrícula mide 1 metro. 
Calcular la fuerza eléctrica que ejerce la carga 
q1 sobre la carga q2. 
Resp. 

 
Fig. Probl. 3 
 
8. Refiriéndonos a la figura del problema 7, calcular el campo eléctrico que ejerce la carga  q1 en el punto 
donde está situada la carga  q2. Calcular luego el campo debido a  q2 en el punto donde está  q1. Dado este 
último campo, calcular la fuerza eléctrica que ejerce q2 sobre q1. 
 
9.  Calcular  el  campo  eléctrico   
producido  por  las  tres  cargas  q1,  q2  y 
q3  en  el  punto  P.  El  lado  de  la 
cuadrícula  mide  1  m.  Suponga  los 
siguientes valores: 
 
 
 
q1 = –4 μC, q2 = 7 μC, q3 = – 5 μC. 
Fig. Probl. 3 
Resp. 
 
 
10. Refiriéndonos a la figura del problema 9, supongamos que en el punto  P estuviese otra carga puntual 
de valor q = 1.5 μC. ¿Cuál sería la fuerza que sufriría esta carga en el campo de las otras 3? 
Resp. 
 
11.  Tres  cargas  puntuales  Q,  –(9/16)Q  y  9Q  se  encuentran  sobre  una  línea  recta,  con  las  distancias 
indicadas en la figura. Calcular la fuerza eléctrica ejercida sobre cada carga por las otras dos. 
 

 
Resp. 0; 0; 0. 
 
1‐35 
 
 
 
12.  Cuatro  cargas  puntuales  están  situadas  en  los   
vértices de un cuadrado de lado “a”. Los signos de 
las cargas se alternan, como se muestra en la figura. 
Calcular  la  magnitud  de  la  fuerza  que  existe  sobre 
cada carga debida a las otras tres. ¿En qué dirección 
está cada fuerza? 
Resp. 
q2 1
F= 2
⋅ ( 2 − ) ; Diagonalmente, hacia dentro. 
a 2
 
13. Dos pequeñas bolas de espuma de estireno de 0.5 g están ensartadas como cuentas en un hilo aislante 
vertical.  La  bola  inferior  está  pegada  al  hilo,  y  la  bola  superior  está  libre  de  moverse.  Si  a  ambas  se  les 
proporciona una cantidad igual de carga negativa de modo que la bola superior queda suspendida sobre 
la inferior a una distancia de 5 cm de ésta. ¿Cuál es el exceso de electrones que posee cada bola? 
Resp.  2.30 (1011). 
 
14. Dos objetos constan cada uno de un par de cargas puntuales iguales y de distinto signo, unidas por 
varillas  aislantes  ligeras  y  rígidas.  Los  objetos  se  colocan  sobre  una  recta  como  se  muestra  en  la  figura. 
Determine si los objetos se atraen, se repelen, o no se afectan mutuamente. 
 

 
Resp. Se atraen. 
 
 
15. Cuatro cargas están dispuestas como se muestra   
en  la  figura.  Calcular  el  campo  eléctrico  en  un 
punto  P  situado  a  una  distancia  muy  grande  “x” 
del origen de coordenadas. Considerar este arreglo 
de cargas como si fuesen dos dipolos. 
 
Resp. 

 
   
 
 
2‐1 

  CAPÍTULO 2 
 
 
CAMPO ELÉCTRICO DE DISTRIBUCIONES 
  CONTINUAS DE CARGA 
 
 
2.1. DENSIDAD DE CARGA ELÉCTRICA. 
  Existen tres clases de distribuciones continuas de carga, denominadas distribución lineal, superficial 
y volumétrica. 
  Si la carga se distribuye continuamente a lo largo de un cuerpo con forma de filamento (alambre, 
hilo, haz, etc.), tal que podamos representarlo apropiadamente por una curva matemática, la distribución 
es lineal. Si la carga de la distribución reside sobre una superficie (hoja, sábana), como por ejemplo una 
lámina  delgada,  la  frontera  o  superficie  exterior  de  un  cuerpo  sólido,  etc.,  entonces  se  denomina 
distribución  superficial.  Finalmente,  la  distribución  es  volumétrica  si  la  carga  ocupa  una  región 
tridimensional del espacio, cuyo volumen debe tenerse en consideración. 
  El modo como la carga eléctrica está repartida por la región espacial ocupada por la distribución de 
carga se describe mediante una función de la posición llamada densidad de carga. Explicaremos con detalle 
este concepto en relación con la distribución más simple, la lineal. Para las otras dos distribuciones la idea 
es completamente análoga. 
kkk 
2.2. DENSIDAD LINEAL DE CARGA ELÉCTRICA. 
  Consideremos un filamento cargado, de longitud L y carga eléctrica total Q. 
  Cuando segmentos de igual longitud del filamento contienen la misma cantidad de carga eléctrica, 
independientemente  de  su  ubicación  a  lo  largo  del  filamento,  la  densidad  de  carga  es  constante  (o 
uniforme).  En  este  caso  la  densidad  lineal  de  carga,  denotada  con  “λ”,  se  define  simplemente  como  el 
cociente de la carga Q y la longitud L del filamento, o sea 
 
 
 
Q Densidad  lineal  de  carga  (constante)  de  un  filamento  cargado 
(1)    λ=   uniformemente. Q es la carga y L la longitud del filamento. 
L
 
 
coulombio C
Las unidades físicas de esta cantidad son  = . 
metro m
 

  Por ejemplo, un trozo de alambre de longitud L = 0.5 m, cargado uniformemente con carga total 
8 μC μC
Q = 8 μC, posee una densidad lineal de carga igual a   λ = = 16 . 
0.5m m
 

  Una densidad lineal de carga variable (o no uniforme) se caracteriza por que trozos de igual longitud 
del filamento no contienen en general la misma carga eléctrica, la cual depende de la ubicación del trozo 
dentro  del  filamento.  En  este  caso  la  densidad  lineal  de  carga  λ  varía  de  punto  a  punto  a  lo  largo  del 
filamento. 
  La carga eléctrica contenida en un punto del filamento es obviamente cero, pues un punto no posee 
dimensiones geométricas. Es necesario considerar un segmento del filamento para poder hablar de carga 
contenida. 
2‐2 

 
  Para  tratar  este  caso,  consideremos  en  el  espacio  tridimensional  una  curva  que  represente  un 
cuerpo filamentario (alambre) cargado eléctricamente. Los puntos de esta curva serán puntos fuente, pues 
existe carga eléctrica distribuída a lo largo de la curva. 
  Finquemos un sistema de coordenadas cartesianas XYZ. Sea P’(x’, y’, z’) un punto fuente arbitrario 
de la curva, con vector de posición  r’. Sea  dr’ el desplazamiento infinitesimal a lo largo de la curva en el 
punto P’ (Fig. 1). 
  Sea  ds’ la longitud del desplazamiento  dr’ (es decir,  |dr’| = ds’), y sea  dQ la carga contenida en 
este pequeño tramo ds’. Esta pequeña carga se denomina elemento de carga. 
 

 
Fig. 1 
 
  Se define la densidad lineal de carga en el punto  P’, denotada con  λ(P’) o con  λ(r’), como la carga 
eléctrica por unidad de longitud residente en el pequeño segmento ds’ localizado en P’, o sea 
 
   
 
dQ
(2)    λ≡   Densidad lineal de carga (variable) 
ds′
 
 
  La longitud ds’ es tan pequeña que el valor de λ se puede asociar con el punto P’. 
  La relación (2) se usa frecuentemente en la forma 
 
   
(3)    dQ = λ ds’  Elemento de carga de una distribución lineal continua. 
 
 
  Expresado  en  la  base  vectorial  ortonormal  cartesiana,  el  vector  de  posición  de  los  puntos  de  la 
curva‐filamento se escribe 
 

    r’ = (x’, y’, z’) = x’ i + y’ j + z’ k 
 

así que la notación λ(r’) significa lo mismo que λ(x’, y’, z’). 
2‐3 

 
EJEMPLO 2.1. Un alambre circular de radio  a = 12 cm, al que le falta un trozo con forma de cuadrante, 
posee una carga eléctrica total uniforme de Q = 18 μC. Calcular su densidad lineal de carga. 
  Dado  que  la  carga  del  anillo  está  distribuída   
uniformemente,  podemos  calcularla  aplicando  la  relación 
(1), λ = Q/L. 
  La longitud del alambre es 
 
1 3π
    L = 2 πa − ⋅ 2 πa = a 
4 2
 
Entonces la densidad lineal es   
  Fig. 2 
 
−6
Q 2Q 2 ⋅ 18(10 )C C
    λ= = = −2
= 3.2(10 −5 )  
πa 3πa 3 ⋅ 3.14 ⋅ 12(10 )m
3 m
2
 
EJEMPLO 2.2. Un trozo de alambre recto de longitud L posee una carga eléctrica total Q no uniforme que 
varía con la posición de acuerdo con la relación  λ = C x2, donde C es una constante con unidades “C/m3” 
y “x” está en metros. Calcular la carga eléctrica total del trozo. El alambre está situado a lo largo del Eje X, 
con su extremo izquierdo en el origen O (Fig. 3). 
  Partimos de la relación (3), o sea 
 

    dQ = λ ds’ 
 

En el caso presente tenemos 
 

    ds’ = dx   
  Fig. 3 
(Recordemos  que  ds’  es un  segmento infinitesimal del  trozo  de alambre. En este  ejemplo simple  hemos 
denotado sin primas las coordenadas de los puntos fuente. No debe haber lugar a confusión) 
Así pues, 
 
    dQ = C x2 dx 
 
Integrando desde x = 0 hasta x = L obtenemos la carga total del alambre: 
 
L
∫ ∫ 0 Cx
    Q = dQ = 2 dx  

 
Se obtiene 
 
1 3
    Q= CL  
3
2‐4 

 
2.3. DENSIDADES SUPERFICIAL Y VOLUMÉTRICA DE CARGA ELÉCTRICA. 
  Para  las  densidades  de  carga  superficial  y  volumétrica  existen  denominaciones  y  definiciones 
análogas  a  las  de  la  densidad  lineal.  Estudie  las  Figs.  4  y  5.  En  ellas,  P’  es  un  punto  cualquiera  de  la 
distribución (es decir, un punto fuente arbitrario). En el punto fuente se localiza el elemento de área o de 
volumen da’ o de dV’. 
 
   

Fig. 4  Fig. 5 
 
  La  densidad  superficial  de  carga,  evaluada  en  el  punto  fuente  P’,  denotada  con  σ,  es  la  carga 
eléctrica  por  unidad  de  área,  alojada  en  un  pequeño  elemento  de  área  da’  ubicado  en  P’.  Las  unidades 
físicas de σ son “coulombio/(metro)2” ó “C/m2”. 
  La densidad volumétrica de carga se denota con  ρ y se define análogamente. Existen las relaciones, 
análogas a (3), 
 
 
Elemento de carga de una distribución superficial continua. 
(4)    dQ = σ da’ 
da’ es el área del elemento de carga. Unidades de σ: C/m2. 
   
(5)    dQ = ρ dV’  Elemento de carga de una distribución volumétrica continua. 
  dV’ es el volumen del elemento de carga. Unidades de ρ: C/m3. 
 
  Si  σ es constante, su valor es el cociente de la carga total  Q de la superficie y el área total  A de la 
misma, o sea 
 
Q coulombio C
(6)    σ=     (Unidades:    2
= 2 ) 
A (metro) m
 
  Análogamente, si ρ es constante, se calcula de 
 
Q coulombio C
(7)    ρ=     (Unidades:    3
= 3 ) 
V (metro) m
 
2‐5 

donde V es el volumen de la distribución de carga volumétrica. 
EJEMPLO 2.3  Una esfera de radio R = 120 mm posee una carga eléctrica positiva y uniforme igual a 
Q = 4.4  μC. Calcular la densidad volumétrica de carga de la esfera. 
  Dado que la carga eléctrica está distribuída uniformemente, la densidad volumétrica de carga  ρ se 
calcula simplemente como el cociente de la carga total Q y el volumen V = (4/3) π R3 de la esfera: 
 
Q Q 3Q 3 ⋅ 4.4 μC μC
    ρ= = = = = 6.1(10 −4 )  
V 4 πR 3 4 πR 3 4 ⋅ 3.14 ⋅ (0.12m)3 m3
3
 
 
EJEMPLO 2.4.  Una lámina delgada de largo “a” y ancho “b” posee carga eléctrica no uniforme, descrita 
por  la  densidad  de  carga    σ  =  C  x  y2.  La  lámina  está  situada  como  se  muestra  en  la  Fig.  6.  Calcular  la 
constante C sabiendo que la carga total de la lámina es Q. 
 

 
Fig. 6 
 
  El elemento de carga  dQ tiene la forma de un pequeño rectángulo de dimensiones  dx y  dy, y área 
da’ = dx dy. Usando la Ec. (4) tenemos 
 
    dQ = σ da’ = C x y2 dx dy 
 
Sabemos que la integral de dQ es la carga total Q de la lámina. Para efectuar la integral doble sobre las 
variables x y y hacemos uso del siguiente teorema: 
 
(8)  ∫
Dada una integral doble  f(u,v)dudv , si el integrando f(u,v) se puede poner como el producto de 

dos funciones, cada una de una variable, o sea  f(u, v) = g(u) h(v), entonces la integral doble es el producto 


de dos integrales simples, en la forma  f(u,v)dudv = ( ∫ g(u)du )( ∫ h(v)dv ) . 
 

Obtenemos entonces 
 

⎛ a ⎞ ⎛ b 2 ⎞ a 2 b3
  ∫ ∫
Q = dQ = Cxy 2dxdy = C ⎜ xdx ⎟ ⋅ ⎜
⎝ 0 ⎠ ⎝ 0

y dy ⎟ = C



2 3
 

 
6Q 6Q
de donde C = . Entonces la densidad de carga es   σ = Cxy 2 = xy 2  
a 2 b3 a 2 b3
2‐6 

 
EJEMPLO 2.5.  Una esfera de radio  R posee una densidad de carga volumétrica que depende solamente 
de la distancia “r” al centro de la esfera, según la ley 
 
    ρ = k r 
 
donde k es una constante con dimensiones C/m4. Calcular la carga total de la esfera. 
  Integrando  la  Ec.  (5‐p4)  tenemos  que  la  carga  total  viene  dada  por  la  integral  (extendida  sobre  el 
volumen de la esfera) 
 
    ∫
Q = ρ(r)dV  
 
Hacemos uso ahora del siguiente teorema 
 
(9)   Al  hacer  la  integral  de  una  función  de  “r”  sobre  una  región  esférica,  el  elemento  de  volumen  se 
puede tomar en la forma  “dV = 4πr2 dr”, que corresponde al volumen de un cascarón esférico de radio 
“r” y grosor “dr”. 
 
Entonces, 
 
R R
∫ 0 ρ ⋅ 4πr ∫ 0 k r ⋅ 4πr
    Q= 2 dr = 2 dr = π k R 4  

 
EJEMPLO  2.6.    Dos  cilindros  sólidos  tienen  la  misma  altura  “h”  pero  diferentes  radios,  R1  y  R2. 
Suponiendo  que  sus  densidades  de  carga  son  iguales  y  uniformes,  ¿Cómo  se  comparan  sus  cargas 
eléctricas? 
  El volumen de un cilindro de altura “h” y radio R es 
 
    V = πR 2 h  
 
así que las densidades de carga de los cilindros son 
 
Q1 Q2
    ρ1 = y ρ2 =  
πR12 h πR 22 h
 
Igualando estas densidades se sigue que la carga es proporcional al cuadrado del radio: 
 
2
Q1 R1
    =  
Q2 R 2
2
2‐7 

 
EJEMPLO 2.7.   Un cascarón cilíndrico de radio  R y altura “h” está cargado uniformemente con densidad 
superficial de carga “σ” (Fig. 7). Supongamos que deseamos describir la carga del cascarón mediante una 
densidad  lineal de carga “λ”, que sea la carga por unidad de longitud del cascarón medida a lo largo del 
eje longitudinal del cilindro. ¿Qué relación existiría entre “σ” y “λ”? 
 
  Calculemos  primeramente  la  carga  total  del   
cascarón.  El  área  de  la  superficie  lateral  del 
cascarón es “2πrh”, de modo que 
 
    Q = σ ⋅ 2πrh  
 
(Las tapas del cilindro no poseen carga). 
  Ahora  obtengamos  la  densidad  lineal  de 
carga  dividiendo  la  carga  total  entre  la  altura  del 
cascarón: 
   
Q σ ⋅ 2 πrh
    λ= =   Fig. 7 
h h
 
Entonces la relación buscada es 
 
    λ = 2πr σ 
 
EJEMPLO 2.8.  Una lámina circular plana y delgada, de radio R, posee una densidad superficial de carga 
eléctrica que varía con la distancia al centro de la lámina según la ley  σ = C r, donde C es una constante. 
Dividamos  la  lámina  en  anillos  diferenciales  de  grosor  “dr”.  (a)  Obtener  la  carga  “dQ”  de  un  anillo 
arbitrario de radio “r”. (b) ¿Cuál es la carga total de la lámina? 
(a)  El área del anillo diferencial es   
 
    da = 2 πrdr  
 
(como si fuese un rectángulo de base 2πr y altura dr) 
Entonces la carga del anillo es 
 
    dQ = σ da = Cr ⋅ 2 πrdr = 2 πCr 2 dr  
 
(b)  La carga total de la lámina es   
Fig. 8 
R 2πCR 3
    ∫
Q = dQ = ∫0 2 πCr 2 dr =
3
 

 
2‐8 

 
2.4. PROCEDIMIENTO GENERAL PARA CALCULAR EL CAMPO ELÉCTRICO PRODUCIDO 
  POR UNA DISTRIBUCIÓN CONTINUA DE CARGA. 

  Consideremos  una  distribución  de  carga  dada,  de  cualquier  tipo,  y  el  problema  de  calcular  el 
campo eléctrico de la misma en algún punto P del espacio (Véase la Fig. 9). 
  Este problema se resuelve basándose en tres ideas principales: 
  (i) Se subdivide la distribución de carga dada en un numeroso conjunto de pequeños elementos de 
carga, que porta cada uno cierta carga diferencial dQ, tan pequeña que se puede considerar como si fuese 
una carga puntual. 
  (ii) Se usa la expresión del campo eléctrico aplicable a una carga puntual, con objeto de calcular el 
campo eléctrico diferencial dE producido por el elemento general de carga dQ en el punto P. 
  (iii)  Se  suman  vectorialmente  (es  decir,  se  integran)  los  campos  eléctricos  diferenciales  dE 
producidos por todos los elementos de carga dQ en que se subdivide la distribución dada. 
 

 
Fig. 9 
 

  Expresaremos  estas  ideas  matemáticamente.  Previamente  recordemos  la  siguiente  notación  y 


nomenclatura: 
  El  punto  campo  P  es  aquel  punto  del  espacio  donde  deseamos  calcular  el  campo  eléctrico.  Para 
efectos  de  este  cálculo,  P  es  un  punto  fijo  del  espacio.  Una  vez  hecho  el  cálculo,  las  coordenadas  de  P 
podrán  considerarse  variables.  Designaremos  con  “P(x,  y,  z)”  las  coordenadas  del  punto  campo,  y  su 
vector de posición con 
 

    r = (x, y, z) = x i + y j + z k 
 

  El  punto fuente  P’ es un punto  arbitrario de la distribución de carga dada. En este punto se localiza 


el  elemento  de  carga  dQ,  el  cual  puede  ser  un  pequeño  segmento,  área  o  volumen,  según  que  la 
distribución  de  carga  sea  lineal,  superficial  o  volumétrica,  respectivamente.  Las  coordenadas  de  P’  se 
designarán con primas en la forma “P’(x’, y’, z’)”, y su vector de posición se denotará con 
 

    r’ = (x’, y’, z’) = x’ i + y’ j + z’ k 
 
2‐9 

 
Las coordenadas de P’ son variables, que varían por toda la región espacial ocupada por la carga. 
  El vector separación (o desplazamiento) que une P’ con P se denota con “R”. Su magnitud “R” es la 
distancia entre los puntos fuente y campo. En la Fig. 9 se advierte la relación 
 
    R = r – r’ 
 
  Pasemos ahora a obtener la expresión matemática general del campo E en P. 
  Dado  que  el  elemento  de  carga  dQ  se  considera  una  carga  puntual,  su  campo  eléctrico  dE 
producido en el punto P tiene una magnitud dE dada por 
 
1 dQ
    dE =  
4 πε0 R 2
 
La dirección de este campo es a lo largo de la línea  P’P, es decir, la misma que la del vector  R, como se 
muestra en la Fig. 9. 
  Ahora bien, sean  α1,  α2 y  α3 los ángulos que forma el campo diferencial  dE con los Ejes X, Y y Z, 
respectivamente. Entonces las componentes de este campo son 
 
1 dQ 1 dQ 1 dQ
    dE x = cos α1 dE y = cos α 2 dE z = cos α 3  
4 πε0 R 2 4 πε0 R 2 4 πε0 R 2
 
Finalmente, para obtener el campo producido por toda la distribución integramos cada componente, de 
tal manera que 
 
1 dQ 1 dQ 1 dQ
(10)    Ex = ∫
4 πε0 R 2
cos α1 Ey = ∫
4 πε0 R 2
cos α 2 Ez = ∫
4 πε0 R 2
cos α 3  

 
Las integrales se extienden sobre toda la región ocupada por la carga. Estas son las expresiones buscadas. 
  En  este  curso  nos  limitaremos  a  los  campos  eléctricos  (y  magnéticos)  producidos  por  cuerpos 
regulares  como  alambres  rectos,  anillos  circulares,  esferas,  cilindros,  planos,  etc.  En  la  mayoría  de  los 
casos que trataremos aprovecharemos las simetrías de la distribución de carga dada, así como la ubicación 
del punto campo, para reducir el problema al cálculo de solamente una o dos de las integrales (10). 
  En las secciones 5, 6 y 7 consideramos sendos ejemplos de aplicación de las fórmulas (10). 
2‐10 

 
EJEMPLO 2.9.  Un anillo circular de radios interior 
R1  y  exterior  R2  y  grosor  muy  delgado,  de  tal 
manera  que  se  puede  considerar  plano,  posee  una 
carga  eléctrica  uniformemente  distribuída. 
Demostrar  que  el  campo  eléctrico  del  anillo  en  los 
puntos sobre su eje de simetría perpendicular a su 
plano apunta a lo largo de dicho eje. 
 
  Coloquemos  el  anillo  con  su  centro  en  el 
 
origen  de  un  sistema  cartesiano  XYZ,  como  se  Fig. 10 
muestra  en  la  Fig.  10.  Se  desea  demostrar  que  el 
campo eléctrico del anillo en los puntos del Eje Z  
está precisamente en la dirección de este eje. 
  Esto  lo  podemos  ver  superponiendo  vectorialmente  los  campos  eléctricos  diferenciales  “dE” 
producidos por los elementos de carga del anillo. Veamos. 
 

 
Fig. 11 
 
  Consideremos  dos  elementos  de  carga  (dQ)1  y  (dQ)2  dispuestos  simétricamente  con  respecto  al 
origen,  como  vemos  en  la  Fig.  11.  Las  áreas  de  estos  elementos  son  iguales,  lo  mismo  que  sus  cargas 
diferenciales: (dQ)1 = (dQ)2. 
  Los campitos eléctricos  dE1 y  dE2 que producen estos elementos en un punto cualquiera del Eje Z 
tienen  la  misma  magnitud,  y  sus  direcciones  son  tales  que  las  componentes  de  estos  campitos  sobre  el 
plano  XY  son  iguales  y  de  signos  contrarios.  Estas  últimas  componentes  se  cancelan.  Por  otra  parte,  las 
componentes a lo largo del Eje Z se suman. 
  Todo el anillo se puede dividir en tales parejas de elementos de carga, por lo que la suma vectorial 
de todos estos campos diferenciales, es decir, el campo total del anillo, solamente tendrá componente Z en 
los puntos del Eje Z, Q.E.D. 
  El razonamiento que acabamos de emplear se denomina “de simetría”. En otros casos de simetría el 
campo total puede calcularse evaluando solamente una de las integrales (10), como veremos. 
2‐11 

 
2.5. CAMPO ELÉCTRICO GENERADO POR UN TROZO DE ALAMBRE RECTO DE LONGITUD L Y CARGA 
  ELÉCTRICA TOTAL Q DISTRIBUÍDA UNIFORMEMENTE. 

  He aquí el procedimiento para calcular este campo, paso a paso: 
1. Hacer una figura clara de la situación y escoger un sistema de Ejes XYZ conveniente. 
  Notemos que el campo del trozo de alambre tiene simetría rotacional alrededor del eje coincidente 
con el mismo, de tal manera que no se pierde generalidad si colocamos el trozo como se muestra en la 
Fig. 12, con el alambre a lo largo del Eje Y, y su centro en el origen del sistema XY. 
   
2. Marcar el punto campo P(x, y, z) y el punto   
fuente general P’(x’, y’, z’). 
  Deseamos  calcular  el  campo  E  en  un 
punto  arbitrario  P(x,  y)  del  plano  XY.  Este  es 
entonces el punto campo. 
  El  punto  fuente  general  es  un  punto 
cualquiera  del  trozo  de  alambre.  Es  un  punto 
variable  cuyas  coordenadas  son  P’(0,  y’).  La 
coordenada y’ varía entre –L/2 y +L/2. 
  En  el  punto  fuente  P’  se  localiza  el 
elemento  de carga  dQ. En  la Fig.  12  no  hemos 
trazado la forma  de  este  elemento,  lo  haremos 
un poco más adelante. 
 
  Tracemos  el  segmento  P’P,  denotando   
su longitud con “R”.  Fig. 12 
 
3. Obtener la magnitud del campo dE producido por el elemento de carga en el punto campo P. 
  En el punto fuente  P’ imaginemos colocada una pequeña carga  dQ. El campo  dE que produce esta 
carga en P se obtiene con la fórmula del campo de una carga puntual, es decir, 
 
1 dQ
    dE = ⋅     (Esta es la magnitud del campo dE producido por dQ) 
4 πε0 R 2
 
Como vemos en la Fig. 12, dE está dirigido a lo largo de la línea P’P. De la misma figura obtenemos 
 
    R = x 2 + (y − y′)2  
 
de tal manera que 
 
1 dQ 1 dQ
(11)    dE = ⋅ 2 = ⋅ 2  
4 πε0 R 4 πε0 (x + (y − y′)2 )
 
  En  la  siguiente  etapa  analizamos  con  más  detalle  la  forma  geométrica  del  elemento  de  carga  dQ, 
que se supone localizado en P’. 
2‐12 

4. Trazar por separado el detalle del elemento de carga  dQ. Expresar su dimensión geométrica “  ds’  ” 


en términos de las diferenciales de las coordenadas del punto fuente. 
 
  El  detalle  del  elemento  dQ,  ubicado  en  el   
punto (0, y’), se muestra en la Fig. 13.   
  El  elemento  de  carga  es  un  trocito  recto  de 
alambre  de  longitud  dy’,  que  porta  carga  dQ. 
Tenemos  así  que  la  relación    dQ  =  λ  ds’  se 
convierte en 
 
(12)    dQ = λ dy’ 
 
  Puesto  que  la  carga  está  distribuída 
uniformemente,  la  densidad  de  carga  lineal  del 
alambre es simplemente 
 
Q
    λ=    
L
Fig. 13 
 
de modo que 
 
Q
(13)       dQ = λdy′ = dy′  
L
 
5. Sustituir dQ en la expresión para dE, y obtener las componentes dEx y dEy del campo. 
  Poniendo (13) en (11), 
 
1 λdy′ Q dy′
(14)    dE = ⋅ 2 = ⋅ 2  
4πε0 (x + (y − y′) ) 4 πε0 L (x + (y − y′)2 )
2

 
  El ángulo que forma dE con el Eje X es 
“α”, el mismo que vemos en la Fig. 14 junto 
al punto P’. Se tiene 
 
x x
cos α = =
R x 2 + (y − y′)2
(15)         
y − y′ y − y′
sen α = =
R x 2 + (y − y′)2
 
Las  componentes  del  campo  eléctrico 
 
diferencial dE son entonces  Fig. 14 
 
2‐13 

Q xdy′
    dE x = dE ⋅ cos α = ⋅ 2  
4 πε0 L [x + (y − y′)2 ]3 / 2
 
Q (y − y′)dy′
    dE y = dE ⋅ sen α = ⋅ 2  
4 πε0 L [x + (y − y′)2 ]3 / 2
 
  Advierta que las coordenadas (x, y) del punto campo P son constantes. La única variable aquí es y’.  
Se tiene 
 
Qx L/2 dy′
(16)       E x = dE x = ∫ 4 πε0 L ∫−L / 2 [x2 + (y − y′)2 ]3 / 2  
 
Q L/2 (y − y′)dy′
    E y = dE y = ∫ 4 πε0 L ∫−L / 2 [x 2 + (y − y′)2 ]3 / 2
 

 
6. Evaluar las integrales de  dEx  y  dEy. Los límites de integración deben cubrir toda la distribución de 
carga. 
  Para calcular las integrales, hacemos el cambio de variable 
 

    u = y − y’    du = −dy’ 
 

y Ex y Ey se vuelven 
 
Qx L/2 du Q L/2 udu
    Ex = −
4 πε0 L ∫−L / 2 ⎡ 2 2 ⎤3 / 2
    Ey = −
4 πε0 L ∫−L / 2 [x2 + u2 ]3 / 2  
⎣x + u ⎦
 

Consultando las tablas de integrales encontramos 
 
du 1 u udu 1
(17)    ∫ = ⋅ +C ∫ =− +C 
(a ) (a )
2 3/2 2 2 3/2
2
+u a a + u2
2 2
+u a + u2
2

lo que nos da para Ex y Ey finalmente las expresiones: 
 
⎛ ⎞
⎜ L L ⎟
L
y− y+
1 ⎢⎡ ⎤ 2
Qx y − y′ Q ⎜ ⎟
(18a)         Ex = − ⋅ ⎥ =− 2 − 2
4 πε0 L x 2 ⎢ x 2 + (y − y′)2 ⎥ 4 πε0 Lx ⎜ 2 2 ⎟ 
⎣ ⎦ − L2 ⎜ x2 + ⎛ y − L ⎞ 2 ⎛
+ +
L⎞ ⎟
⎜ ⎜ ⎟ x ⎜ y ⎟ ⎟
⎝ ⎝ 2⎠ ⎝ 2⎠ ⎠
⎛ ⎞
⎜ ⎟
L

Q ⎡ ⎤ 2

⎢ 1 ⎥ Q ⎜ 1 1 ⎟
(18b)        Ey = = ⎜ −  
4 πε0 L ⎢ x 2 + (y − y′)2 ⎥ 4 πε0 L 2 2 ⎟
⎣ ⎦ − L2 ⎜ x + y−
2 ⎛ L ⎞ 2 ⎛ L ⎞
x + ⎜ y + ⎟ ⎟⎟
⎜ ⎜ ⎟
⎝ ⎝ 2⎠ ⎝ 2⎠ ⎠
2‐14 

7. Discusión. 

 
 

Fig. 15 
 

  Con ayuda de la Fig. 15 podemos simplificar estas expresiones a lo siguiente: 
 
Q Campo eléctrico de un trozo de alambre de longitud L 
(19a)       Ex = (sen α1 − sen α 2 )  
4 πε0 ⋅ Lx y carga uniforme Q. 
  Note que estas expresiones ya no dependen de dónde 
Q ⎛ 1 1 ⎞ se  coloque  el  origen  de  coordenadas  del  sistema  XY. 
(19b)      Ey = ⎜ − ⎟ 
4 πε0 L ⎝ L2 L1 ⎠ El  Eje  Y  debe  estar  a  lo  largo  del  alambre  y  dirigido 
del extremo designado “1” hacia el extremo “2”. 
 
Los ángulos  α1 y  α2 son los que forman las líneas desde los extremos del alambre hasta  P. Estos ángulos 
se miden desde la dirección positiva del eje X en sentido antihorario. Si se miden en sentido horario deben 
tomarse como negativos. 
  En  la  Fig.  16  se  muestran  esquemáticamente  unas  líneas  del  campo  eléctrico  del  alambre.  Son 
hipérbolas cuyos focos son los extremos del alambre. 
 

 
Fig. 16 
2‐15 

  Examinemos las expresiones resultantes. 
  El campo es infinito en los puntos dentro del alambre, donde x = 0. 
  En el eje de simetría perpendicular al alambre (es decir, a lo largo del Eje X) se anula la componente 
Ey, puesto que allí se verifica que L1 = L2 (Fig. 17). En este caso, notando que α2 es negativo y que 
α1 = – α2 tenemos 
 
Q Q
    Ex = (sen α1 − sen α 2 ) = (2 sen α1 )  
4 πε0 ⋅ Lx 4 πε0 ⋅ Lx
 

o sea 
 
 
Campo eléctrico (componente única  Ex) de un trozo de 
Q 1
(20)    Ex = ⋅  
4 πε0 x x + (L / 2)2
2 alambre  en  los  puntos  sobre  su  eje  de  simetría 
perpendicular (Eje X, origen en el centro del trozo). 
 
 

   
   
Fig. 17  Fig. 18 
 
  Sobre  el  eje  longitudinal  del  alambre,  fuera  de  él  (es  decir,  a  lo  largo  del  Eje  Y),  se  anula  la 
componente  Ex,  puesto  que  allí  se  tiene  α1  =  α2  =  90°  en  la  parte  superior  y  α1  =  α2  =  −90°  en  la  parte 
inferior (Fig. 18). En este caso, la componente única del campo es 
 
 
Campo  eléctrico  (componente  única  Ey)  de 
⎛ ⎞
Q ⎜ 1 1 ⎟ Q 1 un  trozo  de  alambre  en  los  puntos  sobre  su 
21)  Ey = ⎜ − ⎟ =  
4 πε0 L ⎜ y − L y + L ⎟ 4 πε0 2 ⎛ L ⎞2 propio  eje  (Eje  Y,  origen  en  el  centro  del 
⎝ 2 2⎠ y −⎜ ⎟ trozo). Válida para  |y| > L/2. 
⎝2⎠
 
  En las partes negativas de los Ejes X y Y las expresiones de los campos son las mismas, (20) y (21), 
excepto que habría que cambiarles el signo algebraico. 
2‐16 

EJEMPLO  2.10     Usando  el  campo  del  trozo  de  alambre  obtenido  en  la  sección  anterior,  obtener  por  un 
proceso de límite el campo eléctrico de un alambre recto infinito. 
‐‐‐‐‐‐‐‐‐‐‐‐‐‐‐‐‐‐‐‐‐‐‐‐‐‐ 
  Usaremos las expresiones para el campo de un trozo de alambre, dadas por las Ecs. (19a,b‐p14), a 
saber, 
 
Q Q ⎛ 1 1 ⎞
    Ex = (sen α1 − sen α 2 )       Ey = ⎜ − ⎟ 
4 πε0 ⋅ Lx 4 πε0 L ⎝ L2 L1 ⎠
 
  Para  obtener  el  campo  de  un  alambre  recto  infinito  haremos  tender  la  longitud  L  a  infinito.  Pero 
debemos  advertir  que  no  basta  con  hacer  tender  L  →  ∞  en  las  expresiones  Ex  y  Ey.  Debemos  tomar  en 
cuenta que la densidad de carga λ debe mantenerse constante, así que el límite correcto es 
 
Q
    L → ∞       y    Q → ∞    con    λ= = constante 
L
 
Entonces, antes de calcular el límite incluyamos la constante λ en las expresiones de Ex y Ey: 
 
λ λ ⎛ 1 1 ⎞
    Ex = (sen α1 − sen α 2 ) Ey = ⎜ − ⎟ 
4 πε0 x 4 πε0 ⎝ L2 L1 ⎠
 
Está claro que cuando L tiende a infinito,   α1  → 90°  y   α2  →  −90° (Véase la Fig. 15 en la página 14), y la 
diferencia entre L1 y L2 tiende a cero. Las componentes de E tienden a 
 
 
Campo  eléctrico  de  un  alambre  infinito  cargado 
λ
(22a)       Ex =   uniformemente con densidad lineal  λ, en un punto 
2 πε0 ⋅ x
a distancia “x” del alambre. 
 
(22b)       Ey = 0 
 
  Las  líneas  de  fuerza  del  campo  eléctrico  de 
un  alambre  infinito  son  ortoaxiales,  es  decir, 
perpendiculares  al  alambre.  Si  λ  es  positiva,  las 
líneas “nacen” en el alambre (caso de la Fig. 19); el 
campo  E  se  dirige  huyendo  del  alambre.  Si  λ  es 
negativa,  las  líneas  terminan  en  el  alambre, 
“mueren”  en  él.  Las  líneas  de  fuerza  están 
contenidas  en  una  familia  de  planos 
perpendiculares al alambre.   
 
 
Fig. 19 
 
2‐17 

 
EJEMPLO  2.11.   Refiriéndonos  a  la  Fig.  20, 
calcular el campo eléctrico del trozo de alambre 
de  longitud  L,  cargado  uniformemente  con 
carga  total  Q,  en  el  punto  cuyas  coordenadas 
son (L, L/4). 
 
  Repitamos  aquí  las  fórmulas  (19a,b) 
encontradas en la página 14: 
 
Q
  Ex = (sen α1 − sen α 2 )    
4 πε0 ⋅ Lx
Fig. 20 
Q ⎛ 1 1 ⎞
  Ey = ⎜ − ⎟ 
4 πε0 L ⎝ L2 L1 ⎠
 
  Tracemos  las  longitudes  L1  y  L2  desde   
los extremos del alambre hasta el punto campo 
P,  así  como  los  ángulos  α1  y  α2,  como  se 
muestra en la Fig. 21. (Ojo: L1 se mide desde el 
extremo con coordenada Y negativa). 
  Recordemos que estos ángulos se miden 
desde  la  dirección  horizontal  en  cada  extremo 
y en sentido antihorario, de tal manera que en 
este caso  α1 es positivo y α2 negativo. 
  Tenemos 
   
⎛ 3L ⎞ 2
2 25L2 5L Fig. 21 
  L1 = ⎜ ⎟ +L = =  
⎝ 4 ⎠ 16 4
 
2
⎛L⎞ 17 L
  L2 = ⎜ ⎟ + L2 =  
⎝4⎠ 4
 

3L / 4 3L / 4 3 L/4 L/4 1
  sen α1 = = =       sen α 2 = − =− =−  
L1 5L / 4 5 L2 17 L / 4 17
 
Sustituyendo en las expresiones de Ex y Ey, 
 
Q ⎛3 1 ⎞ Q ⎛3 1 ⎞
    Ex = ⎜ − (− )⎟ = ⋅⎜ + ⎟ 
4 πε0 ⋅ L ⋅ L ⎝ 5 17 ⎠ 4 πε0 ⋅ L2 ⎝5 17 ⎠
 
Q ⎛ 1 1 ⎞ Q ⎛ 1 1⎞
    Ey = ⎜
⎜ − ⎟=
⎟ ⎜ − ⎟ 
4πε0 L ⎝ 17 L / 4 5L / 4 ⎠ πε0 L2 ⎝ 17 5 ⎠
2‐18 

EJEMPLO  2.12.     Tres  barras  de  la  misma  longitud  L,  cargadas  uniformemente,  forman  un  triángulo 
equilátero,  como  se  muestra  en  la  Fig.  22.  La  barra  en  la  base  porta  carga  “–Q”,  y  las  otras  dos  barras 
portan carga “Q” cada una. Calcular el campo eléctrico en el baricentro del triángulo. 
   

 
 
Fig. 22  Fig. 23 
 
  El baricentro del triángulo es el punto de intersección de las medianas, que son las rectas trazadas 
desde cada vértice hasta el punto medio del lado opuesto, o sea, en la Fig. 23, las rectas AM1, BM2 y CM3. 
  El baricentro  D divide a cada mediana en una relación 1:2, de tal manera que las longitudes de los 
segmentos DM1, DM2 y DM3 son iguales todas a la tercera parte de la longitud de la mediana. 
  Se tiene entonces 
 

1 1 3 3
    DM1 = DM2 = DM3 = 
3
( L sen 60° ) = ⋅
3 2
L=
6

 

  En  la  Fig.  23  se  muestran  los  campos  eléctricos  E1,  E2  y  E3  producidos  en  el  baricentro  D 
respectivamente por las barras BC, AC y AB. 
  Aplicaremos ahora la fórmula (20‐p15), que nos da la magnitud de cada campo: 
 
Q 1
    E= ⋅  
4 πε0 x x2 + (L / 2)2
 

3
A una distancia  x = L  tenemos las magnitudes 
6
 
Q 1 Q 6
    E1 = E 2 = E 3 = ⋅ = ⋅ 2 
4 πε0 2
⎛ 3L ⎞ ⎛ L ⎞ 2 4 πε0 L
3
L ⎜ ⎟ +⎜ ⎟
6 ⎝ 6 ⎠ ⎝2⎠
 

El campo total en el baricentro tiene solamente componente vertical, igual a 
 
Q ⎛ 6 6 6 ⎞ Q 12
    E y = E1y + E 2y + E 3y = ⎜ 2 cos 60° + 2 cos 60° + 2 ⎟= ⋅  
4 πε0 ⎝ L L L ⎠ 4 πε0 L2
2‐19 

 
2.5. CAMPO ELÉCTRICO DE UNA ESPIRA CIRCULAR DE RADIO “a”, CARGADA UNIFORMEMENTE CON CARGA 
  TOTAL “Q”, EN UN PUNTO CUALQUIERA SOBRE SU EJE DE SIMETRÍA PERPENDICULAR. 

 
  El  cálculo  de  este  campo  eléctrico  en  un  punto  arbitrario  del  espacio  (x,  y,  z)  conduce  a  unas 
integrales  no  elementales.  Sin  embargo,  es  fácil  de  calcular  para  puntos  campo  situados  sobre  el  eje  de 
simetría perpendicular al plano de la espira, cosa que haremos en este ejemplo. 
  Seguiremos los mismos pasos que en el ejemplo relativo al trozo de alambre recto. 
 
1. Hacer una figura clara de la situación y escoger un sistema de Ejes XYZ conveniente. 
  Observe  la  Fig.  24:  hemos  colocado  la  espira  sobre  el  plano  XY,  y  escogido  los  Ejes  XYZ  simétri‐
camente, como vemos en dicha figura. El centro de la espira coincide con el origen del sistema XYZ. 
 

 
Fig. 24 
 

2. Marcar el punto campo P(x, y, z) y el punto fuente general P’(x’, y’, z’). 
  Deseamos calcular el campo  E en un punto arbitrario  P(0, 0, z) del Eje Z. Este es entonces el punto 
campo. 
  El  punto  fuente  general  es  un  punto  cualquiera  de  la  espira.  Es  un  punto  variable  cuyas 
coordenadas  son  P’(x’,  y’).  De  hecho  aquí  conviene  localizar  a  P’  no  mediante  sus  coordenadas 
cartesianas, sino mediante el ángulo θ’ que forma la línea OP’ con el Eje X. 
 
3. Obtener la magnitud del campo eléctrico dE producido por el elemento de carga en el 
  punto campo P. 
  El elemento de carga  dQ, situado en el punto  P’, produce en  P un campo  dE cuya magnitud  dE 
viene dada por la expresión del campo de una carga puntual, es decir, 
 
1 dQ
    dE = ⋅  
4 πε0 R 2
 
Como vemos en la Fig. 24, dE está dirigido a lo largo de la línea P’P. De la misma figura obtenemos 
 
2‐20 

    R = a2 + z2  
2‐21 

de tal manera que 
 
1 dQ 1 dQ
(23)       dE = ⋅ 2 = ⋅ 2  
4 πε0 R 4 πε0 a + z 2
 
4. Trazar por separado el detalle del elemento de carga  dQ. Expresar su longitud   ds’   en términos de 
las diferenciales de las coordenadas del punto fuente. 
  Veamos  ahora  el  detalle  del  elemento  de   
carga,  que  se  muestra  sombreado  en  la  Fig.  25.  Es   
un  pequeño  arco  de  círculo  que  subtiende  un 
ángulo dθ’ en el origen. Su longitud es ds’ = a dθ’. 
  La relación  dQ = λ ds’ se convierte en 
 
    dQ = λ a dθ’ 
 
  Por  otra  parte,  la  densidad  de  carga  de  la 
espira, como es constante, se obtiene dividiendo su 
carga total Q por su circunferencia, o sea, 
   
Q  
    λ=   Fig. 25 
2 πa
 
de tal manera que obtenemos para dQ la expresión 
 
Q Q
(24)       dQ = λ ⋅ adθ′ = ⋅ adθ′ = ⋅ dθ′  
2 πa 2π
 
5. Sustituir dQ en la expresión para dE, y obtener las componentes dEx y dEy del campo. 
  Sustituyendo lo anterior en la expresión para dE, 
 
1 dQ Q dθ′
    dE = = ⋅ 2  
4 πε0 a + z
2 2
4 πε0 ⋅ 2 π a + z 2
 
  Volvamos  a  la  Fig.  24,  que  muestra  el  campo  dE  producido  por  el  elemento  de  carga  en  P’.  Está 
claro que los campos diferenciales dE producidos por los demás elementos de carga “giran” alrededor del 
Eje Z, de tal manera que las componentes horizontales de todos estos campitos se cancelan. Es decir, por 
simetría  el  campo  E  en  P  tendrá  solamente  componente  Z.  Dado  que  dE  forma  con  el  Eje  Z  un  ángulo 
cuyo coseno es   z / a 2 + z 2 , tendremos 
 
Q dθ′ z Q zdθ′
    dE z = ⋅ = ⋅ 
4 πε0 ⋅ 2 π a + z
(
a 2 + z 2 4 πε0 ⋅ 2 π a 2 + z 2 )
2 2 3/2
2‐22 

6. Evaluar las integrales de dEx y dEy. Los límites de integración deben cubrir toda la distribución 
  de carga. 
  La  integral  de  dEz  es  inmediata,  puesto  que  la  única  variable  es  θ’  (observe  que  tanto  z  como  el 

( )
3/2
factor  a + z
2 2
 son constantes y pueden sacarse de la integral). El resultado es 

 
Q z 2π
    Ez = ⋅ ∫0 dθ ʹ  
(
4 πε0 ⋅ 2 π a + z 2
)
2 3/ 2

 
 
Campo eléctrico de una espira circular de radio “a” 
Q z
(25)    Ez =   y  carga  total  uniforme  “Q”,  en  un  punto  sobre  su 
(
4 πε0 a 2 + z 2
)
3/ 2
eje  de  simetría  perpendicular,  situado  a  una 
distancia “z” del centro de la espira. 
 
La expresión (25) es válida también para z < 0. 
 
7. Discusión. 
a) Cuando z es muy grande en comparación con “a” se puede despreciar el término a2 en el denominador 
de (25), y entonces el campo es aproximadamente igual a 
 
Q
     Ez =  
4 πε0 z 2
 
o sea, desde lejos, la carga del anillo se ve como si fuera puntual, como era de esperarse. 
 
b) Si z es muy pequeña, o sea cerca del centro de la espira, podemos aproximar el campo desarrollándolo 
en potencias de la cantidad pequeña  z2/a2 << 1: 
(Para ello usamos el desarrollo binomial  (1 + x)n ≈ 1 + nx + ... ) 
 
3

−3 ⎛ z2 ⎞ ⎛ 3z 2 ⎞ 1
1 1 1 2
  = = = a ⎜⎜ 1 + ⎟ = a −3 ⎜⎜ 1 − 2 + ... ⎟⎟ ≈ 3  
3 3 3
a 2 ⎟⎠
(a ) ⎝ ⎝ ⎠ a
2a
2
+z 2 2 ⎡ 2⎛ z2 ⎞⎤ 2 ⎛ z 2 ⎞2
⎢a ⎜ 1 + 2 ⎟⎥ a3 ⎜ 1 + 2 ⎟
⎣⎢ ⎝ a ⎠ ⎦⎥ ⎝ a ⎠
  
(Note que esto equivale a despreciar el término z2 en el binomio elevado a la 3/2). Entonces, muy cerca del 
centro del anillo, sobre el eje de simetría, tendremos 
 
Qz
(26)       Ez =    
4 πε0 a 3
 
2‐23 

 
EJEMPLO 2.13.  Se coloca una espira (radio “a”, carga uniforme “Q”) de tal modo que el Eje X pasa por su 
centro  perpendicularmente  al  plano  de  la  espira.  ¿En  qué  punto  “xm”  ocurre  el  máximo  del  campo 
eléctrico? 
  La Ec. (25) de la sección precedente nos da el 
campo de la espira a lo largo del Eje X en la forma 
 
Q x
Ex =
(
4 πε0 a 2 + x 2
)
3/ 2

     

 
El máximo del campo  Ex se encuentra imponiendo 
la condición 
 
dE x
    =0 
dx  
  Fig. 26 
  Pongamos  
 

( )
Q −3 / 2
    Ex = x a 2 + x2  
4πε0
 
y utilicemos la fórmula para la derivada de un producto: 
 
dE x Q ⎡ 2 2 −3 / 2 ⎛ 3 ⎞⎤
    = ⎢(a + x ) + x ⋅ ⎜ − (a 2 + x 2 )−5 / 2 ⋅ 2x ⎟ ⎥ = 0 
dx 4 πε0 ⎣ ⎝ 2 ⎠⎦
 
dE x Q ⎡ 1 3x 2 ⎤
    = ⎢ 2 − 2 5/ 2 ⎥
=0 
dx 4 πε0 ⎣ (a + x )
2 3/2
(a + x ) ⎦
2

 
Multiplicando toda la ecuación por  (a 2 + x 2 )5 / 2  se llega a 
 
    (a 2 + x 2 ) − 3x 2 = 0  
 
  Existen entonces dos soluciones, dadas por 
 
2a 2a
    x= y x=−  
2 2
 
La primera de estas corresponde al máximo de Ex. 
2‐24 

 
EJEMPLO  2.14.    Dos  espiras  circulares,  una  con   
carga  uniforme  Q  y  radio  R,  y  la  otra  con  carga 
uniforme  –Q/4  y  radio  R/2,  tienen  sus  planos 
paralelos y el mismo eje de simetría perpendicular 
(Eje X). Los centros de las espiras están localizados 
en  x  =  0  y  x  =  s.  Calcular  el  campo  eléctrico 
producido  por  las  dos  espiras  en  el  punto  de 
coordenada x = 2s. 
   
Fig. 27 
 

  La Ec. (25‐p21) nos da el campo de una espira de carga Q y radio R a lo largo del Eje X en la forma 
 
c arg a x
E=
(
4 πε0 radio 2 + x 2
)
3/ 2

     
 
(En la Ec. (25) hemos cambiado Q por “carga”, “a” por “radio” y “z” por “x”, para adaptarla a este caso). 
  Entonces el campo de la espira de carga Q, en el punto campo considerado, es 
 
Q 2s Q 2s
    E1 = =  
4 πε0 (R + (2s) )
2 2 3/ 2
4 πε0 (R + 4s 2 )3 / 2
2

 
y el campo de la espira de carga –Q es 
 
−Q / 4 s Q/4 s Q 2s
    E2 = =− =−  
4 πε0 ((R / 2) + s )
2 2 3/2
4 πε0 ⎛ R 2 3 / 2
4 πε0 (R + 4s 2 )3 / 2
2
2⎞
⎜ +s ⎟
⎝ 4 ⎠
El campo total es 
 

    E = E1 + E2 = 0 
 
EJEMPLO 2.15.  Una carga puntual negativa –q, de masa  m, se libera del reposo sobre el eje de simetría 
perpendicular de un anillo circular de carga positiva Q y radio  R, muy cerca de su centro. Demostrar que 
la carga ejecuta un movimiento armónico simple y calcular su frecuencia angular de oscilaciones. 
 

 
Fig. 28 
2‐25 

 
  La carga –q experimenta la fuerza del campo eléctrico Ez debido al anillo. Esta fuerza es 
 
    F = –q Ez 
 
  Por  otra  parte,  la  Ec.  (26‐p21)  nos  da  el  campo  eléctrico  del  anillo  muy  cerca  de  su  centro,  en  la 
forma 
 

Qz
Ez =
4πε0 R 3
     
 
de tal manera que la fuerza es 
 
qQ
F=− ⋅z
4πε0 R 3
     
 
Abreviemos la constante 
 
qQ
K=
4 πε0 R 3
     
 
con lo que 
 
    F = – K z 
 
Se  trata  en  este  caso  de  una  fuerza  restauradora,  análoga  a  la  fuerza  que  un  resorte  lineal  de  constante 
elástica “k” ejerce sobre una masa sujeta a su extremo libre (F = – k x). El movimiento de la carga –q es 
completamente análogo al de la masa en el resorte; es un movimiento armónico simple. 
  Para la masa en el resorte, la frecuencia angular de oscilaciones es 
 
k
    ω = 2π  
m
 
Por analogía, la frecuencia angular de la carga –q será 
 
K qQ
    ω = 2π = 2π  
m 4 πε0 mR 3
 
2‐26 

 
2.6. CAMPO ELÉCTRICO PRODUCIDO POR UNA LÁMINA DELGADA PLANA CIRCULAR DE RADIO “a” 
   Y CARGA ELÉCTRICA TOTAL Q DISTRIBUÍDA UNIFORMEMENTE. 

 
  Coloquemos  la  lámina  circular  (o  “disco”)   
sobre  el  plano  XY,  con  su  centro  en  el  origen  O, 
como  se  muestra  en  la  Fig.  29.  El  campo  eléctrico 
posee simetría rotacional alrededor del Eje Z. 
  El punto campo es un punto arbitrario sobre 
el Eje Z, de coordenadas cartesianas P(0, 0, z). 
  El punto fuente general es un punto arbitrario 
del  disco,  donde  suponemos  localizada  la  carga 
puntual  dQ.  Conviene  usar  aquí  coordenadas 
polares en el plano XY, con lo que tendremos 
P’(r’, θ’), donde r’ y θ’ son variables cuyo rango de 
variación viene dado en la Fig. 29. 
  El campo eléctrico dE producido por la carga 
puntual  dQ  en  el  punto  P  está  dirigido  según  la   
línea P’P. Como usualmente, denotamos con “R” la   

distancia entre P’ y P, igual a  0 ≤ r’ ≤ a    0 ≤ θ’ ≤ 2π 


   
Fig. 29 
    R = z 2 + r ′2  
 

  Entonces la magnitud de dE es 
 
1 dQ 1 dQ
(27)    dE = ⋅ 2 = ⋅ 2  
4 πε0 R 4 πε0 z + r′2
 

  Consideremos ahora el elemento de carga “dQ”. 
  Puesto  que  la  carga  está  distribuída  uniformemente,  la  densidad  de  carga  superficial  del  disco  es 
simplemente 
 

Carg a total del disco Q


(28)    σ= =  
Área del disco π a2
 

  Por  otra  parte,  como  vemos  en  la 


Fig. 30, la carga dQ ocupa el elemento de 
área en coordenadas polares. Su área es 
 
(29)    da’ = r’ dr’ dθ’ 
 
 
Fig. 30 
Poniendo (28) y (29) en la relación dQ = σ da’ obtenemos 
 
Q
(30)    dQ = σ daʹ = r′dr′dθ′  
πa 2
2‐27 

Sustituyendo (30) en (27), 
 
1 dQ Q r′dr′dθ′
    dE = ⋅ 2 = ⋅ 2  
4 πε0 R (4 πε0 ) π a z + r′2
2

 
Por  simetría,  el  campo  del  disco  tiene  solamente  componente  Z.  Dado  que  dE  forma  con  el  Eje  Z  un 
z
ángulo α cuyo coseno es “ z/r “, o bien  , dicha componente es 
z 2 + r ′2
 
Qz r′dr′dθ′
    dE z = dE cos α = ⋅  
(4 πε0 ) π a ( )
2 3/2
z 2 + r ′2
 
  Para evaluar  Ez debemos efectuar una integral doble, sobre las variables  r’ y  θ’. Para ello hacemos 
uso del teorema dado por la ecuación (8‐p5). Obtenemos 
 
⎛ ⎞
r′dr′ ⎟⎛ 2π ⎞
⋅ ⎜⎜
Qz a
    Ez = ∫0 ⎟⎜ ∫0 dθ′ ⎟  
(4 πε0 )πa 2 ⎜
⎝ (z 2
+ r′ )
2 3/ 2
⎟⎝

 
La  fórmula  (17‐p13)  nos  permite  evaluar  la  primera  integral.  La  segunda  es  simplemente  igual  a  2π.  El 
resultado, puesto en términos de la densidad de carga, es 
 
σ ⎛ z ⎞ Campo  eléctrico  de  una  lámina  circular  de  radio 
(31)    Ez = ⎜⎜ 1 − ⎟⎟   “a”  y  densidad  de  carga  “s”,  en  un  punto  a  altura 
2ε 0 ⎝ z2 + a 2 ⎠ “z” sobre su eje de simetría perpendicular. 
 
Discusión. 
  Durante el cálculo de  Ez hemos supuesto que  z > 0. Por simetría, está claro que la expresión de  Ez 
válida para z < 0 es 
 

σ ⎛ z ⎞
(32)        Ez = − ⎜⎜ 1 − ⎟⎟        Campo de un disco (para z < 0)         
2ε 0 ⎝ z2 + a 2 ⎠
 

  Hay un par de puntos importantes que podemos extraer de la expresión (31). 
  Primero: el campo eléctrico no es singular en z = 0, es decir, en el centro del disco. 
  Al acercarnos al centro del disco por arriba (z → 0+), el campo tiende al valor 
 
σ
(33)    Ez =  
2ε0
 

Acercándonos al centro del disco por valores negativos de z (o sea z → 0–), el campo tiende a 
 
σ
(34)    Ez = −  
2ε 0
2‐28 

Segundo: ¡El campo eléctrico en el centro del disco no depende del radio “a”! 
  En la Fig. 31 se muestran esquemáticamente las líneas de fuerza del campo eléctrico del disco. 
 

 
Fig. 31 
 
EJEMPLO  2.16.   Consideremos  una  lámina  circular  (o  “disco”)  de  radio  arbitrario,  cargada 
uniformemente. Extraigamos del disco un trozo circular concéntrico y situemos una carga testigo  q en el 
centro  del  hueco,  como  vemos  en  la  Fig.  32.  Demostrar  que  esta  carga  no  sufre  fuerza  eléctrica  neta, 
porque el campo del ánulo circular es cero en su centro. 
‐‐‐‐‐‐‐‐‐‐‐‐‐‐‐‐‐‐‐‐‐‐‐‐‐‐‐‐‐‐‐‐ 
  La  explicación  se  basa  en  el   
principio de superposición de campos. 
  Podemos  ver  el  campo  del  ánulo 
en  el  centro  como  la  suma  vectorial  de 
dos  campos:  el  de  un  disco  sólido, 
 
cargado positivamente con densidad  σ, y   
el de otro disco concéntrico con la misma  Fig. 32 
densidad de carga pero negativa (Fig. 33) 
 
Sabemos que el campo del disco positivo   
en su centro (por la parte de arriba, o sea 
en z = 0+) es (Ec. (33)) 
 
σ
  Ez =   ↑ 
2ε o
 
Por  otra  parte,  el  campo  del  disco 
 
negativo  en  su  centro  (en  z  =  0+)  no 
 
depende  de  su  radio,  que  puede  ser  tan 
Fig. 33 
pequeño como se quiera. 
  Este campo apunta hacia abajo en z = 0+. Ambos campos se cancelan, QED. El mismo resultado se 
obtiene si consideramos el campo total de los dos discos de la Fig. 33 en la parte de abajo del centro. 
2‐29 

EJEMPLO 2.17.   Calcular el campo eléctrico de una hoja plana infinita cargada con densidad superficial 
constante de carga “σ”. 
‐‐‐‐‐‐‐‐‐‐‐‐‐‐‐‐‐‐‐‐‐‐‐‐‐‐‐‐‐‐‐‐‐ 
  El resultado principal de la Sección 5 (Ec. 31‐p26) puede utilizarse para calcular este campo. Veamos. 
  El campo de una  placa infinita cargada con densidad superficial constante  σ se obtiene haciendo 
tender el radio del disco de la Sección 5 a infinito. El resultado es un campo constante en todo el espacio: 
 
σ ⎛ z ⎞ σ
    lím ⎜1 − ⎟⎟ =  
a →∞ 2ε 0 ⎜ z + a2
2 2 ε
⎝ ⎠ 0

  
σ
(35)    E=     Campo de una hoja infinita     
2ε o
 
  Esta  clase  de  campo  se  denomina  uniforme.  Las  líneas  de  fuerza  del  campo  son  rectas 
perpendiculares al plano de la placa, como vemos en la Fig. 34. 
 

 
 
Fig. 34 
 
  Si bien la expresión (35) del campo E es válida para una placa infinita, se puede usar cuando la placa 
tiene gran extensión y el punto campo considerado está cerca de la parte central de la misma. 
  Un dispositivo que consta de dos placas paralelas, de separación  d y la misma área  A, una de ellas 
cargada positivamente con densidad +σ y la otra negativamente con la misma densidad (−σ), constituyen 
lo que se denomina un capacitor de placas paralelas (Fig. 35). 
  Si  A  es  grande  y  d  es  pequeña,  el  campo  eléctrico  del  capacitor  es  prácticamente  cero  fuera  del 
mismo, y dentro, no muy cerca de las orillas, es igual a 
 
σ
(36)    E=        Campo de un capacitor de placas paralelas   
ε0
 
2‐30 

 
 

 
 
Fig. 35 
 
  Esto  lo  podemos  demostrar  superponiendo  los  campos  de  ambas  placas  en  todo  el  espacio.  En  la 
región  por  encima  de  la  placa  positiva,  lo  mismo  que  en  la  región  por  debajo  de  la  placa  negativa,  los 
campos  de  una  y  otra  placa  poseen  direcciones  opuestas  y  se  anulan.  En  la  región  entre  las  placas  los 
campos tienen la misma dirección (hacia abajo) y se suman. 
 
2.7. SUPERPOSICIÓN VECTORIAL DE CAMPOS ELÉCTRICOS. 
  Utilizando  el  principio  de  superposición  vectorial  de  los  campos  eléctricos,  podemos  calcular  el 
campo eléctrico generado por dos o más distribuciones de carga de las consideradas en las secciones 3, 4 y 
5, si bien el punto campo debe estar localizado convenientemente. Veremos algunos ejemplos. 
 
EJEMPLO  2.18.     A  un  alambre  recto  vertical  infinito  le  falta  un  pedazo  de  longitud  ʺaʺ.  En  el  extremo 
inferior de este pedazo (punto F) toca un disco de radio ʺaʺ, cuyo plano es perpendicular al alambre, como 
se muestra en la Fig. 36. Calcular el campo eléctrico en el punto P a una altura ʺaʺ sobre el centro del disco 
y en su eje de simetría perpendicular. El alambre tiene densidad de carga λ, y el disco σ (constantes). 
‐‐‐‐‐‐‐‐‐‐‐‐‐‐‐‐‐‐‐‐‐‐‐‐‐‐‐‐‐‐‐‐‐‐‐ 
 
 

   
Fig. 36  Fig. 37 
 

  El  campo  deseado  lo  podemos  obtener  superponiendo  los  campos  de  las  tres  distribuciones 
mostradas  en  la  Fig.  37.  Note  que  el  trozo  de  alambre  de  longitud  ʺaʺ  y  carga  negativa  (densidad  −λ) 
cancela el campo del trozo positivo frente a él, quedando la distribución original de la Fig. 36. Colocamos 
un sistema de ejes X y Y como se muestra en la Fig. 36, de modo que las coordenadas del punto campo son 
P(a, a). A continuación las contribuciones de cada distribución: 
2‐31 

 
Fig. 38 
 
Trozo  de  alambre.  Usando  las  fórmulas  (19a,b) 
obtenidas en la página 14 tenemos 
 

λ 2 (
λ 2− 2 ) 
  x =−
E(1) y =−
E(1)
8 πεoa 8 πεo a
   
 

Disco. De la fórmula (31) de la página 26, 
 
(
σ 2− 2 ) 
    y =
E(2)
4ε o
 
 
   
 

Alambre  infinito..  De  las  fórmulas  (22a,b)  de  la 


página 16, 
 
λ
    x =
E(3)  
2 πε 0 a
 
  Superponiendo  los  campos  obtenidos 
tenemos para el campo en P la expresión 
 
    (
E = E(1) (3) (1)
) (
x + Ex i + E y + E y j  
(2)
)
 
Fig. 35 
O sea 
 

λ ⎛ 2⎞ 2− 2 ⎛ λ ⎞
    Ex = ⎜1 − ⎟ Ey = ⎜− + σ⎟  
2 πε0 a ⎝ 4 ⎠ 4ε0 ⎝ 2 πa ⎠
 
2‐32 

 
EJEMPLO 2.19.  Una lámina plana muy grande, cargada uniformemente con densidad superficial de carga 
“σ”, tiene una porción hueca circular. El radio de esta parte circular es “R”. Calcular el campo eléctrico de 
la lámina, cerca de ella, sobre el eje perpendicular que pasa por el centro de la parte circular. 
 

 
Fig. 36 
 
  Dado  que  la  lámina  es  muy  grande,  supondremos  que  es  infinita.  El  campo  E  que  deseamos 
calcular es la superposición vectorial de dos campos, a saber: 
 
• El campo de una lámina plana infinita, cuya densidad de carga es σ, dado por 
 
σ
    E1 =  
2ε 0
 
  (Consulte la Ec. (35) en la página 28) 
 
• El campo de una lámina circular plana de radio R y densidad de carga –σ, dado por 
 
−σ ⎛ z ⎞
    E2 = ⎜⎜ 1 − ⎟⎟
2ε 0 ⎝ z + R2
2
⎠ 
 
  (Consulte la Ec. (31) en la página 26). 
 
  Tenemos entonces 
 
σ −σ ⎛ z ⎞
    E = E1 + E 2 = + ⎜⎜ 1 − ⎟⎟  
2ε 0 2ε 0 ⎝ R 2 + z2 ⎠
 
σz
    E=  
2ε 0 R 2 + z 2
2‐33 

 
2.7. PROBLEMAS. 
1.  Una esfera sólida de radio R posee una densidad volumétrica de carga no uniforme  ρ que varía con la 
distancia “r” al centro de la esfera conforme a la relación 
 
C
    ρ=  
r2
 
donde C es una constante cuyas unidades son C/m. Calcular esta constante sabiendo que la carga total de 
la esfera es Q. 
Sugerencia. Dada la simetría esférica de la función  ρ se puede tomar el elemento de volumen de la esfera 
en la forma dV = 4πr2 dr. 
Resp. C = Q/4πR 
 
2.  La superficie lateral de un cascarón cilíndrico (“tubo”) de radio R = 0.65 m y altura h = 0.82 m posee una 
distribución superficial de carga eléctrica dada por σ = 130 μC/m2. Calcular la carga total alojada en dicha 
superficie lateral. 
Resp. Q = 435.36 μC. 
 
3.  Una lámina circular delgada de radio “a” posee una carga total uniforme “Q”. Dentro de la lámina se 
toma un anillo circular cuyos radios menor y mayor son “r” y “r + dr”, donde “dr” es una cantidad infini‐
tesimal. Se  desea describir  la  densidad de  carga de  este  anillo  mediante una densidad lineal  de  carga  λ 
¿Cuál sería el valor de esta densidad? 
 

 
 
Resp.  
λ = σ dr 
 
4.   Dos  esferas  sólidas  contienen  la  misma  carga  total  Q,  uniformemente  distribuída.  Los  radios  de  las 
esferas son “a” y “b”. ¿Cómo se comparan sus respectivas densidades de carga volumétrica ρa y ρb? 
Resp. 
ρa b 3
=  
ρb a 3
 
2‐34 

 
5.  Un trozo de alambre recto de longitud L y carga total Q distribuída uniformemente se coloca con un 
extremo en el origen de un sistema cartesiano XY, y el otro extremo en el punto (L, 0), como se muestra en 
la figura. 
  Siguiendo  paso  a  paso  el  método  de 
integración  ilustrado  en  la  Sección  5  (página 
11),  obtenga  el  campo  eléctrico  del  alambre 
en un punto arbitrario P(x, y) del plano. 
Resp. 
Ex = (kQ/L)(1/L2 – 1/L1) 
Ey = (kQ/Ly)(cos α1 – cos α2) 

(α1 y α2 son los ángulos que forman con el Eje X las líneas desde los extremos del alambre hasta P). 
 

6.   Un  trozo  de  alambre  recto  de  longitud  L  se 


coloca  a  lo  largo  del  Eje  Y,  con  su  centro  en  el 
origen  de  coordenadas.  La  carga  del  alambre,  Q, 
está uniformemente  distribuída.  Calcular  el  campo 
eléctrico  producido  por  el  alambre  en  un  punto 
arbitrario del Eje Y, fuera del alambre. 
No  utilice  las  fórmulas  de  la  Sección  5;  resuelva 
desde  primeros  principios,  usando  el  método  de 
integración dado allí. 
Resp.   kQ/(y2–(L2/4)) 
 
 
7.   Utilizando  las  fórmulas  (19a,b)  dadas  en  la 
página  14,  calcular  el  campo  eléctrico  de  un 
alambre  recto  semiinfinito  con  densidad  lineal  de 
carga constante  λ, en un punto a una distancia “s” 
perpendicular a un extremo. 
Resp. 
Ex = – Ey = kλ/a 
 

 
 
2‐35 

 
8.   Calcular  el  campo  eléctrico  generado  por  un 
cuadrante  de  círculo  de  radio  “a”  y  carga  total  Q 
distribuída uniformemente, en su centro O. 
Resp. 
Ex = Ey = – 2kQ/πa2 

 
 
9.   Usando  el  resultado  del  problema  8,  calcular  el 
campo eléctrico de un aro semicircular de radio “a” 
en  su  centro.  La  carga  total  del  aro,  Q,  está 
distribuída uniformemente. 
Resp. 
  Ey = – 4kQ/πa2. 

 
10.   Usando  el  resultado  del  problema  8  (con 
adaptaciones), calcular el campo eléctrico de un aro 
semicircular  de  radio  “a”  en  su  centro.  La  mitad 
izquierda del aro posee carga total positiva Q/2, y la 
mitad  derecha  la  posee  negativa,  –Q/2,  ambas 
distribuídas uniformemente. 
Resp. 
   

 
11.  Calcular  el  campo  eléctrico  del  ánulo 
circular  que  se  muestra  en  la  figura,  en  el 
centro  del  mismo.  Los  radios  menor  y 
mayor son “a” y “b”, respectivamente, y la 
carga total Q del ánulo es uniforme. 
Resp. 
4kQ ⎛b⎞
  Ey = − ln ⎜ ⎟  
π(b2 − a 2 ) ⎝a⎠
2‐36 

 
 
12.  Un alambre infinito con densidad de   
carga  constante  λ  está  dispuesto   
tangencialmente  a  un  aro  con  densidad 
de carga constante  λ. El plano del aro es 
perpendicular  al  alambre.  Calcular  el 
campo  eléctrico  en  el  punto  P  mostrado, 
a  distancias R  del  centro C  del aro  y del 
alambre.   
Resp. Componentes del campo: 
 (λ/2πε0R),    λ/(2ε0R ⋅ 23/2)  
 
13.  Tres placas paralelas de la misma área poseen   
densidades de carga   
 
σ1 = 2,    σ2 = −5     y    σ3 = 6  
 
μC
todas en  . 
m2
Calcular  el  campo  eléctrico  en  todo  el  espacio., 
suponiendo  que  las  placas  son  muy  extensas  en   
comparación con sus distancias mutuas. 
Resp. Desde arriba de la placa superior, con c = 1/2ε0, 
3c ↑ , 1c ↓ , 9c ↑ , 3c ↓  
 
 
14.   Un  dispositivo  está  formado  por  una  placa  plana  y  extensa  (que  se  puede  considerar  infinita  para 
efectos de calcular su campo eléctrico), un disco de radio R, y un aro delgado de radio R. Los planos del 
disco  y  aro  son  paralelos  a  la  placa.  Las  densidades  de  carga  de  estos  cuerpos  son  σ,  σ  y  λ, 
respectivamente. El centro del disco está a distancia “a” de la placa, y el del aro a distancia “b” por encima 
del centro del disco. Calcular el campo eléctrico producido por estos 3 cuerpos justo arriba y debajo del 
centro del disco. 

 
Resp. 
Justo arriba del centro del disco: Ez = (1/2ε0) (2 σ –  λRb/(b2 + R2)3/2). 
3‐1 

  CAPÍTULO 3 
   
  LA LEY DE GAUSS 
 
 
 
3.1. FLUJO DE UN CAMPO VECTORIAL A TRAVÉS DE UNA SUPERFICIE. 
  Si las líneas de fuerza de un campo vectorial  E atraviesan una superficie  S , entonces existe “flujo” 
del campo a través de esta superficie. El “ flujo de  E a través de  S ” es una cantidad escalar denotada por 
ΦE(S ). 
  La expresión matemática más general del flujo de un campo vectorial a través de una superficie es 
una  integral  de  superficie,  cuya  evaluación  en  el  caso  general  queda  fuera  de  los  objetivos  del  presente 
curso. Sin embargo, existen casos en los que el flujo es fácil de evaluar, sobre todo cuando el campo E y la 
superficie S son simples, y el campo posee simetrías. A estos casos nos limitaremos en este capítulo. 
 
3.2. FLUJO DE UN CAMPO ELÉCTRICO CONSTANTE A TRAVÉS DE UNA SUPERFICIE PLANA (RECTÁNGULO). 
  El caso más simple del flujo ocurre cuando: 
• El campo E es uniforme. 
(Como por ejemplo el campo dentro de un capacitor de placas paralelas) 
• La superficie S  es un rectángulo de área A. 
 

    Un  campo  uniforme  posee  líneas  de 


fuerza  paralelas  entre  sí  e  igualmente 
espaciadas.  Supongamos  que  E  apunta 
verticalmente hacia arriba y que el rectángulo 
S está dispuesto oblicuamente con respecto a 
las líneas de fuerza (Fig. 1). Denotemos con  θ 
el  ángulo  que  forma  el  plano  de  S  con  un 
plano horizontal. 
 
  En  este  caso  se  define  el  flujo  de  E  a 
través de  S como el producto de la magnitud 
E  del  campo  y  el  área  A ⊥   que  el  rectángulo  
presenta perpendicularmente al campo: 
 
 
 
(1)    ΦE = E A⊥  
Fig. 1 
 

El área  A ⊥  es la proyección del área A sobre el plano horizontal, o sea 
 

    A ⊥  = A cos θ 
 

de tal manera que el flujo (1) queda 
 

    ΦE = E A cos θ 
3‐2 

 
  Esta expresión se puede poner en forma más conveniente y compacta como un producto escalar de 
dos vectores: uno es el llamado vector área del rectángulo, A, cuya magnitud es el área del mismo y cuya 
dirección es perpendicular al plano del rectángulo; el otro vector es el campo eléctrico E. Tenemos así 
 
 
 
Flujo de un campo eléctrico E uniforme a través de un  
(3)    φE = E • A 
rectángulo cuyo vector área es A. 
 
 
El  vector  área  A  del  rectángulo  S  tiene  por  magnitud  el  área  A  de  S  y  por  dirección  la  que  es 
perpendicular al plano de S (una de las dos posibles, escogida como convenga). 
  El  flujo  vale  cero  si  E  y  A  son  perpendiculares,  es  decir,  si  las  líneas  de  fuerza  de  E  corren 
paralelamente al rectángulo S. 
 
3.3. INTERPRETACIÓN DEL FLUJO DEL CAMPO DE VELOCIDADES DE UN FLUIDO. 
  El nombre de flujo proviene de la hidrodinámica. Si el campo vectorial en cuestión es el campo de 
velocidades de un fluido, el flujo posee una interpretación simple. Veamos. 
 

 
Fig. 2 
 

  En la Fig. 2 se muestra la sección longitudinal de una tubería cilíndrica recta, de área transversal A, 
por  la  que  circula  un  líquido  con  velocidad  constante  v.  Resulta  que  el  flujo  de  v  a  través  de  A  es  el 
volumen  de  líquido  que  pasa  por  la  sección  transversal  del  tubo  por  unidad  de  tiempo.  Esta  cantidad  se  llama 
volumen m 3
también “gasto” en hidrodinámica; sus unidades físicas son  = . Esto lo podemos demostrar 
tiempo s
como sigue: 
  Desde cualquier sección transversal del fluido (por ejemplo la sección mostrada en la Fig. 3.2, en la 
que hemos trazado los vectores  A y  v), marquemos hacia atrás una distancia  “vΔt”, igual a la que viaja 
toda partícula del fluido durante un tiempo arbitrario Δt. Entonces, todas las partículas que se encuentren 
en el cilindro de área  A y altura  vΔt lograrán pasar por la sección considerada al cabo del tiempo  Δt. El 
volumen de estas partículas es 
 
    ΔV = A v Δt 
 
Dividiendo  este  volumen  por  el  tiempo  Δt  hallamos  el  volumen  que  pasa  por  unidad  de  tiempo, 
resultando 
 
ΔV
    = Av = v • A        Q.E.D. 
Δt
3‐3 

 
 

Fig. 3 
 

  Claramente, el gasto a través de una sección oblicua como la mostrada en la Fig. 3 es el mismo que 
el ya calculado. Si el vector área de dicha sección oblicua es A, entonces el área de la sección transversal es 
A cos  θ,  donde  θ  es  el  ángulo  entre  A  y  v.  Se  sigue  nuevamente  que  para  cualquier  sección  (oblicua o 
perpendicular) 
 

    Φv = vA cos θ = v • A 
 
EJEMPLO 3.1. Un rectángulo ABCD de lados 90 mm y 60 mm tiene una arista AB sobre el plano XY,  
paralela  al  Eje  X,  como  se  muestra  en  la  Fig.  4.  El   
rectángulo se halla en el seno de un campo eléctrico 
constante  E  dirigido  a  lo  largo  del  Eje  Y,  cuya 
magnitud  es  E  =  120  N/C.  Calcular  el  flujo  del 
campo eléctrico a través del rectángulo. 
 
  Emplearemos  la  fórmula  (1),  Φ E = E A ⊥ .  El 
área  A ⊥   que  el  rectángulo  presenta  perpendicu‐
larmente  al  campo  E  es  la  del  rectángulo  B1C1CD 
que vemos en la Fig. 4. El ángulo θ que se forma en 
 
el vértice C entre los lados BC y CC1 es   
 
Fig. 4 
⎛ 40 ⎞
  θ = angsen ⎜ ⎟ = 26.39°  
⎝ 90 ⎠
 
y el área A del rectángulo ABCD es 
 
    A = 90 mm × 60 mm = 5400 (10–6 m2) = 0.0054  m2 
 
de tal manera que 
 
    A ⊥  = A cos θ = 0.0054 ⋅ cos 26.39° = 0.00484 m2 
 
Entonces el flujo es 
 
    Φ E = E A ⊥  = 120 (N/C) ⋅ 0.00484 m2 = 0.5808 N ⋅ m2/C. 
3‐4 

 
EJEMPLO  3.2.  Calculemos  el  flujo  de  un  campo  eléctrico  constante  E,  a  través  de  un  cubo  orientado 
según las líneas de fuerza (Fig. 5). 
  El  cálculo  se  fundamenta  en  la  siguiente  convención:  “El  vector  área  de  una  superficie  cerrada  se 
dirige hacia fuera de la región limitada por la superficie”. 
  Supongamos  que  el  campo  está 
verticalmente hacia arriba. 
Numeremos las caras del cubo desde i = 1 
hasta  i  =  6,  de  modo  que  i  =  1  e  i  =  2 
correspondan  respectivamente  a  las  caras 
inferior y superior. 
  Sea  Ai  el  vector  área  de  una  cara 
cualquiera “i”. El flujo a través de esta cara 
es 
 

    E • Ai 
 

  Es  evidente  que  en  las  caras 


  verticales  no  existe  flujo  puesto  que  el 
 
campo  es  perpendicular  al  vector  área 
Fig. 5 
correspondiente. 
Por otra parte, el flujo a través de las caras inferior y superior juntas es 
 

    E • A1 + E • A2 
 

  Pero  A1 =  −A2, de tal manera que el flujo es cero a través de estas dos caras, y por tanto totaliza 


cero calculado sobre todo el cubo. 
  Note que el flujo entra al cubo, o sea es negativo, por la base. Sale del cubo, o sea es positivo, por la 
tapa. El flujo saliente es igual al negativo del flujo entrante, así que el flujo total es cero. 
  Este resultado se puede generalizar a cualquier superficie cerrada. En el caso de un poliedro P  el  
flujo a través de él es (Fig. 6)   
   
N N
  Φ E (P ) = ∑ E • Ai = E • ∑ Ai = 0  
i =1 i =1
 
donde  estamos  suponiendo  que  E  es 
constante,  de  modo  que  puede  sacarse  de  la 
suma. 
Como vemos, para demostrar que ΦE(P ) = 0, 
habría  que  demostrar  que  el  área  vectorial   
total del poliedro es cero:   
 
Fig. 6 
N
∑ Ai = 0  
i =1
 

No haremos la demostración. 
3‐5 

 
 
  De hecho, el área vectorial total de cualquier   
superficie cerrada es cero (Fig. 7), así que el flujo a 
través  de  ella  de  un  campo  eléctrico  constante  es 
cero  (el  mismo  flujo  que  entra  también  sale  de  la 
superficie). 

 
Fig. 7 
 
EJEMPLO  3.3.  En  una  región  del   
espacio  existe  un  campo  eléctrico 
constante, dado en unidades S.I. por 
 
  E = 12.4 i – 9.1 j + 15.2 k 
 
Calcular  el  flujo  de  este  campo  a 
través de un triángulo cuyos vértices 
son  los  puntos  A,  B  y  C  mostrados 
en la Fig. 8. 
‐‐‐‐‐‐‐‐‐‐‐‐‐‐‐‐‐‐‐‐‐‐‐‐‐ 
  Usaremos  para  el  cálculo  la 
expresión (3), o sea 
   
  φE = E • A  Fig. 8 
 
 
  Vamos a calcular el vector área “A” del triángulo ABC. Para ello hacemos el producto vectorial 
 
1 → →
    A= AB× AC  
2
 
Tenemos (unidades m2) 
 
i j k
1 1
    A = −4 7 0 = ( 21 i + 12 j + 28 k ) = 10.5 i + 6 j + 14 k  
2 2
−4 0 3
 
(Note que las componentes del vector área son las áreas de las proyecciones del triángulo ABC sobre los 
planos  YZ, XZ y XY, es decir, las áreas de los triángulos OBC, OCA y OAB, respectivamente). 
  El flujo buscado es entonces 
3‐6 

 
    φE = E • A 
 
    φE = (12.4 i – 9.1 j + 15.2 k) esca (10.5 i + 6 j + 14 k) 
 
    ΦE = 288.4 N‐m2/C 
 
3.4. DEFINICIÓN DEL FLUJO DEL CAMPO ELÉCTRICO EN EL CASO GENERAL. 
  Pasemos ahora a la definición de flujo en el caso general. Ahora el campo eléctrico  E será variable 
(sus líneas de fuerza serán curvas) y la superficie S  será alabeada (Fig. 8). 
 

 
Fig. 8 
 

  La expresión del flujo  ΦE  =  E  •  A vale cuando el campo es constante y la superficie es plana, cosa 


que no ocurre aquí. De todas maneras podemos valernos de esta expresión utilizando el cálculo integral. 
  Dividamos la superficie dada en elementos de superficie, tan pequeños que podamos suponer que 
son  planos  y  que  el  campo  eléctrico  no  varía  apreciablemente  en  ellos.  Sea  “da”  el  área  de  un  elemento 
representativo,  E el valor del campo en el elemento, y  da el vector área asociado al mismo. Entonces el 
flujo “dΦE” a través del elemento es, por (3),  dΦE  =  E  • da, y el flujo a través de toda la superficie es la 
integral extendida sobre toda ella: 
 
   

(4)    ∫
Φ E ( S ) = E • da    Expresión general del flujo del campo eléctrico E 
a través de una superficie arbitraria S 
S
 
 

  En este curso calcularemos la integral solamente para casos muy simples. 
3‐7 

3.5. FLUJO DEL CAMPO ELÉCTRICO DE UNA CARGA PUNTUAL A TRAVÉS DE 
  UNA SUPERFICIE ESFÉRICA CENTRADA EN LA CARGA. 

  Como un primer ejemplo de cálculo de la integral (4), y como primer paso para obtener la ley de 
Gauss, calcularemos el flujo del campo eléctrico de una carga puntual Q a través de una superficie esférica 
E  de radio “R” centrada en la carga (Fig. 9). Situemos la carga  Q en el origen de coordenadas. Su campo 
eléctrico en el punto r es 
 

1 Qr
E=  
4 πεo r 3
 
Sustituyendo este campo en la expresión general del flujo tenemos 
 
Q r • da

Φ E ( E ) = E • da =
4 πεo ∫ r3
 
E E
   

   
   
Fig. 9  Fig. 10 
 
  Ahora bien, observe las Figs. 9 y 10. Se advierte que en todo punto de la superficie E , los vectores r y 
da son paralelos, de modo que el producto escalar se puede poner en la forma r • da = r da cos 0° = 
r da, y la integral queda 
 
Q rda Q da
    ΦE ( E ) =
4 πεo ∫ r 3
=
4 πεo ∫ r2  
E E
 
Pero sobre la superficie de integración E  la “r” es constante, igual al radio “R”, de modo que 
 
Q da Q Q Q
ΦE ( E ) =
4 πεo ∫ r2 =
4 πεo R E
2 ∫
da =
4 πεo R 2
⋅ 4 πR 2 =
εo
 
E
 
Hemos utilizado el hecho de que la integral de “da” sobre  E  es el área de la superficie esférica de radio 
“R”, o sea 4πR2. En conclusión: 
3‐8 

 
 
 
Flujo del campo eléctrico de una carga puntual Q 
Q
(5a)    ΦE ( E ) =   a través de una esfera E  centrada en la carga. 
εo
Este flujo no depende del radio de la esfera. 
 
 

(5b)       Si la carga Q se halla fuera de la superficie esférica, el flujo de su campo eléctrico a través 
    de dicha superficie es cero. No demostraremos esta aseveración. 
 

 
  Echemos mano de una analogía para intuir por qué el flujo del campo de Q es el mismo a través de 
toda superficie esférica centrada en la carga (independientemente del radio de la misma). 
  Pensemos en  Q como si fuese un punto donde se genera fluido que emana hacia todas direcciones, 
y en el campo eléctrico como si fuese el campo de velocidades de este fluido (que va como 1/r2). Entonces 
el flujo del campo será el gasto hidrodinámico, es decir, el volumen de líquido que atraviesa la superficie 
esférica  por  segundo.  Pero  el  mismo líquido que atraviesa  una  esfera  de  radio cualquiera,  digamos, “a”, 
pasará luego por las esferas de mayor radio. Ello, claro está, si no existen otras cargas eléctricas dentro de 
estas esferas, donde se genere “fluido” adicional. De esta consideración podemos intuir que la superficie 
no  necesita  ser  esférica,  y  que  la  cantidad  de  líquido  que  pasa  por  cualquier  superficie  cerrada  que 
contenga a la carga (punto fuente del fluido) será siempre la misma. 
 
3.6. FLUJO DEL CAMPO ELÉCTRICO DE UNA CARGA PUNTUAL A TRAVÉS DE UNA SUPERFICIE ARBITRARIA S. 
  El resultado representado por (5a) y (5b) se puede generalizar al caso en que la superficie  E no es 
una esfera sino una superficie S cerrada arbitraria, y la carga Q se encuentra en un punto arbitrario dentro 
o fuera del espacio limitado por esta superficie S.  
 
  El flujo del campo eléctrico de una carga puntual Q a través de una superficie arbitraria S  es: 
 
Q
(6a)    ΦE ( S ) =      si Q está en el espacio dentro de la superficie S (Fig. 11) 
εo
 

(6b)    ΦE(S) = 0    si Q está fuera del espacio interior limitado por la superficie S (Fig. 12) 


 
 

 
   
 
 
Fig. 11. Flujo = Q/ε0  Fig. 12. Flujo = 0 
 
3‐9 

3.7. LEY DE GAUSS PARA UN CONJUNTO DE CARGAS PUNTUALES. 
  La ley de Gauss es una extensión del resultado obtenido en el apartado precedente.  
 

 
 
Fig. 13 
 
  Sea Q1, Q2, Q3, …., QN un conjunto de cargas puntuales y S  una superficie arbitraria. Numeremos 
las cargas de modo que las primeras k de ellas queden fuera de la superficie, y las restantes dentro 
(Fig. 13). Sea Ei el campo eléctrico producido por la carga Qi. 
  El flujo del campo eléctrico total E a través de S es 
 
    ∫ ∫ ∫ ∫ ∫
Φ E ( S ) = E • da = (E1 + E2 + ... + E N ) • da = E1 • da + E 2 • da + ... + E N • da  
S S S S S
 
es decir, igual a la suma de los flujos individuales producidos por todas las cargas. 
  Ahora  bien,  por  (6a,b),  las  primeras  k  cargas  no  producen  flujo  puesto  que  están  fuera  de  S  ;  en 
cuanto a las demás, de la k+1 hasta la N, el flujo de cada una es igual a cada carga dividida entre ε0, de tal 
modo que 
 
Q k +1 Q k + 2 Q Q
    ∫
Φ E ( S ) = E • da =
ε0
+
ε0
+ ... + N = in  
ε0 ε0
S
 
Hemos puesto Qin ≡ carga total encerrada dentro de S . Este resultado es la ley de Gauss para un conjunto 
de cargas puntuales, que formularemos a continuación para una distribución de carga arbitraria. 
3‐10 

3.8. LEY DE GAUSS EN GENERAL, VÁLIDA PARA CUALQUIER DISTRIBUCIÓN DE CARGA. 
 
LEY DE GAUSS 
Sea  D  una  distribución  de  carga  arbitraria,  y  E  su  campo    eléctrico.  Sea  S   una  superficie  escogida 
arbitrariamente y Qin la carga eléctrica que queda incluída dentro de S. 
Entonces el flujo del campo eléctrico E  a través   
de S viene dado por la expresión 
 
 
Qin
(7)    ΦE (S ) =  
ε0
 
 
 
Fig. 14
 
  La superficie arbitraria S  a la que hace referencia la ley de Gauss se denomina superficie gaussiana. 
En las aplicaciones de la ley de Gauss podemos escoger esta superficie según convenga. 
  Consideremos estas dos situaciones: 
  En la primera (Fig. 15), la superficie gaussiana no capta ninguna carga de la distribución D, es decir, 
Qin = 0. De acuerdo con la ley de Gauss, el flujo de E a través de S  es entonces cero. 
   

   
   
Fig. 15  Fig. 16 
 

En la otra situación (Fig. 16), la superficie gaussiana encierra enteramente la distribución  D. Entonces  Qin 


es igual a toda la carga Q de la distribución, y el flujo es Q dividida por ε0. 
  Es  importante  advertir  que  en  la  ley  de  Gauss  figura  el  campo  E  producido  por  toda  la  carga 
eléctrica presente. Aunque las cargas que quedan fuera de la superficie gaussiana no contribuyan al flujo a 
través de ella, sí contribuyen al campo total E. 
 

 
Fig. 17 
3‐11 

  Examinemos ahora el caso representado en la Fig. 17. La superficie gaussiana, delineada a trazos, 
consta de dos superficies  S  1 y  S  2, que encierran un espacio central hueco. En este espacio se encuentra 
cierta distribución de carga, de carga total Q que supondremos positiva por concreción. 
  La región “U” limitada por ambas superficies, o sea por  S1 ∪ S2  , que excluye el espacio hueco, no 
contiene carga, de donde por la ley de Gauss el flujo del campo de  Q a través de  S1 ∪ S2  es cero. Ello lo 
podemos ver también como sigue. 
  Existe  un  flujo  que  entra  en  U  a  través  de  S1;  este  flujo  es  negativo  porque  E  y  da1  tienden  a 
oponerse  sobre  esta  superficie. Por  otra  parte,  hay  otro  flujo que  sale  de  U   a  través de  S2; este  flujo es 
positivo. El flujo neto a través de U  es cero, como ya dijimos. 
  Debido  a  la  convención  de  signo  para  el  vector  área  de  las  superficies,  un  flujo  entrante  en  una 
región espacial es negativo. Un flujo saliente es positivo. 
 

 
 
Fig. 18 
 
  Si  una  carga  puntual  Q  no  se  halla  “ni  dentro  ni  fuera”  de  la  superficie  gaussiana,  sino 
precisamente sobre ella (ver la Fig. 18), el flujo del campo eléctrico de Q a través de la superficie es Q/2εo, 
o sea la mitad del que sería si Q estuviese dentro. No demostraremos este resultado. 
 
3.9. CONDUCTORES Y LEY DE GAUSS. 
  Los  átomos  de  un  material  conductor  poseen  en  su  capa  externa  uno  o  varios  electrones  muy 
débilmente ligados al núcleo. Bajo la influencia de campos eléctricos, incluso muy débiles, estos electrones 
pueden  vagar  por  todo  el  conductor,  constituyendo  una  corriente  eléctrica  durante  todo  el  tiempo  que 
exista un campo eléctrico dentro del conductor. 
  Por  ejemplo,  el  átomo  de  cobre,  un  conductor  excelente,  posee  un  número  atómico  de  29.  En  sus 
primeras  tres  capas  electrónicas  posee  2,  8  y  18  electrones,  en  total  28  electrones.  El  electrón  número  29 
ocupa la última capa y constituye un “electrón libre”. Así, cada átomo de cobre aporta 1 electrón libre. 
  Cada átomo de aluminio, cuyo número atómico es 13, aporta 3 electrones libres. 
  Cualquier carga en exceso contenida en un material conductor reside en su superficie. Imagine que 
se tiene un trozo de, digamos, cobre, y se le inyecta en su región central cierta carga negativa en exceso. 
Inmediatamente los electrones en exceso se distancian unos de otros bajo la acción de sus propios campos 
eléctricos,  yéndose  hasta  la  superficie  del  trozo  (Fig.  19).  Ya  en  la  superficie,  los  electrones  dejan  de 
moverse, formándose una situación electrostática. Los electrones no salen de la superficie hacia el espacio 
exterior al trozo (tal movimiento induciría una carga positiva en el trozo que atraería al electrón de nuevo 
hacia la superficie). 
3‐12 

 
 
   

 
 
 
 
Fig. 19  Fig. 20 
 
  En la situación electrostática mencionada, el campo eléctrico es nulo en todo punto del interior del 
conductor. Esto lo podemos ver como sigue. 
  Tomemos  una  superficie  gaussiana  enteramente  dentro  del  conductor  (Fig.  20).  Dado  que  esta 
superficie no contiene carga (Qin = 0), la Ley de Gauss implica que el flujo del campo eléctrico a través de 
ella  es  igual  a  cero.  Si  modificamos  la  superficie  gaussiana,  manteniéndose  la  misma  completamente 
dentro del conductor, la carga Qin sigue siendo cero y el flujo del campo eléctrico también. Esto solamente 
puede suceder si el campo eléctrico es cero idénticamente dentro del conductor. 
3‐13 

3.10. APLICACIÓN DE LA LEY DE GAUSS AL CÁLCULO DE CAMPOS ELÉCTRICOS CON SIMETRÍA ESFÉRICA. 
  La  ley  de  Gauss  es  una  de  las  leyes  fundamentales  de  la  electrostática.  Entre  sus  múltiples 
aplicaciones se cuenta la del cálculo de campos eléctricos, tema que discutiremos en este apartado. 
  No  cualquier  campo  eléctrico  puede  calcularse  con  la  ley  de  Gauss.  Debe  tratarse  de  campos  con 
simetrías especiales. Presentémoslos. 
 
Campos con simetría esférica. 
  Un campo eléctrico posee simetría esférica si la carga eléctrica que lo genera posee una densidad de 
carga que depende solamente de la distancia “r” a un punto central. 
En  este  caso  las  líneas  de  fuerza  son  líneas  radiales  que  emanan  de  tal  punto  central  hacia  todas 
direcciones. Además, la magnitud del campo depende solamente de la distancia “r” a tal punto central y 
por tanto es la misma para todos los puntos con la misma “r”. 
Ya conocemos un caso: el campo de una carga puntual posee simetría esférica. 
  La  Fig.  21  muestra  algunas  distribuciones  de  carga  cuyos  campos  poseen  simetría  esférica  (Se 
supone  aquí  que  las  densidades  de  carga  son  constantes).  El  campo  eléctrico  de  tales  distribuciones  de 
carga posee líneas de fuerza radiales. 
 
   

 
   
 
Esfera sólida 
Cascarón esférico (delgado) 
   

   
   
Esfera sólida dentro de esfera  Carga puntual central, esfera 
hueca concéntrica  sólida y cascarón 
Fig. 21 
 
  Veamos  a  continuación  cómo  es  que  la  ley  de  Gauss  nos  facilita  calcular  el  campo  radial  de  esta 
clase de distribuciones de carga. 
3‐14 

 
  Sea  C  el  centro  de  simetría  de  la  distribución  (Fig.  22).  Con  centro  en  C,  tracemos  una  superficie 
gaussiana  esférica  de  radio  arbitrario  “r”.  Según  la  ley  de  Gauss,  el  flujo  de  E  a  través  de  tal  superficie 
gaussiana S es 
 
Qin
    ∫
Φ E ( S ) = E • da =
ε0
 
S
 

El cálculo de la integral resulta trivial debido a estas dos propiedades: 
 

  − E es paralelo a da en todo punto de S
  − La magnitud de E es la misma en todo punto de S 
 

 
Fig. 22 
 
  He aquí el desarrollo de la integral, a la luz de estas propiedades: 
 
      ∫ E • da = ∫ Eda ∫
= E da = E A ⊥  
S S S
 
 
Porque E es 
  Porque E y da 
constante 
  son paralelos 
 sobre S 
  sobre S 
 
Hemos puesto   
 

    ∫ da = A⊥ = “Área atravesada perpendicularmente por el campo” 
S
 

En este caso de simetría esférica,  A ⊥ es el área de la esfera de radio “r”, o sea 4πr2. 
  Igualando el valor del flujo, E A ⊥ , a lo que nos dice la ley de Gauss tenemos 
 
Q in
    EA ⊥ =  
ε0
 

  Despejemos el campo E y enunciemos el resultado en estos términos: 
3‐15 

 
Si se tiene que 
 

• El campo eléctrico E es paralelo a da sobre la parte de la superficie gaussiana S donde existe flujo. 
 

• La magnitud del campo, E, es constante sobre la parte de la superficie gaussiana donde existe flujo. 
 
entonces la magnitud del campo sobre S viene dada por 
 
Qin
(8)    E=  
ε0 A ⊥
 
donde  Qin es  la  carga  neta  contenida  dentro  de la superficie  gaussiana, y  A ⊥   es el área atravesada 
perpendicularmente por el campo. 
 
 
  Como veremos, la misma fórmula es aplicable a las otras dos simetrías. 
 
EJEMPLO 3.3.  Calcular el campo eléctrico en todo el espacio, generado por una esfera sólida de radio R y 
carga total Q distribuída uniformemente (Fig. 23). Usar la ley de Gauss para el cálculo. 
 

 
 
Fig. 23 
 
Procedimiento: 
(I)  La distancia de un punto espacial arbitrario al centro de la esfera se denotará con “r”. Lo primero es 
dividir el espacio en regiones, delimitadas por las discontinuidades de la carga. En el ejemplo presente, 
distinguimos dos regiones: 
 
    Región 1:    0 < r < R 
 

    Región 2:    r > R 
 
(II)  Para  calcular  el  campo  en  un  punto  P  cualquiera  de  la  Región  1  (dentro  de  la  esfera),  debemos  trazar  una 
superficie  gaussiana  que  pase  por  ese  punto  y  tenga  la  misma  simetría  que  la  distribución  de  carga.  Obviamente 
será una esfera concéntrica a la región cargada; el radio de la superficie gaussiana es “r” (donde  0 < r < R). Vea la 
Fig. 24. 
 
 
 
 
3‐16 

 
 
   

 
   
 
 
 
Fig. 24   (Región 1) 
Fig. 25   (Región 2) 
 
(III)  Para calcular el campo E en un punto P de la Región 1 usamos la fórmula (19), 
 
Qin
    E=  
ε0 A ⊥
 
de tal manera que el cálculo del campo se reduce al cálculo de las cantidades  Qin y  A ⊥ . Para campos 
con simetría esférica, tendremos siempre que  A ⊥ = 4πr2. 
En  cuanto  a  Qin,  notemos  que  la  superficie  gaussiana  encierra  una  fracción  de  la  carga  total  Q.  Para 
obtener Qin, calculemos primeramente la densidad de carga volúmica, 
 
Q Q
    ρ= =  
V 4
πR 3
3
 

Usando ahora la relación “Carga = Densidad x Volumen” tendremos para la carga dentro de la esfera 
de radio “r” la expresión 
 
4 Q 4 r3
    Qin = ρ ⋅ π r 3 = ⋅ π r3 = 3 Q  
3 4 3 3 R
πR
3
 

  Sustituyendo lo encontrado para  A ⊥  y Qin obtenemos para el campo E dentro de la esfera: 
 

r3
Q
Qin R 3 Q
    E= = = r    (Para 0 ≤ r ≤ R) 
ε0 A ⊥ ε0 ⋅ 4 πr 2
4 πε0 R 3
 
3‐17 

  Este campo es nulo en el centro de la esfera (r = 0) y va aumentando linealmente con la distancia 
“r”. La expresión es válida para 0 ≤ r ≤ R. 
  Repitamos  el  cálculo  para  la  Región  2.  Observe  la  Fig.  25,  donde  hemos  trazado  la  superficie 
gaussiana  apropiada.  El  área  A ⊥   sigue  siendo  4πr2,  pero  ahora  la  carga  que  encierra  la  superficie 
gaussiana es toda la carga de la distribución, o sea Q. Entonces, 
 
Qin Q Q
    E= = =     (Para r ≥ R) 
ε0 A ⊥ ε0 ⋅ 4πr 2
4 πε0 r 2
 
Este campo es idéntico al que produciría una carga puntual cuya carga fuese la misma que la de la esfera 
sólida, y que estuviese situada en el centro de la esfera. Esto es válido para puntos fuera de la esfera. 
  Notemos que ambas expresiones dan el mismo valor en r = R. 
  La Fig. 26 es un gráfico del campo E como función de “r”. 
 

 
 
Fig. 26 
 

EJEMPLO  3.4.  Dos  cascarones  esféricos  de  radios 


R1  y  R2  portan  sendas  cargas  eléctricas  Q1  y  Q2, 
distribuídas  uniformemente.  Calcular  el  campo 
eléctrico  generado  en  todo  el  espacio  por  esta 
distribución. 
‐‐‐‐‐‐‐‐‐‐‐‐‐‐‐‐‐‐‐‐‐‐‐‐‐‐‐‐‐‐‐‐ 
  Denotemos con “r” la distancia de los puntos 
del  espacio  al  centro  común  de  los  cascarones. 
Como  primer  paso,  dividiremos  el  espacio  en  las 
siguientes regiones:   
   
  Región 1:    0 ≤ r ≤R1  Fig. 27 
  Región 2:    R1 ≤ r ≤R2 
  Región 3:     r ≥ R2 
 
  Para calcular el campo eléctrico en cada una de estas regiones, debemos escoger un punto arbitrario 
“P” en cada una, trazar una superficie gaussiana esférica que pase por este punto, y aplicar la fórmula 
(8‐p14). 
  Las Figs. 28, 29 y 30 muestran las superficies gaussianas apropiadas para cada región. 
3‐18 

 
3‐19 

 
 

   
Fig. 28  Fig. 29 
Qin = 0  Qin = Q1 
 
   
  Note lo siguiente: 
• P  es  un  punto  arbitrario  de  la  región 
considerada  en  cada  caso.  Su  distancia  al 
centro se designa siempre con “r”. 
• La  superficie  gaussiana  que  pasa  por  P  es 
una esfera (trazos a rayas en las figuras). 
• En  cada  figura  se  muestra  cuál  es  la  carga 
contenida por la superficie gaussiana. 
  Dado  que  el  campo  es  radial,  el  área  de  la 
superficie  gaussiana  atravesada  perpendicular‐
mente  por  el  campo  es  el  área  de  toda  esta   
Fig. 30 
superficie, es decir,  A ⊥ = 4 πr . 
2
Qin = Q1 + Q2 
  Tenemos entonces, según la fórmula (8‐p4): 
   
  En la región 1, Qin = 0 y 
 
Qin 0
    E= = =0 
ε0 A ⊥ ε0 ⋅ 4πr 2
 

  En la región 2, Qin = Q1 y 
 
Qin Q1
    E= =  
ε0 A ⊥ ε0 ⋅ 4πr 2
 

  En la región 3, Qin = Q1 + Q2 y 
 
Qin Q + Q2
    E= = 1 =0 
ε0 A ⊥ ε0 ⋅ 4πr 2
 

Así pues, no existe campo eléctrico en la región más interna (libre de carga), y en las otras dos regiones el 
campo se asemeja al de una carga puntual (Q1 en un caso, Q1 + Q2 en el otro). 
3‐20 

 
3.11. APLICACIÓN DE LA LEY DE GAUSS AL CÁLCULO DE CAMPOS ELÉCTRICOS CON SIMETRÍA CILÍNDRICA. 
Campos con simetría cilíndrica 
  Una distribución de carga posee simetría cilíndrica si: 
• La densidad de carga de la distribución depende solamente de la distancia “r” a un eje. 
• La distribución se extiende longitudinalmente hasta el infinito. 
  En la Fig. 31 se muestran tres ejemplos: un alambre recto (imagínelo extendido longitudinalmente 
hasta  infinito),  cargado  uniformemente  con  densidad  lineal  λ;  un  cilindro  sólido  infinito  de  densidad 
volúmica ρ; y un cascarón cilíndrico delgado cuya carga se describe mediante una densidad superficial σ. 
o  mediante  una  densidad  lineal  λ.  Mediante  combinaciones  de  estas  distribuciones  se  pueden  formar 
otras distribuciones coaxiales más complejas con la misma simetría. 
   

 
   
   
Fig. 31  Fig. 32 
 
  El  campo  eléctrico  de  tales  distribuciones  posee  líneas  de  fuerza  ortoaxiales,  es  decir, 
perpendiculares al eje de simetría de la distribución. 
  La  superficie  gaussiana  que  se  usa  para  calcular  el  campo  tiene  la  forma  de  un  cilindro  cuya 
superficie lateral debe contener al punto campo considerado; la altura de este cilindro se fija en un valor 
arbitrario “h” (que, como veremos, se cancelará y no aparecerá en la expresión del campo); el radio de este 
cilindro gaussiano se denotará con “r”. 
Imagine que el cilindro de trazos a rayas de la Fig. 32 es una superficie gaussiana. Está claro que no 
hay flujo del campo a través de las tapas del cilindro. Solamente la superficie lateral de este cilindro es la 
que el campo atraviesa perpendicularmente. En esta porción de la superficie gaussiana podemos verificar 
Qin
que se cumplen las condiciones de validez de la fórmula   E =  a saber: el campo E es paralelo a da, 
ε0 A ⊥
y la magnitud de E es constante. Tendremos que 
 
    A ⊥  = área de la superficie lateral del cilindro = 2πr h 
 
  Las  expresiones  que  se  obtendrán,  válidas  para  distribuciones  de  extensión  infinita,  se  pueden 
aplicar a casos reales en que la altura de la distribución de carga es mucho mayor que su radio, y el punto 
campo considerado está relativamente cerca de la parte central de la distribución. 
3‐21 

 
EJEMPLO  3.5.  Calcular  el  campo  eléctrico  producido  por  una  distribución  que  consta  de  un  cilindro 
sólido  central  de  radio  R1,  cuya  densidad  volúmica  uniforme  es  ρ,  y  un  cascarón  cilíndrico  coaxial  de 
radio R2, cargado con densidad superficial uniforme σ. 
  Distinguimos tres regiones: 
 

    Región 1:   0 < r < R1  
    Región 2:   R1 < r < R2 
    Región 3:   r > R2 
 

que  corresponden  a  ubicaciones  dentro  del  cilindro  sólido,  en  el  espacio  vacío  entre  ambos  cilindros,  y 
completamente fuera de la distribución, respectivamente. 
   
 

 
   
   

Fig. 33.  Región 1  Fig. 34.   Región 2 
   
He aquí las expresiones de Qin: 
 
Región 1:  Qin = ρ π r2 h 
 
Región 2:  Qin = ρ π R12 h 
 
Región 3:  Qin = ρ πR12 h + σ 2πR2 h 
 
En las tres regiones se aplica 
 
 
      A ⊥ = 2 πrh  
Fig. 35.   Región 3   
 
 
Qin
Sustituyendo sucesivamente en la fórmula   E =   encontramos 
ε0 A ⊥
3‐22 

ρπr 2 h ρ
En la Región 1:    E= = ⋅r  
ε0 ⋅ 2 πrh 2ε0
 
ρπR 12 h ρR 12 1
En la Región 2:    E= = ⋅  
ε0 ⋅ 2 πrh 2ε0 r
 
ρπR 12 h + σ ⋅ 2 πR 2 h ⎛ ρR 12 + 2σR 2 ⎞ 1
En la Región 3:    E= = ⎜⎜ ⎟⎟ ⋅  
ε0 ⋅ 2 πrh ⎝ 2ε 0 ⎠ r
 
3.12. CÁLCULO DE CAMPOS CON SIMETRÍA PLANA. 
 
   
EJEMPLO  3.6.  Usando  la  Ley  de  Gauss, 
calcular  el  campo  eléctrico  en  todo  el 
espacio,  generado  por  una  placa  plana 
infinita, de grosor w y densidad uniforme ρ   
(Fig. 36).   
Fig. 36 
   
 
  La  densidad  de  carga  posee  simetría  plana.  Las  líneas  de  fuerza  del  campo  eléctrico  son  rectas 
perpendiculares a la placa y se dirigen alejándose de la misma (ver la Fig. 37).  
  Definamos el plano XY coincidente con el plano de simetría de la placa, y el eje Z perpendicular a 
ésta. Deseamos averiguar cómo depende el campo E de la coordenada “z”. 
 

 
 
Fig. 37 
 

  Bastará con obtener el campo en estas dos regiones: 
 
w w
    Región 1:  0 < z <         Región 2:  z >    
2 2
 
3‐23 

 
 
Fig. 38 
 
  En la Región 1 escogemos una superficie gaussiana con forma de prisma rectangular de altura  2z y 
tapas de área  A. Debemos disponer este prisma simétricamente con respecto a la placa con objeto de que 
el campo  E sea el mismo en ambas tapas. El área que el campo atraviesa perpendicularmente es el área 
conjunta de las dos tapas, 
 
    A⊥ = 2A 
 
Por otra parte, la carga incluída es 
 
    Qin = (densidad) x (volumen) = ρ 2z A 
 
con lo que en la Región 1 tendremos 
 
Qin ρ ⋅ 2zA ρ
    E= = = ⋅z  
ε0 A ⊥ ε0 ⋅ 2A ε0
 
  En la Región 2 escogemos el prisma con sus tapas fuera de la placa (véase la Fig. 38). Tendremos 
que Qin = ρ A w. El campo es 
   
Q ρ ⋅ wA ρw
    E = in = =  
ε0 A ⊥ ε0 ⋅ 2A ε0
 
El campo es constante dentro de la placa. 
  La  Fig.  39  muestra  el  gráfico  del  campo 
eléctrico como función de “z”.   
 
Fig. 39 
3‐24 

3.13. PROBLEMAS. 
  Usando la ley de Gauss, calcular los campos indicados. Suponer distribuciones uniformes de carga 
en todos los casos. 
 
1.  Esfera  hueca  de  carga  total  Q,  radio   
interior R1 y radio exterior R2., junto con una 
carga puntual “q” en su centro. 
Resp. Las cargas Qin son 
 
3 3  
r − R1
  q;  q + Q ; q + Q   
3 3
R 2 − R1
Fig. Probl. 1 
 
 
2.  Sistema  de  tres  hojas  planas  infinitas,   
con  las  densidades  superficiales  de 
carga mostradas. 
Sugerencia.  Suponer  campos  E1  y  –E1 
arriba de la placa superior y debajo de la 
inferior,  respectivamente.  Suponer 
campos  E2  y  –E2  en  las  otras  dos   
 
regiones. 
Fig. Probl. 2 
σ
Resp. 0;   
εo
 
3.  Cilindro  sólido  infinito,  de  radio  R  y 
densidad volúmica de carga ρ. 
Resp. 
 
ρ r ρR 2
;  
2ε0 2 ε0 r
 

 
Fig. Probl. 3 
4.  Un  “tubo  de  fuerza”  es  una   
superficie  formada  lateralmente  por 
líneas  de  fuerza,  y  provista  de  dos 
“tapas”.  Suponer  que  no  existe  carga 
dentro del tubo y demostrar con la ley 
de Gauss que el flujo entrante por una 
tapa es igual al saliente por la otra. 

 
Fig. Probl. 4 
3‐25 

5. Un capacitor cilíndrico consta de dos cascarones cilíndricos de radios “a” y “b” y densidades lineales  λ 
y −λ. Calcular el campo eléctrico dentro y fuera del capacitor. 
λ
Resp.  0;  
2πε or

 
Fig. Probl. 5 
 
 
6.  Usando  la  ley  de  Gauss,  calcular  el  campo   
eléctrico  en  todo  el  espacio,  generado  por  la 
siguiente distribución de carga: 
– Una carga puntual Q 
–  Un  cascarón  esférico  delgado  de  radio  R  y 
densidad  de  carga  superficial  constante  σ,  cuyo 
centro es la carga Q. 
Resp. 
 
 
 
Fig. Probl. 6 
 
7.  Usando  la  ley  de  Gauss,  calcular  el  campo 
eléctrico  en  todo  el  espacio,  generado  por  la 
siguiente distribución de carga: 
–  Una  esfera  sólida  de  radio  “a”,  con  densidad 
volúmica de carga constante “r”. 
–  Un  cascarón  esférico  delgado  de  radio  “b” 
concéntrico con la esfera, y cargado uniformemente 
con carga total “– Q”. 
Resp. 
 
 
 
Fig. Probl. 7 
 
 
 
4‐1 

  CAPÍTULO 4 
   
  POTENCIAL ELÉCTRICO, ENERGÍA Y VOLTAJE 
 
 
 
4.1. INTRODUCCIÓN. 
  Como  ya  hemos  visto,  las  influencias  eléctricas  de  las  cargas  se  pueden  describir  mediante  sus 
campos eléctricos (vectores de intensidad eléctrica “E”). Existe otra manera, en la que estas influencias se 
describen  mediante  una  función  escalar  de  la  posición  denominada  el  potencial  eléctrico,  denotado 
usualmente  mediante  el  símbolo  “φ”.  Por  ser  φ  una  función  (o  campo)  escalar,  esta  descripción  es  más 
simple que la que se vale de los campos eléctricos, puesto que estos últimos son cantidades vectoriales. Más 
aun,  el  potencial  eléctrico  es  una  cantidad  que  guarda  una  relación  muy  estrecha  con  los  conceptos  de 
energía  eléctrica  y  voltaje  (o  tensión)  eléctrico,  de  gran  importancia  en  las  aplicaciones  del 
electromagnetismo.  
  El potencial eléctrico se define en términos de un trabajo, o equivalentemente en términos de una 
energía  potencial.  Recomendamos  al  lector  refrescar  sus  conocimientos  sobre  los  conceptos  de  trabajo  y 
energía potencial en su texto de mecánica. 
 

4.2. POTENCIAL ELÉCTRICO DEL CAPACITOR DE PLACAS PARALELAS. 
  En esta sección discutiremos el concepto de potencial eléctrico para el caso simple de un capacitor 
de placas paralelas cuyo campo eléctrico es, como sabemos, constante. 
  Consideremos pues un sistema eléctrico formado por un capacitor de placas paralelas, “C ”, y una 
carga puntual positiva “q” situada entre las placas. Denotemos este sistema con el símbolo “{C, q}”. 
   Sea  “d”  la  distancia  entre  las  placas  del  capacitor,  “A”  el  área  de  cada  placa,  “Q”  la  carga  del 
capacitor, y “σ” su densidad de carga superficial (Fig. 1). 
 

 
Fig. 1 
 

Coloquemos un Eje Y con su origen  O  en la placa negativa y dirigido hacia la placa positiva. Sea “y” la 
coordenada vertical de la carga puntual en un punto arbitrario del espacio dentro de las placas. 
  Vamos a realizar el siguiente proceso: desde la placa negativa (y = 0) transportaremos la carga “q” 
hacia  la  placa  positiva,  aplicándole  una  fuerza  F  (hacia  arriba)  igual  y  opuesta  en  todo  momento  a  la 
fuerza “qE” que el campo eléctrico del capacitor ejerce (hacia abajo) sobre la carga. En otras palabras, el 
transporte  se  realizará  cuasiestáticamente:  en  todo  punto  la  carga  q  estará  prácticamente  en  equilibrio 
dinámico (moviéndose con una velocidad constante insignificante). 
4‐2 

  Llamaremos  “sistema  externo  {Sext}”  al  agente  que  aplica  la  fuerza  F  (la  naturaleza  física  de  este 
agente  no  es  relevante  aquí).  Así  pues,  en  este  proceso  tendremos  implicados  dos  sistemas:  el  sistema 
capacitor‐carga,  {C, q}, y el sistema externo {Sext}. 
  Al desplazarse la carga  q una distancia “y”, la fuerza  F (o sea el sistema externo) realiza sobre ella 
un trabajo positivo igual a: 
 
(1)    Wext = F y      (Trabajo = Fuerza × Distancia recorrida) 
 
Como  estamos  suponiendo  que  en  todo  momento  F  es  igual  al  negativo  de  la  fuerza  eléctrica  “qE”,  
tendremos que el trabajo se puede escribir también 
 
(2)    Wext = qE y         (ya que F = |F| = |– qE| = qE) 
 
  Como vemos en (2), este trabajo es una función de la posición “y”. 
  Con base en (2), vamos ahora a definir una cantidad que se refiere exclusivamente al capacitor (o a 
su  campo  eléctrico).  Obtenemos  esta  cantidad  simplemente  dividiendo  (2)  por  “q”,  o  sea,  obteniendo  el 
trabajo externo por unidad de carga: 
 
Wext
    = Ey  
q
 

Esta cantidad es por definición el potencial eléctrico “del capacitor” (o “asociado con el campo eléctrico 
del capacitor”) en el punto arbitrario “y”. Se denota usualmente con la letra griega “φ”. 
 
  El potencial eléctrico asociado con el campo eléctrico de un capacitor de 
 
placas paralelas, en un punto a una distancia “y” de la placa negativa, 
Wext
(3)  φ(y) = = Ey   φ(y),  es  el  trabajo  por  unidad  de  carga  que  debe  realizar  un  agente 
q
externo para transportar cuasiestáticamente una carga puntual positiva 
desde  la  placa  negativa  hasta  la  posición  “y”,  manteniéndose  el 
capacitor fijo y su carga invariable durante este proceso.. 
 
Recuerde:  “cuasiestáticamente”  significa  que  el  agente  externo  ejerce  sobre  la  carga  q  una  fuerza  que 
cancela exactamente en todo punto la fuerza eléctrica debida al capacitor. La carga q llega al punto “y” sin 
haber adquirido energía cinética alguna. 
  Aunque la definición (3) vale para el campo de un capacitor, la definición general del potencial se 
expresa en forma muy similar, como veremos más adelante. 
  La definición (3) nos trae a la mente otra definición muy similar usada en mecánica, la de energía 
potencial. 
  Efectivamente,  si  nos  preguntamos  adónde  va  el  trabajo  realizado  por  el  agente  externo,  la 
respuesta es: “se convierte en energía potencial del sistema  {C, q}”. El proceso consiste entonces en una 
transferencia de energía, en forma de trabajo, desde el sistema externo  {Sext} hacia el sistema  {C, q}.  Esta 
clase de energía se denomina energía potencial eléctrica. 
4‐3 

  Denotando la energía potencial eléctrica con “U”, tendremos entonces 
 
  U = Wext = qEy 
 
  Pongamos esta importante relación en un cuadro: 
 
  Energía  potencial  eléctrica  del  sistema  {C,  q},  formado  por  un 
(4)    U = qE y  capacitor  de  placas  paralelas  y  una  carga  puntual  q,  en  la 
configuración  en  que  la  carga  está  a  una  distancia  “y”  de  la  placa 
negativa. 
 
  En virtud de la igualdad  Wext = U y la definición (3), se advierte que podemos dar una definición 
alternativa del potencial eléctrico del capacitor en los siguientes términos: 
 
  El  potencial  eléctrico  asociado  con  el  campo  eléctrico  de  un 
 
capacitor de placas paralelas, en un punto a una distancia “y” de la 
U
(5)    φ(y) = = Ey   placa negativa,  φ(y), es la energía potencial eléctrica por unidad de 
q
carga del sistema formado por el capacitor y la carga, cuando ésta se 
encuentra a una distancia “y” de la placa negativa. 
 
Observaciones 
• Como sabemos de la mecánica, a todo par de cuerpos físicos que interaccionan mediante una fuerza 
conservativa  le  podemos  asociar  una  energía  potencial.  Entre  las  fuerzas  conservativas  se  cuenta  la 
fuerza eléctrica, con la que interaccionan en el caso presente el capacitor y la carga puntual. La energía 
potencial U que discutimos arriba es una propiedad de ambos cuerpos, el capacitor y la carga. De todas 
maneras,  en el uso  común,  en lugar  de  la frase “la  energía  potencial  eléctrica del  sistema  {C, q}”  se 
suelen usar también las frases “la energía potencial eléctrica de la carga en el campo del capacitor” o 
simplemente “la energía potencial de la carga”, con las reservas del caso. 
• Las  unidades  físicas  del  potencial  eléctrico  en  el  Sistema  Internacional  de  Unidades  se  sacan  de  (5) 
como el cociente de unidades de energía y de carga, es decir, 
 

julio J
    = ≡ voltio = V  
coulombio C
 
• Notemos  que  el  potencial eléctrico  del capacitor, “φ = E y”,  es  una  función de la  posición  “y”. No  es 
necesario que exista una carga puntual dentro del capacitor para que esta cantidad esté bien definida. 
• La relación (5), escrita en la forma 
 

(6)    U = q φ 
 

puede leerse como sigue: cuando una carga puntual  q se encuentra en un punto donde el potencial 
eléctrico vale “φ”, la energía potencial de la carga en el campo del capacitor vale “qφ”. 
  De la expresión del potencial eléctrico de un capacitor,  φ = Ey, sacamos que todos los puntos de la 
placa negativa, donde y = 0, están a potencial cero. Por otra parte, los puntos de la placa superior positiva, 
donde y = d, se encuentran a potencial φ(d) = Ed. 
4‐4 

 
4.3. VOLTAJE O TENSIÓN ELÉCTRICA A TRAVÉS DEL CAPACITOR. 
  La diferencia de potencial entre las placas, se define como el voltaje o la tensión eléctrica “V” entre 
(o a través de) las placas. Tenemos así 
 

    V = φ(d) – φ(0) = Ed 
 

  Voltaje  o  tensión  eléctrica  entre  (o  a  través  de)  las  placas  de  un 
(7)    V = E d  capacitor de placas paralelas. E = |E| es el campo del capacitor y “d” es 
la distancia entre sus placas. 
 
  Nota.  Estamos  usando  el  mismo  símbolo  “V”  para  denotar  tanto  la  unidad  S.I.  de  voltaje  (voltio) 
como el voltaje mismo. Es fácil determinar del contexto cuál es el significado de “V” en cada caso. 
  El punto (o puntos) donde el potencial vale cero se denomina punto de referencia del potencial. En 
el  caso  del  capacitor,  el  punto  de  referencia  es  cualquier  punto  de  la  placa  negativa.  En  algunas 
situaciones, al punto de referencia del potencial eléctrico se le denomina “tierra”. 
  La relación (7) nos proporciona unas unidades S.I. alternativas para el campo eléctrico, a saber 
 
voltio V
    {Unidades S.I. del campo eléctrico} =  =  
metro m
 

De ahora en adelante usaremos estas unidades para el campo eléctrico. 
 
EJEMPLO 4.1. Para un capacitor de placas paralelas se tienen los siguientes datos: 
 
Carga    Q = 5 (10 – 8) C 
Área de placa    A = 200 cm2 
Distancia entre placas    d = 0.2 cm 
 
Calcular el voltaje entre las placas del capacitor. 
‐‐‐‐‐‐‐‐‐‐‐‐‐‐‐‐‐‐‐‐ 
  Dada la carga Q y el área A del capacitor, calculamos su densidad de carga: 
 
Q 5(10 −8 )C C
    σ= = = 2.5(10 −6 )  
A 200(10 )m − 4 2 m2
 
Seguidamente  podemos  obtener  el  campo  eléctrico  del  capacitor.  Poniendo  valores  expresados  en 
unidades S.I. obtenemos 
 

σ 2.5(10 −6 ) ⎛ V ⎞ 
    E= = = 2.82(10 5 ) ⎜ ⎟
ε0 8.854(10 −12 ) ⎝m⎠
 

Usando (7‐p4), el voltaje a través del capacitor es, en unidades S.I., 
 
    V = E d = 2.82 (105) ⋅ 0.2 (10–2) = 0.564 (103) = 564   (V) 
‐‐‐‐‐‐‐‐‐‐‐‐‐‐‐‐‐‐‐‐‐‐‐‐‐‐‐‐‐‐‐‐‐‐‐‐‐ 
4‐5 

 
4.4. MOVIMIENTO DE CARGAS ELÉCTRICAS DENTRO DEL CAMPO DE UN CAPACITOR DE PLACAS PARALELAS. 
  Consideremos una partícula de masa  m y carga eléctrica  q que se mueve dentro del campo de un 
capacitor de placas paralelas. 
 

 
Fig. 2 
 
  Supongamos  que  la  partícula  entra  por  la  parte  izquierda  en  el  tiempo  t  =  0,  con  coordenada 
horizontal  x0, coordenada vertical “y0” y velocidad “v0”, como se muestra en la Fig. 2. Ignoremos el peso 
de  la  partícula,  suponiéndolo  mucho  menor  que  la  fuerza  eléctrica.  El  campo  eléctrico  es  vertical  hacia 
abajo, así que la fuerza eléctrica sobre la carga q es puramente vertical y su componente Y es 
 
(8)    Fy = – qE 
 
Como esta fuerza es constante, la aceleración de la partícula será también constante y enteramente en la 
dirección “–Y”: 
 
Fy qE
(9)    ay = =−  
m m
 
No  hay  aceleración  a  lo  largo  del  Eje  X.  La  partícula  ejecuta  un  movimiento  de  tiro  parabólico,  cuyas 
ecuaciones son 
 
x = x0 + v0x ⋅ t
(10)    1 qE 2  
y = y0 + v0y ⋅ t + a y t 2 = y0 + v0y ⋅ t − t
2 2m
 
Derivando estas ecuaciones con respecto al tiempo obtenemos las velocidades vx y vy. 
  Si colocamos el origen O del sistema de coordenadas X,Y en el punto donde la carga entra al campo 
del capacitor, entonces la ecuación de la trayectoria es una parábola con ecuación: 
 

a y x2
(11)    y = (tan θ) ⋅ x −  
2v 02 cos 2 θ
 

donde “θ” es el ángulo que forma el vector velocidad v0 con el Eje X en el origen. 
4‐6 

 
  Notemos la similaridad de este movimiento con el movimiento de una masa puntual en el campo 
gravitatorio cerca de la tierra (el llamado “tiro parabólico”). En esta clase de movimiento la energía de la 
masa puntual se conserva, y es igual a 
 
1
    E =  mv 2 + mgy  
2
 
donde la coordenada “y” se mide desde el suelo hacia arriba. El término “mgy” es la energía potencial 
gravitatoria de la partícula. Existe una expresión similar en el caso capacitor‐carga que nos ocupa. 
  La energía cinética del sistema {Capacitor, Carga q} es puramente la de la carga q, pues el capacitor 
está fijo. Por otra parte, la energía potencial (eléctrica) de este sistema es, usando (4), U = qEy. Entonces la 
energía total es 
 
1
(12)    E =  mv 2 + qEy   Energía total de una carga puntual en el campo de 
2 un capacitor de placas paralelas. 
 
La  ley  de  conservación  de  la  energía  expresa  que  esta  cantidad  permanece  constante  durante  el 
movimiento de la carga. 
 
EJEMPLO 4.2. Se tiene un capacitor de placas paralelas cuyo voltaje entre placas es V = 600 voltios. Desde 
la placa positiva se deja ir del reposo una carga puntual  q = 8  μC, cuya masa es  m = 2.5 (10–4) kg. ¿Con 
qué velocidad impactará la placa negativa? Resolver con el método de conservación de la energía. 
‐‐‐‐‐‐‐‐‐‐‐‐‐‐‐‐‐‐‐‐‐‐‐‐‐‐‐‐‐‐‐‐‐‐‐‐‐‐‐‐‐‐‐‐ 
  Suponiendo placas horizontales, usaremos un sistema con el Eje X horizontal y el Eje Y vertical. El 
origen estará en un punto de la placa negativa. 
  Cuando la carga  q parte de la placa positiva  (y = d), la energía total del sistema capacitor‐carga es 
solamente potencial eléctrica y vale, por (4‐p3), 
 

    E (d) = U = qE d 
 

  Cuando  la  carga  llega  a  la  placa  negativa  (y  =  0),  la  energía  total  del  sistema  es  puramente  la 
energía cinética K adquirida por la carga, o sea 
 
1
    E (0) = K =  mv 2
2  
 

  Aplicando la ley de conservación de la energía total tenemos E (d) = E (0) o sea 
 

1 2qEd 2qV
    mv 2 = qEd ⇒ v= =  
2 m m
 
donde hemos usado la relación (7‐p4): V = E d.  Sustituyendo valores numéricos, 
 
2qV 2 ⋅ 8(10 −6 )C ⋅ 600V m
    v= = = 6.2  
m − 4
2.5(10 )kg s
4‐7 

 
EJEMPLO 4.3.  Se tiene un capacitor de placas paralelas cuadradas, de lado 5 cm. La distancia entre placas 
es  2 cm. Desde la orilla izquierda del capacitor, a media distancia entre las placas, se proyecta una carga 
puntual positiva q = 8 μC horizontalmente con una velocidad v0 = 100 m/s. la masa de esta carga es 

m  =  10–4  kg.  Calcular  la  densidad  de  carga  σ  del  capacitor,  sabiendo  que  la  carga  abandona  el  campo 
eléctrico a una distancia de  0.4 cm de la placa inferior. Suponer campo uniforme dentro del capacitor, y 
nulo fuera de él. 

 
Fig. 3 
‐‐‐‐‐‐‐‐‐‐‐‐‐‐‐‐‐‐‐‐‐‐‐‐‐‐‐‐‐‐‐‐‐‐ 
  Escojamos los Ejes X y Y como vemos en la Fig. 3. Sabemos que la carga q ejecuta un movimiento de 
tiro parabólico dentro del capacitor. Su aceleración en X es cero, y en Y es igual a 
 
Fy qE
    ay = =−  
m m
 

Desde que entra al capacitor, la carga empieza a moverse hacia abajo, según la ecuación de movimiento 
 

1 qE 2
    y = ay t2 = − t  
2 2m
 

  Aplicando esta ecuación al punto donde la carga abandona el capacitor tenemos  (en unidades S.I.) 
 

8(10 −6 )E 2
    −0.006 = − tv
2(10 −4 )  
 
donde  tv  es  el  tiempo  de  vuelo.  Podemos  calcular  este  tiempo  examinando  el  movimiento  en  X.  La 
velocidad en X es constante, así que  tv es el tiempo que invierte la carga en recorrer  5 cm con velocidad 
constante v0, o sea 
 
0.05m
    tv = = 5(10−4 )s  
100m / s
 

Sustituyendo este tiempo en la ecuación anterior y despejando el campo E obtenemos 
 

    E = 6 (105) V/m 
 

Usando ahora la relación E = σ/ε0 se encuentra 

    σ = ε0 E = 8.854 (10–12) ⋅ 6 (105) = 5.3 (10‐ 8)    (C/m2) 
4‐8 

 
4.5. DEFINICIÓN GENERAL DEL POTENCIAL ELÉCTRICO. 
  En esta sección generalizaremos la definición del potencial eléctrico que introdujimos anteriormente 
para el caso especial del capacitor de placas paralelas. 
  Sea D una distribución de carga arbitraria, y E el campo eléctrico que genera (Fig. 5). 
 

 
Fig. 5 
 
  El potencial eléctrico φ(P) producido por una distribución de carga dada D en el punto P, se 
  define  como  el  trabajo  por  unidad  de  carga  que  debe  realizar  un  agente  externo  para 
(13)  transportar una carga puntual positiva cuasiestáticamente (contra la fuerza del campo de  D) 
  desde  un  punto  de  referencia  P0  (escogido  arbitrariamente)  hasta  el  punto  considerado  P, 
manteniendose fija la distribución D durante tal desplazamiento. 
 
  Compare esta definición con la definición (3‐p2) para el caso del capacitor. 
 

Observaciones: 
• El agente externo al que hace referencia esta definición ejerce una fuerza  F que es en todo punto de  Γ 
igual y opuesta a la que ejerce el campo eléctrico de la distribución de carga dada, o sea F = –qE. 
• El trabajo realizado por este agente externo se puede calcular  a lo largo de cualquier trayectoria  o camino 
Γ que enlace el punto de referencia  P0 y el punto considerado  P. Esta libertad de elección del camino 
Γ  se  debe  a  que  la  fuerza  eléctrica  es  conservativa  (y  por  ende  F  también,  pues  es  el  negativo  de  la 
fuerza eléctrica), de tal manera que su trabajo no depende de la trayectoria. Dado que  P0 está fijo y 
que la trayectoria Γ es arbitraria, tendremos que el potencial no depende esencialmente ni de P0 ni de 
Γ, siendo una función exclusivamente de la posición P. 
• Denotando con  Wext(Γ; P0 → P) el trabajo del agente externo a lo largo de la trayectoria  Γ entre los 
puntos P0 y P, tenemos según la definición dada de potencial eléctrico la relación 
4‐9 

Wext ( Γ; P0 → P ) Wext
(14)    φ(P) ≡        (Abreviado:   φ = ) 
q q
 
• En  vista  de  la  igualdad  “F = –qE”,  el  trabajo  que  realiza  el  agente  externo  “F”  es  el  negativo  del 
trabajo que realiza el campo eléctrico “E”, de tal manera que la Ec. (14) es equivalente a esta otra: 
 
WEléc. ( Γ ; P0 → P) WEléc.
(15)    φ(P) ≡ −        (Abreviado:    φ = − ) 
q q
 
donde WEléc. es el trabajo realizado por el campo eléctrico. 
 

• El  punto  de  referencia  P0  que  figura  en  la  definición  (13)  es  un  punto  fijo  arbitrario  del  espacio. 
Escoger este punto equivale a escoger el “nivel cero” del potencial eléctrico φ, es decir, el punto donde 
el potencial vale cero por definición. Esto lo podemos ver de (14): 
 

Wext ( Γ; P0 → P0 )
    φ(P0 ) ≡ =0 
q
 

ya que el trabajo desde P0 hasta P0 es obviamente cero. 
 
• Usando la expresión general del trabajo de una fuerza F a lo largo de una trayectoria Γ, es decir, 
 
P
(16)    Wext = ∫
Γ P0
F • dr  
 

(r = vector de posición de un punto arbitrario de Γ. dr = desplazamiento infinitesimal a lo largo de Γ) 
y usando además la relación F = – qE en (16) tenemos 
 
P P P
    Wext ( Γ; P0 → P ) = ∫ F • dr = − Γ ∫P qE • dr = −q ⋅ Γ ∫P E • dr  
Γ P0 0 0

 
Sustituyendo esto en (14) obtenemos la siguiente expresión general para el potencial eléctrico: 
 
P
∫ E • dr  
Fórmula general para calcular el potencial eléctrico 
(17)    φ(P) = −
Γ P 0 φ, dado el campo eléctrico E. 
 
Note que esta expresión ya no contiene ninguna referencia al agente externo, que es solamente un artificio 
conveniente  para  enlazar  los  conceptos  de  potencial  eléctrico  y  energía  potencial  eléctrica.  El  potencial 
está determinado exclusivamente por el campo eléctrico E. 
• Supongamos que cambiamos el punto de referencia del potencial, digamos de P0 a otro punto fijo P0’. 
¿Qué efecto tiene este cambio sobre los valores del potencial? Veamos. 
Partamos de la expresión integral del potencial, referido al punto P0’, o sea, denotándolo con “φ’(P)”, 
 
P
    φ′(P) = − ∫P′ E • dr  
0
4‐10 

Usando la propiedad de aditividad de la integral de línea, podemos descomponer esto como 
 
⎛ P0 P ⎞ P0 P
    φ′(P) = − ⎜
⎝ ∫P′ E • dr + ∫P E • dr ⎟⎠ = −∫P′ E • dr − ∫P E • dr  
0 0 0 0

 
Pero la última integral en el miembro derecho es el potencial en P, referido al punto P0, con lo que 
 
P0
    φ′(P) = − ∫P′ E • dr + φ(P)  
0

 
La diferencia entre los valores del potencial referido a los puntos P0 y P0’ es por tanto la integral 
 
P0
    − ∫P′ E • dr  
0

 
Pero  esta  integral  tiene  un  valor  constante  independiente  del  punto  P  considerado,  puesto  que  los 
puntos de referencia P0 y P0’ son puntos fijos. 
Esto  significa  que  al  cambiar  el  punto  de  referencia  del  potencial,  cambian  también  los  valores  del 
potencial en todos los puntos del espacio, pero este cambio es el mismo en todos los puntos. Por lo 
tanto, las diferencias de potencial entre cualquier par de puntos no se ven afectadas. 
Esta situación es similar a la que ocurre en el campo gravitatorio constante de la Tierra con la energía 
potencial  gravitatoria:  el  punto  de  referencia  de  esta  energía  se  toma  usualmente  al  nivel  del  suelo 
(superficie  terrestre).  Si  se  cambia  la  altura  de  este  punto  de  referencia,  cambian  los  valores  de  la 
energía potencial gravitatoria en todos los puntos, pero las diferencias de tales energías no se alteran, 
y son estas diferencias las que entran en la expresión de la conservación de energía mecánica. 
• La integral (17) es una “integral de línea”. No discutiremos aquí los métodos existentes para evaluarla 
en  el  caso  general.  No  obstante,  para  los  campos  eléctricos  simétricos  que  consideramos 
principalmente en este curso, la integral de línea se puede reducir fácilmente a una integral ordinaria 
de una sola variable. Para ello es menester escoger una trayectoria de integración Γ conveniente, como 
veremos. 
  Análogamente  a  lo  que  sucede  con  el  sistema  capacitor‐carga  discutido  en  la  sección  anterior,  el 
trabajo  Wext  que  realiza  el  agente  externo  no  se  pierde,  sino  que  se  transforma  en  energía  potencial 
eléctrica “U” del sistema {D, q}. Usando entonces “Wext = U” y la relación (14), φ = Wext/q, tenemos. 
 
  Energía  potencial  eléctrica  del  sistema  {D,  q}  en  la 
(18)    U = q φ(P)  configuración en que la carga q se encuentra en el punto P. 
  D  es la distribución de carga que produce el potencial φ. 
 
  Antes de obtener algunos potenciales eléctricos importantes, introduciremos el concepto general de 
voltaje o tensión eléctrica entre dos puntos. 
 
4‐11 

 
4.6. VOLTAJE O TENSIÓN ELÉCTRICA. 
  En la página 4 definimos el voltaje a través de las placas de un capacitor de placas paralelas como la 
diferencia  de  potencial  existente  entre  dichas  placas.  La  definición  general  de  voltaje  es  completamente 
similar. 
  Sean  P1 y  P2 dos puntos de una región espacial donde existe un campo vectorial eléctrico  E y un 
campo escalar potencial eléctrico asociado φ. 
  La diferencia de potencial eléctrico entre P1 y P2 se denomina el voltaje o tensión entre (o “a través 
de”) ambos puntos. Lo designaremos con la letra V: 
 
   

(19)  Voltaje entre P1 y P2:  V ≡ φ(P2) − φ(P1)        (Unidades: voltio = V). 


 
 

De la expresión del potencial (Ec. (17)), 
 
P
    φ(P) = − ∫
Γ P0
E • dr  
 

se sigue que el voltaje entre P1 y P2 es 
 

P2 ⎛ P1 ⎞ P2 P0
(20)    φ(P2 ) − φ(P1 ) = − ∫ P0
E • dr − ⎜
⎝ P0

E • dr ⎟ = −
⎠ P0 ∫
E • dr −
P1 ∫
E • dr  
 

de tal manera que el voltaje entre P1 y P2 se puede escribir como una sola integral, a saber, 
 
P2
∫P
Diferencia  de  potencial  o  voltaje  entre  dos  puntos 
(20)    V(P1 → P2 ) = − E • dr    
1 P1 y P2. 
 

Notemos que el punto de referencia  P0 desaparece al hacer la diferencia (19), es decir, el voltaje entre dos 
puntos no depende de dónde se escoja el punto de referencia del potencial. 
  ¿Cuál es la interpretación física del voltaje entre dos puntos? 
  Sabemos  que  el  trabajo  que  realiza  la  fuerza  eléctrica  “qE”  sobre  una  carga  puntual  q,  entre  dos 
puntos P1 y P2, viene dado por la expresión 
 
P2
(21)    WE (P1 → P2 ) = ∫P 1
qE • dr  
 

de modo que, comparando (21) con (20), tenemos que 
 
 
El voltaje entre dos puntos dados es el negativo del trabajo por unidad de 
W
(22)    V = − Eléc.   carga  que  realiza  el  campo  eléctrico  sobre  una  carga  puntual  positiva 
q
cuando  ésta  se  desplaza  entre  ambos  puntos  a  lo  largo  de  cualquier 
trayectoria. 
 
 

  El  trabajo  que  realiza  el  campo  eléctrico  sobre  una  carga  puntual  q  al 
(23)    WEléc. = – qV  desplazarse  ésta  entre  dos  puntos  cuya  diferencia  de  potencial  es  V,  es 
  igual al negativo del producto de la carga y el voltaje entre ambos puntos. 
4‐12 

 
EJEMPLO  4.5.   La  figura  muestra  los  valores  del  potencial  eléctrico  en  un  conjunto  de  puntos  de  una 
región donde existe un campo eléctrico. Sea q = 5 μC una carga eléctrica dentro de esta región. 
(a) ¿Cuánto trabajo invierte el campo eléctrico cuando la carga q se desplaza del punto A al B? 
(b) ¿Cuánto trabajo debería invertir un agente externo para transportar cuasiestáticamente la carga dada 
entre los puntos E y G? 
(c)  ¿Cuánto  trabajo  invierte  el  campo  eléctrico  sobre  la  carga  cuando  ésta  se  desplaza  del  punto  F  al  A, 
luego al C, luego al E y finalmente de regreso a F? 
‐‐‐‐‐‐‐‐‐‐‐‐‐‐‐‐‐‐‐‐‐‐‐‐‐‐‐‐‐‐‐‐‐‐‐‐‐‐‐‐‐ 
  Usaremos la ecuación (23), a saber, 
 
  WEléc. = – qV        ( = – Wext) 
 
(a) Entre los puntos A y B existe una diferencia de 
potencial o voltaje igual a 
 
  V(A → B) = VB – VA = 15 V – 40 V = – 25 V 
 
 
Entonces 
 
 
Fig. 33 
  WEléc. = – 5 μC ⋅ (–25 V) = 125 μJ 
 
(b) Dado que Wext = – WEléc., el trabajo que realizaría el agente externo sería 
 
    Wext = qV 
 
donde V es el voltaje entre los puntos E y G. 
(Note que, de acuerdo con esta ecuación, para que una carga puntual positiva q pase de un punto dado a 
otro  a  mayor  potencial  eléctrico,  el  agente  externo  debe  comunicar  a  la  carga  una  energía  (en  forma  de 
trabajo) igual a “qV”. En el segundo punto la energía potencial de la carga en el campo eléctrico dado es 
precisamente mayor que en el primer punto, en una cuantía “qV”.) 
 
Entre los puntos E y G este trabajo vale 
 
    Wext = q(VG – VE) = 5 μC (16 V – (–26 V)) = 5 mC (42 V) = 210 μJ 
 
(c) El trabajo realizado por el campo eléctrico o por el agente externo cuando la carga recorre un camino 
cerrado  “F → A → C → E → F”  es cero: 
 
    WEléc.(F → A → C → E → F) = –q(V(F → A) + V( A → C) + V( C → E) + V( E → F) = 
      = –q ((40–(–9)) + (5–40) + (–26–5) + (–9–(–26))) = –q(66–66) = 0 
 
(Con respecto a este resultado consulte la Sección 4.6). 
4‐13 

 
4.6. INTEGRAL DE LÍNEA DEL CAMPO ELÉCTRICO A LO LARGO DE UNA CURVA CERRADA. 
  Sea  Γ  una  curva  cerrada.  Consideremos  la  integral  de  línea  del  campo  eléctrico  a  lo  largo  de  Γ. 
Podemos evaluar tal integral partiendo de algún punto arbitrario P y recorriendo  Γ en el sentido positivo 
hasta regresar al mismo punto P (Fig. 33). 
 

 
Fig. 22 
 
  Por la propiedad de aditividad de la integral, ésta se puede escribir en la forma 
 
S P

Γ∫ C1 ∫ P
    E • dr = E • dr + ∫ E • dr  
C2 S

 
donde C1 y C2 son porciones complementarias de la curva Γ, tal como se muestra en la Fig. 22. 
  Ahora bien, de acuerdo con la Ec. (20‐p??), tenemos 
 
S
C1 ∫ P
    E • dr = −(φ(S) − φ(P)) = −V(P → S)  

 
y también 
 
P

C2 ∫ S
    E • dr = −(φ(P) − φ(S)) = −V(S → P)  

 
En vista de que V(P → S) = – V(S → P), tenemos que la integral de línea considerada es cero. 
 
 
La  integral  de  línea  del  campo  eléctrico  a  lo  largo 
(33)   
Γ ∫ E • dr = 0   de cualquier curva cerrada Γ es cero. 
 
 
Fundamentalmente, esta propiedad de la integral de E se deduce de que E es un campo conservativo. 
4‐14 

 
4.7. POTENCIAL ELÉCTRICO DE UNA CARGA PUNTUAL. 
  He aquí el procedimiento para calcular el potencial eléctrico de una carga puntual Q, por medio de 
la integral 
 
P
[(17‐p10)]    φ(P) = − ∫ E • dr  
Γ P 0

 
• Coloquemos la carga Q en el origen de un sistema de coordenadas polares, y sea “r” la coordenada 
radial de los puntos del plano, medida entonces desde Q. La otra coordenada polar, el ángulo  θ, no 
intervendrá en el cálculo dada la simetría esférica del campo eléctrico de la carga (Fig. 6). 
• Sea  P el punto arbitrario del espacio donde deseamos evaluar el potencial eléctrico. Denotemos con 
“rP” la coordenada radial (constante) de este punto. 
• Sea P0 un punto fijo arbitrario, escogido como punto de referencia. Sea “r0” su coordenada radial. 
• Escojamos  una  trayectoria  de  integración  Γ  o  curva  que  enlace  los  puntos  P0  y  P.  Sea  “r”  la 
coordenada  radial  (variable)  de  un  punto  arbitrario  del  camino  Γ.  Sea  r  el  vector  de  posición  de  tal 
punto y tracemos allí los vectores E y  dr. 
 

 
Fig. 6 
 
La expresión vectorial del campo eléctrico de la carga, existente en el punto “r”, es 
 
kQ r r |r | kQ
    E=     (dirección radial, magnitud  E = E = kQ = kQ = ) 
r3 r3 r3 r2
 
Sustituyendo en la integral (17‐p10) del potencial tenemos 
4‐15 

rP rP kQ r rP r • dr
    φ(rP ) = − ∫
Γ r
0
E • dr = − ∫
Γ r 0 r 3
• dr = −kQ∫
Γ r 0 r3
 

 
Esta última integral es fácil de evaluar usando la relación general, válida para cualquier vector A, 
 
A • dA = A x dA x + A y dA y =
1
2
1
2
1
2
(
1
2
1
)
d(A x2 ) + d(A 2y ) = d A x2 + A 2y = d A 2 = (2AdA) = AdA  
2
( )
 
que nos da, aplicada al vector r: 
 
    r • dr = r dr          (Ojo: “dr” no es la magnitud del vector “dr”) 
 
Con esto, la integral queda 
 
r
rP r • dr rP rdr rP dr ⎡ 1⎤ P  
    φ(rP ) = − kQ
Γ r0
∫ r3
= − kQ
Γ
∫r 0 r3
= − kQ
Γ
∫r 0 r2
= − kQ ⎢⎣ − r ⎥⎦
r0

 
Evaluando para los límites de integración dados se obtiene la expresión 
 
⎛ 1 1⎞
    φ(rP ) = kQ ⎜ − ⎟  
⎝ rP r0 ⎠
 
  A  la  vista  de  esta  expresión,  conviene  hacer  lo  siguiente:  dado  que  P  es  un  punto  arbitrario  del 
espacio  (que  permanece  fijo  durante  la  integración),  podemos  renombrar  su  coordenada  “rP”  como  “r”. 
Además  podemos  escoger  el  punto  de  referencia  en  el  infinito,  con  lo  que  el  término  (1/r0)  desaparece. 
Así, 
 
  Potencial eléctrico generado por una carga puntual Q en un 
Q kQ punto arbitrario a distancia “r” de la carga. 
(24)    φ(r) = =  
4 πε0 r r El punto de referencia está en el infinito  ( lím φ(r) = 0 ). 
r →∞
 
 
Observaciones: 
• Dado que siempre  r  > 0, tenemos que si  Q es positiva, entonces el potencial eléctrico es positivo en 
todo  el  espacio.  Si  Q  es  negativa,  el  potencial  eléctrico  es  negativo  en  todo  el  espacio.  El  potencial 
tiene una singularidad en el punto donde se encuentra la carga, donde se hace infinito. 
• Si  las  coordenadas  radiales  de  P1  y  P2 son  r1  y  r2,  respectivamente,  entonces  el  voltaje  entre  ambos 
puntos, debido al campo eléctrico de la carga puntual Q,  es 
 
⎛ 1 1⎞ Voltaje  entre  dos  puntos  P1  y  P2  en  el 
(25)    V(P1 → P2 ) = φ(r2 ) − φ(r1 ) = kQ ⎜ − ⎟  
⎝ r2 r1 ⎠ campo de una carga puntual Q. 
 
4‐16 

  Recordando la relación U = qφ  entre el potencial eléctrico φ producido por una distribución D y la 
energía  potencial  eléctrica  U  del  sistema  {D,  q},  podemos  agregar  lo  siguiente  (donde  D  es  ahora 
simplemente otra carga puntual Q): 
 
 
   
  Energía potencial eléctrica del sistema de dos 
kQq cargas  puntuales  {Q,  q},  en  cualquier  confi‐
(26)    U=  
r guración  en  que  la  distancia  entre  las  cargas 
  es “r”. 
 
Fig. 7 
 
 
4.8. POTENCIAL ELÉCTRICO DE UN SISTEMA DE CARGAS PUNTUALES. 
  Sea  {Q1, Q2, …, QN}  un sistema de cargas puntuales que ocupan puntos fijos en el espacio. Sean  E1, 
E2, …, EN  los campos eléctricos individuales producidos por cada carga, respectivamente. 
  De  acuerdo  con  (17‐p10),  el  potencial  en  el  punto  arbitrario  P,  generado  por  el  sistema  de  cargas 
puntuales, es 
 
P
    φ(P) = − ∫ E • dr  
Γ P 0

 
donde E = E1 + E2 + … + EN es el campo eléctrico debido a todas estas cargas. Sustituyendo este campo en la 
expresión anterior del potencial y usando la propiedad de distributividad de la integral de línea, 
 
P P P P
    φ(P) = − ∫
Γ P0
(E1 + E2 + ... + EN ) • dr = −
Γ P0
∫ E1 • dr − ∫
Γ P0
E2 • dr − ... − ∫
Γ P0
EN • dr  
 

Pero las integrales del miembro derecho son los potenciales individuales que producirían las cargas por sí 
solas. Se obtiene entonces 
 

    φ(P) = φ1(P) + φ2(P) + … + φN(P) 
 

Este  resultado  se  puede  generalizar:  “El  potencial  eléctrico  producido  por  un  conjunto  de  cargas 
puntuales  y/o  distribuciones  continuas  de  carga  es  la  suma  algebraica  de  los  potenciales  eléctricos 
producidos por cada carga y/o distribución”. Decimos pues que el potencial es una cantidad “aditiva”. 
  Utilizando  esta  propiedad  de  aditividad,  junto  con  la  expresión  (24)  del  potencial  producido  por 
una carga puntual, tenemos: 
 
    Potencial  eléctrico  debido  a  un 
 
conjunto  de  cargas  eléctricas  puntuales 
N
kQ N
∑ rii  
kQ1 kQ 2 Q Q1,  Q2,  …,  QN,  en  un  punto  P  arbitrario 
(27)    φ(P) = + + ... + =k
r1 r2 rN
i =1 del  espacio,  a  distancias  respectivas    r1, 
r2, …, rN de las cargas (Fig. 8). 
4‐17 

 
 

 
Fig. 8 
 
EJEMPLO 4.6.   ENERGÍA DEL ÁTOMO DE HIDRÓGENO EN EL MODELO DE BOHR. 
  En el modelo de Bohr del átomo de hidrógeno, el electrón describe órbitas circulares alrededor del 
protón (Fig. 9). 
  De  acuerdo  con  (20),  la  energía  potencial 
eléctrica del sistema {protón, electrón} es 
 

ke 2
    U=−  
r
 

donde 
 

    e = 1.6 (10–19) C 
 
 
es la carga del protón y “–  e” la del electrón, y “r”   
es el radio de la órbita del electrón.  Fig. 9 
  Suponiendo que el protón está fijo y que el 
electrón  se  mueve  circularmente  con  velocidad  “v”,  la  energía  cinética  del  átomo  es  (puramente  la  del 
electrón): 
 
1
    K= me v2  
2
 
donde  me es la masa del electrón. Entonces la energía total “E” del átomo de hidrógeno en el modelo de 
Bohr viene dada por 
 
ke 2 1
    E=− + me v2       (Protón fijo) 
r 2
 
  Podemos  dar  otra  expresión  más  simple  para  esta  energía,  aplicando  algo  de  dinámica  clásica. 
Apliquemos la segunda ley de Newton al movimiento circular del electrón alrededor del protón. 
4‐18 

  El  electrón  ejecuta  un  movimiento  circular   


uniforme alrededor del protón. La fuerza centrípeta 
que  lo  impulsa  es  la  fuerza  eléctrica  debida  al 
protón. Si el radio de la órbita electrónica es “r”, tal 
fuerza tiene una componente radial (única) igual a 
 
ke 2
    Fr = −  
r2
 
 
  La  aceleración  del  electrón  es  puramente 
Fig. 10 
centrípeta, y vale 
 

v2
    ar = −  
r
 
Ahora bien, de acuerdo con la segunda ley de Newton, 
 
⎛ v2 ⎞ ke 2 v2
    Fr = m e a r = m e ⎜ − ⎟        ⇒       − = −m e  
⎜ ⎟ r ⎠ r2 r

 
de donde 
 

1 ke 2
    me v 2 =  
2 2r
 

Sustituyendo esto en la expresión de la energía total tenemos 
 
ke 2 1 ke 2 ke 2 ke 2
    E=− + me v2 = − + =−  
r 2 r 2r 2r
 
 
 
ke 2
(21)    E=−   Energía total del átomo de hidrógeno 
2r
 
 
  En la primera órbita de Bohr, para la cual  r = 0.053 nm, la energía del átomo es 
 

ke 2
    E=− =  –21.73 (10–19) J 
2r
 

(Nota.  En  unidades  “electronvoltio”  de  energía,  definidas  como  “1  eV  =  1.6  (10–19)  J”,  esta  energía  es 
igual a 13.6 eV.) 
‐‐‐‐‐‐‐‐‐‐‐‐‐‐‐‐‐‐‐‐‐‐‐‐‐ 
4‐19 

 
EJEMPLO 4.7.  Calcular el potencial debido al cuadrupolo mostrado en la figura, en los puntos de su eje 
longitudinal. La distancia entre carga y carga es “a”. 
 

 
 
Fig. 11 
‐‐‐‐‐‐‐‐‐‐‐‐‐‐‐‐‐‐‐‐‐‐‐‐‐‐‐‐‐‐‐‐‐‐‐‐‐‐‐‐‐‐‐ 
  Coloquemos  un  Eje  X  en  la  línea  que  une  las  cargas,  con  su  origen  en  la  carga  central.  Sea  P  un 
punto arbitrario del Eje X, de coordenada “x”. 
  Usaremos esto que conocemos desde la geometría analítica: la distancia entre dos puntos del Eje X, 
de coordenadas x1 y x2, es  |x2 – x1|. 
  Entonces las distancias del punto P a las cargas +q izquierda, –2q y +q derecha son respectivamente 
 
    |x + a|,    |x|    y    |x – a| 
 
 de modo que el potencial pedido es, por (22), 
 
⎛ q ( −2q) q ⎞
    φ(x) = k ⎜ + + ⎟    (x ≠ 0, x ≠ a, x ≠ ‐a) 
⎝|x + a| |x| |x − a|⎠
 
  Para  x > a tenemos  |x + a| = x + a,   |x| = x  y  |x – a| = x – a, así que podemos quitar las barras de 
valor absoluto y poner 
 
⎛ q ( −2q) q ⎞
    φ(x) = k ⎜ + + ⎟ 
⎝ x+a x x−a ⎠
 
que podemos simplificar a 
 
2kqa 2
φ(x) =
( )
(23)      ( x > a ) 
x x2 − a 2

 
Para puntos P muy lejanos del dipolo, este potencial se puede poner  
 
2kqa 2
    φ(x) = .    (x >> a) 
x3
 
¿Cuál sería la expresión (sin barras de valor absoluto) del potencial eléctrico para x < – a? 
4‐20 

 
EJEMPLO 4.8..   
  Se  tiene  un  sistema  de  4  cargas  puntuales 
fijas  en  los  vértices  de  un  cuadrado  de  lado  “2a”, 
como se ve en la Fig. A. 
(a)  Calcular  el  potencial  eléctrico  debido  a  estas  4 
cargas en el origen de coordenadas O y en el punto 
P. 
(b) ¿Cuánto trabajo debe realizar el campo eléctrico 
de  las  4  cargas  cuando  una  carga  puntual  q  se 
desplaza desde O hasta P? 
(c) ¿Cuál es el voltaje entre los puntos O y P debido 
al campo eléctrico del cuadrupolo? 

 
Fig. 12 
‐‐‐‐‐‐‐‐‐‐‐‐‐‐‐‐‐‐‐‐‐‐‐‐‐‐‐‐‐‐‐‐‐‐‐‐‐‐‐‐ 
 
(a) Todas las cargas están a la misma distancia  “ 2a ”  del origen O. Aplicando la Ec. (22) tenemos 
 
⎛ −3Q 2Q −Q 2Q ⎞
    φ(O) = k ⎜ + + + ⎟=0 
⎝ 2a 2a 2a 2a ⎠
 
Por  otra  parte,  el  punto  P  está  a  distancia    “ 2a ”  de  las  cargas  “–3Q”  y  “2Q”  superior,  y  a  distancia 
“ 10a ” de las cargas “–Q” y “2Q” inferior. Entonces, 
 
⎛ −3Q 2Q −Q 2Q ⎞ kQ ⎛ 1 1 ⎞
    φ(P) = k ⎜ + + + ⎟ = a ⎜− + ⎟ 
⎝ 2a 2a 10a 10a ⎠ ⎝ 2 10 ⎠
 
(b) Recordemos que en la página 4‐10 se encontró que “El voltaje entre dos puntos dados es el negativo 
del  trabajo  por  unidad  de  carga  que  realiza  el  campo  eléctrico  sobre  una  carga  puntual  cuando  ésta  se 
desplaza entre ambos puntos dados a lo largo de cualquier trayectoria”. 
  En símbolos,  V = – WE / q    (Véase la Ec. (18)) 
  Por consiguiente, el trabajo del campo eléctrico del cuadrupolo sobre “q” desde O hasta P es 
 
kqQ ⎛ 1 1 ⎞
    WE = −q(φ(P) − φ(O)) = ⎜ − ⎟ 
a ⎝ 2 10 ⎠
 
(c) El voltaje entre los puntos O y P es 
 
kQ ⎛ 1 1 ⎞
    V(O → P) = φ(P) − φ(0) = ⎜ − + ⎟ 
a ⎝ 2 10 ⎠
4‐21 

 
4.9. UNA CARGA PUNTUAL MÓVIL EN EL CAMPO DE VARIAS CARGAS PUNTUALES FIJAS. 
  En secciones anteriores encontramos los siguientes resultados: 
• Un  sistema  formado  por  una  distribución  de  carga  fija  D  y  una  carga  puntual  q  posee  una  energía 
potencial eléctrica dada por  U = qφ(P), donde  φ(P) es el valor del potencial en el punto P donde se 
encuentre q. (Consulte la Ec. (13) en la página 4‐9). 
• Un  sistema  de,  digamos,  5  cargas  puntuales  {Q1,  Q2,  …,  Q5},  produce  en  el  punto  P  un  potencial 
eléctrico dado por la expresión (Consulte la Ec. (22) en la página 4‐15) 
 
5
kQ 5
∑ rii  
kQ1 kQ 2 Q
    φ(P) = + + ... + = k⋅
r1 r2 r5
i =1
 
donde r1, r2, …, rN son las distancias de P a las cargas Q1, Q2, …, Q5, respectivamente. 
  Combinando  estos  dos  resultados  podemos   
afirmar  que  el  sistema  {D,  Q6},  donde  D  es  el 
conjunto de las 5 cargas puntuales fijas {Q1, …, Q5}, 
y  Q6  es  otra  carga  puntual  adicional  móvil,  posee 
una energía potencial eléctrica dada por 
 
(24)    U = Q6 φ(P)  
 
kQ1Q6 kQ2 Q6 kQ5Q6
    = + + ... +  
r1 r2 r5
 
 
(Recuerde:  otros  nombres  para  la  expresión  (24)  Fig. 13 
son: “energía potencial eléctrica de Q6 en el campo  
de Q1, Q2, …, Q5”, o bien simplemente “energía potencial eléctrica de la carga Q6”.) 
  Un uso que podríamos dar a la expresión (24) sería para investigar el movimiento de la carga  Q6 
dentro del campo de las cargas fijas  Q1,  Q2, …,  Q5, mediante la ley de conservación de la energía. Aquí 
subyace  una  hipótesis  fundamental,  a  saber,  que  la  distribución  D  =  {Q1,  Q2,  …,  Q5}  está  fija,  es  decir, 
estas 5 cargas están inmóviles y a distancias constantes unas de otras (podemos imaginar que están unidas 
por  unas  varillas  ligeras  muy  delgadas  y  rígidas,  aislantes,  como  se  sugiere  en  la  Fig.  13).  De  hecho,  ya 
hemos estudiado movimientos de cargas dentro del campo eléctrico de distribuciones fijas en la Sección 
4‐3 y en los  Ejemplos 4.2, 4.4 y 4.6, y hemos aplicado la ley de conservación de la energía total. 
  Le energía total del sistema formado por las 5 cargas fijas {Q1, Q2, …, Q5}  y la carga móvil Q6 es 
 
1
(25)    E (P) =  m 6 v 62 + Q6 φ (P) 
2
 
donde  m6 y  v6 son la masa y velocidad de la carga  Q6, respectivamente, y  φ(P) es el potencial debido a 
las 5 cargas {Q1, Q2, …, Q5} en el punto donde se encuentre Q6. 
 La ley de conservación de la energía expresaría entonces que esta E(P) es constante. Veamos un ejemplo. 
4‐22 

 
EJEMPLO 4.9.  Dos cargas iguales, “Q”, que se pueden considerar puntuales, están unidas por una varilla 
aislante,  rígida  y  ligera,  y  separadas  una  distancia  “a”.  Desde  el  punto  medio  entre  ambas  cargas  se 
proyecta una carga negativa “–q” perpendicularmente a la línea de unión (Fig. 14). ¿Qué velocidad “ve” 
debe dársele a esta carga para que escape de la atracción eléctrica de las dos cargas fijas? 
‐‐‐‐‐‐‐‐‐‐‐‐‐‐‐‐‐‐‐‐‐‐‐‐‐‐‐‐‐‐‐‐‐‐‐‐‐‐‐‐‐‐‐‐‐ 
  Teóricamente,  debemos  impulsar  la  carga  “–  q”  con   
suficiente velocidad para que “justamente alcance a llegar al 
infinito”,  lo  cual  significa  simplemente  que  su  velocidad 
debe tender a cero conforme la distancia crece a infinito. 
  Pongamos un Eje X con su origen en el punto medio 
de ambas cargas Q, y su dirección coincidente con la recta 
de movimiento de la carga negativa. 
  El potencial eléctrico debido a las dos cargas Q en el 
punto arbitrario “x” es 
 
2Q  
    φ(x) = k ⋅  
 
(a / 2)2 + x 2
 
Fig. 14 
Por tanto, la energía potencial eléctrica de la carga “–q” en  
el campo de las dos cargas positivas Q, en el punto “x”,  es 
 
−2kqQ
    U = −q φ(x) =  
(a / 2)2 + x 2
 
Si la velocidad de la carga negativa en dicho punto es “v”, entonces la energía total del sistema formado 
por las dos cargas fijas y la carga móvil, en una posición arbitraria “x”, es 
 
1 2kqQ
    E =  mv 2 −  
2 2
(a / 2) + x 2

 
(m = masa de la carga negativa). En el punto de partida (x = 0, v = ve), la energía total es 
 
1 4kqQ
    E  = mve2 −  
2 a
 
Por otra parte, en el infinito  (x = ∞ , v = 0), la energía total es cero. Por lo tanto, despejando la velocidad 
ve de la ecuación E = 0 obtenemos 
 
8kqQ
    ve =  
ma
‐‐‐‐‐‐‐‐‐‐‐‐‐‐‐‐‐‐‐‐‐‐‐‐‐‐‐‐‐‐‐‐‐‐‐‐‐‐‐‐‐‐‐‐‐‐‐‐‐‐‐‐‐‐ 
4‐23 

 
4.10. SOBRE EL CÁLCULO DE POTENCIALES ELÉCTRICOS. 
  En  esta  sección  obtendremos  otra  fórmula  integral  más  simple  que  (12)  para  el  potencial,  válida 
para las siguientes situaciones: 
• Se desea calcular el potencial eléctrico en puntos sobre una línea de fuerza recta del campo E. 
• El campo  E depende exclusivamente  de una sola variable de posición (o coordenada) medida a lo largo 
de dicha línea de fuerza. 
  Ilustraremos en el siguiente ejemplo el método, así como la integral que resulta para el potencial, 
que luego generalizaremos a otros casos similares. 
 
Ejemplo.  Calcular  el  potencial  debido  a  un  alambre  recto  infinito  cargado  uniformemente  con  densidad 
lineal de carga “λ”, en un punto arbitrario del espacio. 
‐‐‐‐‐‐‐‐‐‐‐‐‐‐‐‐‐‐‐‐‐‐‐‐‐‐ 
  Primeramente recordemos cómo es el campo 
eléctrico del alambre (Fig. 15). 
  Las  líneas  de  fuerza  son  rectas  ortoaxiales, 
contenidas en planos perpendiculares al alambre. El 
campo  apunta  alejándose  del  alambre  si  λ  es 
positiva,  y  hacia  él  si  es  negativa.  El  campo 
disminuye  proporcionalmente  con  la  distancia  al 
alambre (factor de proporcionalidad = λ/2πε0). 
  Estudie bien la Fig. 16. Hemos establecido un 
Eje X con su origen O en el alambre, y dirigido a lo 
largo de una línea de fuerza recta perpendicular al 
  alambre. La coordenada medida desde O a lo largo 
Fig. 15  del  Eje  X  se  denotará  con  “x”.  La  componente 
(única)  del  campo  eléctrico  a  lo  largo  del  Eje  X  es, 
como sabemos (Consulte la Ec. (21) en la pág. 2.12): 
 
 
λ
(26)    Ex =  
2 πε0 x
 

La  dirección  de  E  es  la  del  vector  unitario  i  a  lo 


largo del Eje X, de modo que vectorialmente: 
 
λ
    E(x) = E x i = ⋅i  
2 πε0 x
 

Para  obtener  el  potencial  eléctrico  en  un  punto  P 


  cualquiera  (de  coordenada  “x”)  de  la  línea  de 
Fig. 16  fuerza (o Eje X), hay que integrar este campo desde 
algún punto de referencia P0 hasta P. 
Como trayectoria de integración  Γ se escoge precisamente el segmento  P0P de la línea de fuerza recta o 
Eje X. Entonces un punto cualquiera de este segmento tiene vector de posición dado por “  r = x i  “, y el 
desplazamiento infinitesimal se expresa como “  dr = dx ⋅ i  “. 
4‐24 

  Sustituyamos estas expresiones para E y dr en la integral del potencial, o sea 
 
P
[(12)]    φ(P) = − ∫
Γ P0
E • dr  

 
Obtenemos 
 
xP xP
    φ(xP ) = − ∫
Γ x 0
E • dr = −∫
Γ x 0
(Ex i) • (dx i)  

 
Tomando en cuenta que  i • i = 1 tenemos 
 
xP
(27)    φ(xP ) = − ∫x 0
Ex dx  
 

Hemos reducido así la integral de línea (12) a una integral ordinaria de una variable. 
  Sustituyendo (26) en (27) obtenemos el potencial del alambre recto infinito: 
 
xP λ λ
    φ(xP ) = − ∫x dx = − [ln(x)] xxP0  
0 2 πε0 x 2 πε0
 
λ ⎛x ⎞
    φ(xP ) = − ln ⎜ P ⎟  
2πε0 ⎝ x0 ⎠
 

Escogiendo  el  punto  de  referencia  en  x0  =  1  (unidad  de  medida  de  longitud  usada)  y  renombrando  la 
variable “xP” simplemente con “x” tenemos finalmente 
 

λ
(28)    φ(x) = − ln(x)   Potencial eléctrico de un alambre infinito en 
2 πε0
            un punto a una distancia “x” del alambre. 
 
  Ahora bien, aunque en este ejemplo hemos trabajado con una coordenada “x” medida a lo largo de 
una línea de fuerza recta, en otros casos esta coordenada podría designarse como “r”, o “y”, etc., según el 
campo eléctrico de que se trate. El procedimiento para calcular el potencial sería similar, llegándose a una 
fórmula análoga a (27), en la que en lugar de “x” aparecería la coordenada usada en cada caso. Entonces, 
llamando en general “u” a la coordenada en cuestión, podemos dar la siguiente expresión general: 
 
  Fórmula general del potencial eléctrico en un punto arbitrario de 
  una  línea  de  fuerza  recta  del  campo  eléctrico.  “u”  es  la 
u
∫u Eu du  
coordenada  medida  a  lo  largo  de  la  línea  de  fuerza,  y  se  ha 
(29)    φ(u) = −
0 escogido  el  punto  de  referencia  sobre  dicha  línea.  Eu  es  la 
  componente  (única)  del  campo  eléctrico,  la  cual  se  supone 
  dependiente solamente de la variable “u”. 
 
‐‐‐‐‐‐‐‐‐‐‐‐‐‐‐‐‐‐‐‐‐‐‐‐‐‐‐‐‐‐‐‐‐ 
4‐25 

 
EJEMPLO 4.10.  Calcular el potencial eléctrico debido al campo de una esfera sólida de radio R cargada 
uniformemente con carga total Q, en puntos dentro y fuera de la esfera. 
  En el capítulo 3 calculamos ya el campo de una esfera tal. Repitamos aquí los resultados (Fig. 17). 
 
  El  campo  posee  líneas  de  fuerza 
radiales  (hacia  fuera  si  Q  es  positiva  y  hacia 
dentro  si  es  negativa).  Denotando  con  “r”  la 
coordenada radial medida desde el centro de 
la  esfera,  tenemos  para  la  componente  radial 
(única) del campo las expresiones 
 
Qr
    Er =   para  r ≤ R 
4 πε0 R 3
 
Q
    Er =      para r ≥ R 
4 πε0 r 2
 
Notemos  que  fuera  de  la  esfera  el  campo  es 
  idéntico al que  produciría  una  carga  puntual 
 
Q colocada en el centro de la esfera. 
Fig. 17 
 
  Dado que el campo eléctrico posee líneas de fuerza que son rectas (radiales), y que el campo a lo 
largo de una línea de fuerza depende solamente de la variable “r”, podemos aplicar la fórmula (29) que, 
adaptada al caso presente, escribiríamos en la forma 
 
r
    φ(r) = − ∫ r Er dr  
0
 

Escogeremos por conveniencia el punto de referencia en el infinito. 
  Para puntos fuera de la esfera tenemos 
 
r Q Q r dr Q
(30)    φ(r) = − ∫ ∞ 4 πε r 2
0
dr = −
4 πε0 ∫ ∞ r 2 = 4πε0r     (r ≥ R) 

 
(idéntico al potencial de una carga puntual Q situada en el centro de la esfera). 
  Para puntos dentro de la esfera (r  ≤ R) tenemos que separar la integral en dos partes: una desde el 
infinito  hasta  el  punto  r = R,  y  otra  desde  el  punto  r = R  hasta  el  punto  final “r”  considerado.  En  cada 
parte debemos utilizar la expresión vigente para el campo eléctrico. Se obtiene 
 
R r R Q r Qr
    φ(r) = − ∫∞ Er dr − ∫R Er dr = − ∫ ∞ 4 πε r 2
0
dr − ∫ R 4πε0 R 3 dr  
 
Q Q ⎛ r2 R 2 ⎞ Q ⎛ 3R 2 − r 2 ⎞
(31)    φ(r) = − ⎜ − ⎟ = ⎜ ⎟⎟     (r ≤ R) 
4 πε0 R 4πε0 R 3 ⎜⎝ 2 2 ⎟⎠ 4 πε0 ⎜⎝ R 3 ⎠
4‐26 

EJEMPLO 4.11.  CAPACITOR ESFÉRICO. 
  Se denomina capacitor esférico a un dispositivo formado por dos cascarones esféricos concéntricos, 
digamos de radios “a” y “b”, que poseen cargas respectivas +Q y –Q distribuídas uniformemente en su 
superficie.  El  campo  eléctrico  es  cero  dentro  del  cascarón  interior  (r  ≤  a)  y  también  fuera  del  cascarón 
exterior (r ≥ b). Véase la Fig. 18. 
 

 
Fig. 18 
 
  En  un  punto  P  arbitrario  del  espacio  entre  los  dos  cascarones,  de  coordenada  radial  “r”  medida 
desde el centro del cascarón interior, el campo eléctrico vale (Consulte el Ejemplo 3.3 en la página 3‐14) 
 
kQ
    Er =  
r2
 
Aplicando la fórmula (29), tenemos para el potencial eléctrico en la región a ≤ r ≤ b la expresión 
 
r
r r kQ ⎡1⎤ ⎛1 1⎞
    φ(r) = − ∫b Er dr = − ∫ b r2
dr = kQ ⎢ ⎥ = kQ ⎜ − ⎟  
⎣r ⎦ b ⎝r b⎠
 
en donde hemos fijado el punto de referencia en el cascarón exterior (r0 = b). 
  El voltaje a través del capacitor es 
 
⎛1 1⎞
    V ≡ φ(a) − φ(b) = kQ ⎜ − ⎟ − 0  
⎝a b⎠
 

o sea 
 

   
⎛1 1⎞ Voltaje a través de un capacitor esférico. 
(32)    V = kQ ⎜ − ⎟  
⎝a b⎠ (Carga +Q en r = a, y ‐Q en r = b, con a < b). 
 
4‐27 

 
 
EJEMPLO  4.12.   Se  aplica  un  voltaje  V  entre  dos  cascarones   
conductores  esféricos  concéntricos  de  radios  a  y  b.   
Demuestre  que  si  el  cascarón  exterior  se  “aterriza”  (o  sea  se 
conecta  a  tierra,  estableciéndose  φ  ≡  0  allí),  entonces  el 
potencial en el espacio entre los cascarones es 
 
(b − r)a
  φ(r) = V  
(b − a)r

 
‐‐‐‐‐‐‐‐‐‐‐‐‐‐‐‐‐‐‐‐‐‐‐‐‐‐‐‐‐‐‐‐ 
  Antes  de  aplicar  el  voltaje  V  entre  los  cascarones,  no  estaban  cargados.  Al  aplicarlo,  el  cascarón 
interior se carga con “q” y el exterior con “–q”. Existe campo eléctrico solamente en el espacio entre ambos 
cascarones, y viene dado por 
 
q
    Er =  
4 πε0 r 2
 
donde “r” es la coordenada radial medida desde el centro de ambos cascarones. Empleando la Ec. (29‐p24) 
tenemos que el potencial eléctrico es 
 
r r q ⎛1 1⎞
    φ(r) = − ∫b Er dr = − ∫ b 4πε0r 2 dr = kq ⎜⎝ r − b ⎟⎠     (a ≤ r ≤ b) 

 
Advierta que el punto de referencia del potencial, donde  φ ≡ 0, se ha tomado en  r = b, como especifica el 
problema. Por eso la integral para el potencial tiene como límite inferior el valor “b” para “r”. 
  Imponiendo la condición de que el potencial en r = a sea igual a V tenemos 
 
⎛1 1⎞
    φ(a) = V = kq ⎜ − ⎟  
⎝a b⎠
 
De aquí encontramos la carga “q”: 
 
abV
    q=  
k(b − a)
 
Sustituyéndola en la expresión anterior para φ(r) hallamos 
 
abV ⎛ 1 1 ⎞ (b − r)a
    φ(r) = ⎜ − ⎟=V
b−a⎝ r b⎠ (b − a)r  
 
(Se comprueba que φ(b) = 0 y φ(a) =V). 
4‐28 

 
4.11. CÁLCULO DEL POTENCIAL ELÉCTRICO A PARTIR DE LA CARGA. 
  La ecuación 19 en la página 4.12 da el potencial producido por una carga puntual Q en un punto a 
distancia “r” de la carga, a saber, 
 
kQ
[(19)]    φ(r) =  
r
 
  En virtud de que toda distribución de carga, por compleja que sea, puede subdividirse en un gran 
número  de  cargas  puntuales,  podemos  usar  la  expresión  (19)  y  los  métodos  del  cálculo  integral  para 
calcular el potencial eléctrico producido por la distribución. Veamos. 
  En la Fig. 19 se representa una distribución de carga arbitraria. Para calcular su potencial en el 
punto  arbitrario  P,  se  divide  la  distribución  en  un   
gran  número  de  elementos  de  carga  dQ,  tan 
pequeños que se pueden considerar como si fuesen 
cargas puntuales. El elemento general  dQ produce 
entonces en P un potencial dado, según (19), por 
 

kdQ
    dφ =  
R
 

Sumando  algebraicamente  los  potenciales 


producidos  por  todos  los  elementos  de  carga  que 
forman la distribución obtenemos 
   
dQ  
(33)    φ=k
R ∫
    
Fig. 19 

 
Esta integral presupone que el punto de referencia es el infinito. 
 
EJEMPLO 4.13.  Calcular el potencial producido por un anillo delgado de carga total uniforme  Q y radio 
“a”, en un punto P arbitrario sobre su eje de simetría perpendicular (Fig. 20). 
  Para  aplicar  la  fórmula  (33),  primeramente  trazamos  el  elemento  de  carga  general  dQ,  como  se 
muestra en la Fig. 20. Luego trazamos la línea que une dQ con P. 
 

  Advirtamos que  todos  los  elementos  de  carga 


del anillo se encuentran a la misma distancia “R” del 
punto campo P, de tal modo que la integral es trivial:
 
1 dQ 1
    φ=
4 πε0 ∫ R
= ∫
4 πε0 R
dQ  

 
Q Q
    = =  
4 πε0 R 4 πε a 2 + z 2
0
 
Fig. 20   
4‐29 

 
  Hubiéramos obtenido el mismo potencial aplicando la fórmula (29) de la página 4‐21. Consultando 
el campo eléctrico del anillo (Ecuación (24), página 2‐15) tendríamos 
 
z z Q z
    φ(z) = − ∫∞ E z dz = − ∫ ∞ 4πε0 3
dz  
( )
a 2 + z2 2
 
El resultado de esta integral es la expresión obtenida al final de la página anterior. 
 
EJEMPLO  4.14.  Calcular  el  potencial  debido  a  un  trozo  de  alambre  recto  de  carga  total  uniforme  Q  y 
longitud L, en un punto arbitrario sobre la recta perpendicular al alambre, a través de su centro. 
 

 
Fig. 21 
 
  Tenemos 
 
    dQ = λ dy’ = (Q/L) dy’ 
 
    R = x 2 + y ′2  
 
Entonces, 
 

1 dQ 1 Qdy′
    φ= ∫ =
4 πε0 R 4πε0 ∫L x 2 + y′2
 

 
La integral debe abarcar todo el trozo de alambre, es decir, y’ va desde –L/2 hasta L/2. El resultado es 
 
⎛ L + 4x 2 + L2 ⎞
( )
L/2
Q ⎡
ln y′ + x 2 + y′2 ⎤
Q
    φ= = ln ⎜ ⎟   
4 πε0 L ⎣⎢ ⎦⎥ −L / 2 4 πε0 L ⎜ −L + 4x 2 + L2 ⎟
⎝ ⎠
 
El punto de referencia de este potencial es el infinito (Note que  lím φ(x) = 0 ). 
x →∞
4‐30 

 
4.12. SUPERFICIES EQUIPOTENCIALES. 
  Una  superficie equipotencial del campo eléctrico es una superficie tal que el potencial eléctrico es 
constante en todos sus puntos. 
Ejemplos. 
1.  Las superficies equipotenciales del campo eléctrico de un capacitor de placas paralelas son planos 
paralelos a las placas. Esto lo vemos de la expresión del potencial eléctrico, a saber 
   
σy
    φ = Ey =  
ε0
 
Todos  los  puntos  con  la  misma 
coordenada  “y”  están  al  mismo 
potencial.  Estos  puntos  forman  un 
plano (Fig. 4.10) cuya ecuación es 
 
  y = constante 
   
El  campo  eléctrico  del  capacitor es  Fig. 23 
perpendicular  a  estos  planos  en 
todo punto. 
  Otro  ejemplo:  el  potencial  eléctrico  producido  por  una  esfera  de  radio  R,  cargada  uniformemente 
con carga Q, viene dado en puntos fuera de la esfera por 
 
Q
    φ=  
4 πε0 r
 

Por tanto, el potencial eléctrico es constante en puntos a la misma distancia “r” del centro de la esfera, los 
cuales  están  sobre  esferas  concéntricas  a  la  carga.  Estas  esferas  son  las  superficies  equipotenciales  del 
campo de la esfera. 
  Notemos  que  también  en  este  caso,  el  campo  eléctrico  E  es  perpendicular  en  todo  punto  a  la 
superficie equipotencial que pasa por el punto. 
 

 
Fig. 24 
4‐31 

 
  Para el campo dentro de la esfera, las superficies equipotenciales siguen siendo esferas concéntricas, 
pues el campo allí depende solamente de (y es proporcional a) “r”, de tal manera que la ecuación de estas 
superficies equipotenciales es  “r = constante”. 
  Vamos  a  demostrar  a  continuación  que  las  líneas  de  fuerza  del  campo  eléctrico  cortan  a  las 
superficies equipotenciales perpendicularmente en todo punto. 
  En la Fig. 4.12 se representan las líneas de fuerza de algún campo eléctrico  E. La línea transversal 
mostrada,  que  pasa  por  los  puntos  P  y  S,  se  ha  trazado  de  modo  que  corra  perpendicularmente  a  las 
líneas  de  fuerza.  Vamos  a  demostrar  que  el  potencial  eléctrico  es  constante  a  lo  largo  de  dicha  línea 
transversal. 
 

 
Fig. 25 
 
  Sabemos que la diferencia de potencial eléctrico entre los puntos P y S viene dado por la integral  
 
S
[(16)]    V(P → S) = − ∫ P E • dr  
 

Por otra parte, el segmento de curva que une los puntos P y S, mostrado en la Fig. 4.12 es perpendicular 
en cada uno de sus puntos al campo eléctrico existente en esos puntos. Por lo tanto, la integral anterior es 
cero,  puesto  que  E  y  dr  son  perpendiculares  a  lo  largo  del  segmento  PS,  y  su  producto  escalar  es  cero. 
Esto demuestra que 
 
    V(P → S) = φ(S) – φ(P) = 0 
 

o sea que 
 

    φ(S) = φ(P) 
 
Hemos demostrado así que todos los puntos sobre una línea que corra perpendicularmente a las líneas de 
fuerza del campo eléctrico están al mismo potencial. 
  En  el  caso  de  tres  dimensiones  no  tenemos  líneas  (curvas)  equipotenciales,  sino  superficies 
equipotenciales. El potencial eléctrico es una función de la posición: φ = φ(x, y, z). El conjunto de puntos 
4‐32 

donde el potencial tiene un valor fijo (constante) C obedece la ecuación 
 
    φ(x, y, z) = C 
 
que  representa  una  superficie  en  el  espacio  cartesiano  XYZ.  Esta  se  denomina  superficie  equipotencial  del 
campo (Fig. 4.13). En los puntos de una superficie equipotencial, el campo es perpendicular a la superficie, 
esto es, perpendicular al plano tangente a la superficie en el punto considerado o, equivalentemente, está 
en la misma dirección que el vector área da en ese punto. 
 

 
Fig. 26 
 
EJEMPLO  4.15.    La  Fig.  4.15  muestra  las  líneas  de  fuerza  del  campo  eléctrico  generado  por  una  carga 
“6q” (izquierda) y una carga “–q” (derecha), así como algunas superficies equipotenciales. 
 

 
Fig. 30 
4‐33 

 
EJEMPLO 4.16.  Misma cosa que el ejemplo anterior, para cargas “–3q” (izquierda), “–2q” (abajo) y “2q” 
(arriba). 
 

 
Fig. 31 
 
4.13. RELACIÓN ENTRE EL POTENCIAL (O VOLTAJE) Y EL CAMPO ELÉCTRICO. 
  En secciones anteriores hemos introducido dos expresiones integrales para el potencial eléctrico, a 
saber, 
 
P
[(12)]      φ(P) = − ∫
Γ P0
E • dr     (pág. 4‐9) 
 
u
[(29)]      φ(u) = − ∫u Eu du  
0
  (pág. 4‐21) 
 

la segunda válida cuando el campo eléctrico depende de una sola variable “u” medida a lo largo de una 
línea de fuerza recta del campo. 
  Deseamos ahora investigar las relaciones inversas a éstas. 
  La  relación  inversa  de  la  expresión  (29)  es  inmediata.  Del  cálculo  diferencial  de  una  variable 
sabemos que si φ es la integral de Eu, entonces Eu es la derivada de φ. Es decir, la inversa de (29) es 
 
  Relación  entre  el  campo  eléctrico  y  el  potencial 
dφ eléctrico  cuando  el  campo  posee  líneas  de  fuerza 
(37)    Eu = −  
du rectas  y  depende  de  una  coordenada  medida  a  lo 
  largo de la línea de fuerza. 
 
4‐34 

 
EJEMPLO 4.17.    El potencial en puntos dentro de una esfera sólida cargada es 
 
Q ⎛ 3R 2 − r 2 ⎞
    φ(r) = ⎜ ⎟⎟  
4 πε0 ⎜⎝ R 3 ⎠
 

Por (37) (donde “u” de (37) es ahora “r”), el campo eléctrico en puntos dentro de la esfera es 
 
dφ d ⎡ Q ⎛ 3R 2 − r 2 ⎞⎤ Qr
    Er = − =− ⎢ ⎜⎜ ⎟⎟ ⎥ =
⎢⎣ 4 πε0 ⎝ R ⎠ ⎥⎦ 4 πε0 R  
3 3
dr dr

 
EJEMPLO  4.18.   El  potencial  eléctrico  debido  a  un  alambre  recto  infinito  cargado  uniformemente  con 
densidad de carga lineal l, en un punto a distancia “x” del alambre es 
 
λ
φ(x) = − ln(x)
   
2 πε0  
 
Calcular el campo eléctrico en el punto. 
‐‐‐‐‐‐‐‐‐‐‐‐‐‐‐‐‐‐‐‐‐‐‐‐‐‐‐‐‐‐‐‐‐‐‐‐‐‐ 
  Usando la fórmula (37), donde la variable “u”, medida a lo largo de la línea de fuerza, es en este 
caso “x”, tenemos 
 
dφ d ⎛ λ ⎞ λ
    Ex = − = − ⎜− ln(x) ⎟ =  
dx dx ⎝ 2 πε0 ⎠ 2 πε0 x
‐‐‐‐‐‐‐‐‐‐‐‐‐‐‐‐‐‐‐‐‐‐‐‐‐‐‐‐‐‐‐‐‐‐‐‐‐‐‐‐‐‐‐‐‐‐‐‐‐‐‐‐‐‐‐‐‐‐ 
 
  La relación inversa de la integral de línea (12)   
es  tema  del  cálculo  de  varias  variables,  fuera  del 
nivel  matemático  de  este  texto.  Sin  embargo, 
podemos  usar  algunas  ideas  del  cálculo  integral 
para encontrar una relación útil. 
  Vamos  a  efectuar  la  integral  de  línea  (12) 
tomando  como  trayectoria  de  integración  Γ  una 
línea  de  fuerza  (en  general  curva)  del  campo 
eléctrico.  Además  los  puntos  límites  de  la 
integración  serán  dos  puntos  P  y  S  separados  por 
una distancia mucho muy pequeña. 
  Veamos la Fig. 22. El punto  O es el origen de 
un sistema de coordenadas (no mostrado). Desde O   
Fig. 22 
trazamos los vectores de posición “rP” y “rS” de dos 
 
puntos  P  y  S  muy  próximos  situados  sobre  una 
línea de fuerza del campo E. 
4‐35 

 
  P y  S están tan cercanos uno del otro que el segmento  PS puede considerarse recto con muy buena 
aproximación, y además el campo  E puede considerarse constante sobre los puntos de este segmento. El 
vector separación entre P y S, o sea  Δr ≡ rS – rP, es de magnitud muy pequeña. 
  De acuerdo con (16) en la pág. 10, el voltaje entre los puntos P y S es 
 
S
(38)    V(P → S) = − ∫ P E • dr  
 
Ahora  bien,  dado  que  el  campo  se  supone  aproximadamente  constante  (en  magnitud  y  dirección), 
podemos sacarlo de la integral en la forma 
 

⎛ S ⎞
    V(P → S) ≈ −E • ⎜ dr ⎟ = −E • ( rS − rP ) = −E • Δr  
∫   (Δr ≡ rS – rP) 
⎝ P ⎠
 

En virtud de que E y Δr son paralelos, su producto escalar es el producto de sus magnitudes. Poniendo 
 

    |Δr| = Δs 
 

tenemos 
 

    V(P → S) ≈ – E Δs 
 

o bien, dado que el voltaje V es la diferencia de potencial φ(S) – φ(P) ≡ Δφ, obtenemos la relación 
 
    El campo eléctrico en un punto P es el negativo de la tasa de 
Δφ V variación  del  potencial  con  la  distancia  medida  a  lo  largo  y  en  el 
(39)    E≈− =−  
Δs Δs sentido  de  la  línea  de  fuerza a  través  de  P.  Alternativamente,  es el 
voltaje a través de P y otro punto muy cercano, situado en la misma 
línea de fuerza que P, dividido por la distancia entre esos puntos. 
 
  El  signo  negativo  de  (39)  expresa  que  la  dirección  el  campo  E  siempre  es  de  mayor  a  menor 
potencial. Antes de dar un ejemplo de aplicación de (39), conviene introducir otro concepto. 
4‐36 

 
EJEMPLO  4.19.   Las  superficies  equipotenciales  constituyen  un  modo  alternativo  muy  conveniente  de 
obtener una idea gráfica global del campo eléctrico. En la Fig. 4.14 se muestra una familia de superficies 
equipotenciales, formada por un conjunto de superficies donde el potencial toma valores fijos igualmente 
espaciados. 
 

 
 

Fig. 27 
 

Observe la línea de fuerza de campo eléctrico mostrada a través de los puntos P y S. Esta línea interseca 
perpendicularmente a todas las superficies equipotenciales de la familia. 
Para  calcular  aproximadamente  el  campo  en  el  punto  P,  consideramos  otro  punto  S  muy  cercano,  y 
aplicamos la ecuación (39). Por ejemplo, para los puntos P y S de la Fig. 27 tenemos que la magnitud del 
campo en P vale 
 
Δφ 45V − 50V V
    E≈ = = 2500     (dirigido de mayor a menor potencial) 
Δs 0.002m m
 
4‐37 

 
4.14. AUTOENERGÍA DE UNA DISTRIBUCIÓN DE CARGA. 
  En  la  Sección  4.8  discutimos  la  ley  de  conservación  de  energía  para  un  sistema  formado  por  un 
conjunto de 5 cargas puntuales fijas Q1, Q2, …, Q5, y adicionalmente otra carga puntual móvil Q6. Véase 
la Fig. 4.6, repetida a la derecha.   
  Encontramos  para  la  energía  total  de  tal 
sistema la Ec. (25), que repetimos aquí: 
 
1
[(25)]    E (P) =  m 6 v 62 + Q6 φ (P) 
2
 
donde  φ(P)  es  el  potencial  producido  por  las  5 
cargas  fijas  en  el  punto  P  donde  se  encuentre  la 
carga móvil Q6. 
  Ahora  bien,  ¿Cuál  sería  la  energía  total  de   
este sistema {Q1, Q2, …, Q5; Q6} si las 5 cargas Q1 a  [Fig. 4.6, pág. 4‐18] 
Q5 estuviesen también en movimiento (que sería  
como el movimiento de un cuerpo rígido, pues estas cargas están unidas por varillas rígidas aislantes)? 
  Es  más,  podríamos  liberar  la  condición  de  rigidez  y  permitir que  las  5  cargas  Q1,  …,  Q5, al  igual 
que la carga Q6, estuviesen todas en movimiento arbitrario. ¿Cuál sería la energía total del sistema de las 6 
cargas móviles en este caso? Vamos a investigar a continuación esta situación en general, para N cargas. 
  Tenemos  entonces  en  general  un  sistema  aislado  de  N  cargas  puntuales  Q1,  Q2,  …,  QN,  todas 
móviles, y deseamos encontrar la expresión de la energía total del sistema. 
  Una contribución a esta energía total proviene de la energía cinética de las cargas. Sean  m1, m2, …, 
mN las masas y  v1,  v2, …,  vN las velocidades de las cargas en un instante cualquiera. Entonces la energía 
cinética K es 
 
N
∑ 2 mi vi2  
1 1 1 1
(40)    K= m1v12 + m 2 v 22 + ... + m N v 2N =
2 2 2
i =1
 
  La otra contribución a la energía total proviene de la llamada autoenergía o energía de formación 
del sistema, que se define como sigue: 
 
    La autoenergía o energía de formación de un sistema de cargas puntuales 
  {Q1, Q2, …, QN}, en una configuración dada, es el trabajo que un agente externo necesita 
(41)  invertir  para  transportar  a  todas  las  cargas  cuasiestáticamente,  una  por  una,  desde  el 
  infinito  hasta  los  respectivos  puntos  que  ocuparán  en  esa  configuración.  Durante  este 
transporte, las cargas ya configuradas permanecen fijas. 
 
  Esta clase de energía dependerá entonces exclusivamente de las ubicaciones finales de las cargas, no 
así de sus velocidades. 
  Para obtener la autoenergía del sistema de cargas {Q1, Q2, …, QN}, realizamos el siguiente proceso:  
  Para traer la primera carga, Q1, desde el infinito hasta su posición final dentro del sistema, no es  
4‐38 

 
necesario  efectuar  ningún  trabajo  “W1”,  puesto  que  no  existe  campo  eléctrico  que  se  oponga  a  este 
transporte. Una vez puesta Q1 en su lugar final, permanecerá fija allí durante todo el proceso. Entonces 
 

    W1 = 0 
 

  Para  traer  la  segunda  carga,  Q2,  el  agente  externo  debe  realizar  un  trabajo  “W2”  contra  la  fuerza 
eléctrica de la carga  Q1 ya configurada. Pero, recordando la definición general de potencial eléctrico, este 
trabajo es precisamente igual a “Q2  φ1”, donde  φ1 es el potencial debido a la carga  Q1 en el punto donde 
se colocará Q2: Tenemos entonces 
 
Q1
    W2 = Q 2 φ1 = Q 2 ⋅  
4 πε0 r12
 

donde r12 es la distancia entre las cargas Q1 y Q2 en la configuración final. 
  Una vez que cada carga ha sido configurada, se imagina que permanece fija en su punto de destino. 
  Para  traer la tercera  carga,  Q3, debe  realizarse un  trabajo  W3  contra la  fuerza  eléctrica  del  campo 
generado por las dos cargas ya configuradas  Q1 y  Q2. Este trabajo es igual a “Q3  φ12”, donde  φ12 es el 
potencial eléctrico debido a Q1 y Q2 en el punto donde quedó Q3. Así pues, 
 
⎛ Q1 Q2 ⎞ 1 ⎛ Q1Q 3 Q2 Q3 ⎞
    W3 = Q3 φ12 = Q 3 ⋅ ⎜ + ⎟= ⎜ + ⎟
⎝ 4 πε0 r13 4 πε0 r23 ⎠ 4 πε0 ⎝ r13 r23 ⎠
 
 
  Ud capta la idea. El trabajo para traer la cuarta carga Q4 es 
 

1 ⎛ Q1Q 4 Q2 Q4 Q 3 Q4 ⎞
    W4 == ⎜ + + ⎟ 
4 πε0 ⎝ r13 r24 r34 ⎠
 

y asi sucesivamente, de tal manera que el trabajo total para traer las N cargas es 
 
    W = W1 + W2 + W3 + … + WN 
 

y este es la autoenergía del sistema de N cargas. 
 
N Autoenergía o energía de formación de un sistema 
Qi Q k

1
(42)    U=  
de N cargas puntuales Q1, Q2, …, QN.1 
4 πε0 rik
i,k =1
i ≠ k; k > i
 
  En  la  doble  suma  (42)  no  aparecen  términos  con  índices  iguales  (como  r22,  r44,  etc.),  ni  términos 
donde el segundo índice (k) es menor que el primero (i) (como r32, r65, etc.). 

                                                                  
1 A las cargas puntuales per se no se les adjudica ninguna autoenergía. Se consideran entidades ya formadas. 
4‐39 

 
  La energía total del sistema aislado {Q1, Q2, …, QN}  es 
 
  Energía total de un sistema aislado de 
N cargas eléctricas Q1, Q2, …, QN 
Qi Q k
∑ 2 m i vi2 + 4πε0 ∑
1 1
(43)    E = K + U  =  
rik Energía total = Energía cinética + 
i =1 i,k =1
i ≠ k;k > i Autoenergía  (o  Energía  potencial 
  eléctrica) 
 

  La ley de conservación de la energía total expresa que la energía total del sistema se conserva si el 
sistema de cargas está aislado. 
 
EJEMPLO  4.20.   Se  tiene  un  sistema  de  4  cargas  puntuales  situadas  en  los  vértices  de  un  rectángulo  de 
base “a” y altura “b”, como se muestra en la figura. Calcular la autoenergía eléctrica del sistema. 
 

 
Fig. 22 
 
  Poniendo 
 
    s = a 2 + b2  
 
tenemos, según la fórmula (42) 
 
1 ⎛ q( −2q) q ⋅ 5q q( −3q) ( −2q) ⋅ 5q ( −2q) ⋅ ( −3q) 5q( −3q) ⎞
    U= ⎜ + + + + + ⎟ 
4 πε0 ⎝ a s b b s a ⎠
 
(Nota. En un sistema con N cargas puntuales, el número de productos que abarca la suma es 
 
⎛N⎞ 1
    ⎜ 2 ⎟ = N(N − 1)  
⎝ ⎠ 2
 
En el ejemplo presente tenemos entonces 6 productos.) 
 
  Reduciendo obtenemos 
 
q2 ⎛ 17 11 13 ⎞
    U= ⎜− + − ⎟ 
4 πε0 ⎝ a s b⎠
4‐40 

 
EJEMPLO  4.20.   Se  tiene  un  sistema  de  5  cargas  puntuales  Q1,  Q2,  …,  Q5.  Las  primeras  3  cargas  están 
unidas entre sí mediante varillas ligeras rígidas, lo mismo que las dos últimas cargas (Fig. A). 
  Supongamos  que  las  cargas  están  en  movi‐  
miento  arbitrario  (ambos  subsistemas    {Q1,  Q2,  Q3}  y 
{Q4,  Q5}  se  moverían  por  hipótesis  como  cuerpos 
rígidos). 
  Aquí  tendríamos  que  las  autoenergías  de  estos 
subsistemas  (dadas  por  la  Ec.  (42))  se  mantendrían 
constantes  durante  el  movimiento,  puesto  que  las 
distancias  “rik”  entre  sus  cargas  constituyentes  no 
varían. 
  La  autoenergía  del  sistema  {Q1,  Q2,  Q3}  es,   
Fig. 32 
usando la fórmula (42): 
 
1 ⎛ Q1Q 2 Q1Q 3 Q2 Q 3 ⎞
    U123 = ⎜ + + ⎟  = constante 
4 πε0 ⎝ r12 r13 r23 ⎠
 

y la autoenergía del sistema {Q4, Q5} es 
 

1 ⎛ Q4 Q5 ⎞
    U45 = ⎜ ⎟  = constante 
4 πε0 ⎝ r12 ⎠
 

Dado que estas autoenergías son constantes, no tiene caso incluirlas en la ecuación de balance de energía, 
en la cual se consideran solamente cambios de energía entre dos configuraciones dadas. 
  La diferencia entre la autoenergía (o energía potencial eléctrica) del sistema completo de 5 cargas 
{Q1, Q2, …, Q5.} y la suma de las autoenergías de los subsistemas  {Q1, Q2, Q3}  y  {Q4, Q5 }, que es igual a 
 
1 ⎛ Q1Q 4 Q1Q 5 Q2 Q 4 Q2 Q 5 Q3 Q4 Q 3Q 5 ⎞
    Uint = ⎜ + + + + + ⎟ 
4 πε0 ⎝ r14 r15 r24 r25 r34 r35 ⎠
 
se denomina  energía potencial de interacción entre ambos subsistemas. Esta energía potencial  Uint es la 
que  está  variando  (junto  con  la  energía  cinética)  conforme  ambos  subsistemas  se  mueven.  Note  que  la 
energía Uint contiene solamente productos cruzados de las cargas de ambos sistemas {Q1, Q2, Q3} y 
{Q4, Q5}. 
  La  ley  de  conservación  de  la  energía  total  se  escribiría  en  este  caso  en  la  forma  (que  excluye 
términos constantes) 
 

    E = K123 + K45 + Uint 
 

donde K123 y K45 son las energías cinéticas de los subsistemas {Q1, Q2, Q3} y {Q4, Q5 }, respectivamente, y 

Uint  es la energía potencial de interacción entre ambos subsistemas. 
4‐41 

 
EJEMPLO 4.21.  Inicialmente, 4 cargas puntuales de valores 2q, q, 2q y 5q se encuentran en los vértices de 
un cuadrado de lado “a”. La carga “q” es libre de moverse y las otras 3 cargas están fijas. La carga “q” se 
deja ir desde el reposo y momentos después se halla en la posición indicada en la Fig. 4.3, donde posee 
velocidad  v. Suponga que las cargas están en el vacío, sin gravedad; calcular la velocidad  v de la carga 
móvil en términos de las distancias a las demás cargas. 
‐‐‐‐‐‐‐‐‐‐‐‐‐‐‐‐‐‐‐‐‐‐‐‐‐‐‐‐‐‐‐‐‐‐‐‐‐‐‐   
  Dada  la  simetría  del  sistema,  al  liberar  la 
carga  “q”,  su  movimiento  ocurre  a  lo  largo  de  la 
diagonal del cuadrado. 
  Ahora  bien,  el  sistema  de  las  4  cargas  posee 
cierta  energía  potencial  (referida  al  infinito)  en  la 
situación  inicial.  Tal  energía  vendría  dada  por  la 
expresión (42‐p38), o sea 
 
N
Qi Q k

1
U=  
4 πε0 rik
i,k =1
i ≠ k; k > i
   
 

Pero como las cargas 2q, 2q y 5q están fijas, la parte 
 
de  la  energía  que  puede  variar  es  la  energía  de 
Fig. 44 
interacción de la carga “q” con las demás cargas. En 
la situación inicial, esta energía de interacción es 
 
1 ⎛ q ⋅ 2q q ⋅ 2q q ⋅ 5q ⎞
    U(1)
int = ⎜ + + ⎟ 
4 πε0 ⎝ a a 2a ⎠
 

En la situación final, en que la carga móvil ha adquirido la velocidad v, la energía potencial de interacción 
(es decir, la energía potencial variable del sistema total) es 
 
1 ⎛ q ⋅ 2q q ⋅ 2q q ⋅ 5q ⎞
    U(2)
int = ⎜ + + ⎟ 
4 πε0 ⎝ s s d + 2a ⎠
 
  La velocidad v se calcula ahora de la ley de conservación de energía, que se expresa así: 
 
1
    int = U int +
U(1) (2)
mv 2  
2
4‐42 

 
4.15. AUTOENERGÍA DE UN CAPACITOR DE PLACAS PARALELAS. 
  Apliquemos  los  conceptos  introducidos  en  la  sección  anterior  para  calcular  la  autoenergía  de  un 
capacitor de placas paralelas, de carga “Q”, área “A” y distancia entre placas “d”. 
  En  la  situación  inicial  (figura  a  la  derecha,   
parte  de  arriba)  se  tiene  el  capacitor  sin  carga 
alguna. En esta situación su autoenergía U es cero. 
  En  la  situación  final  (parte  central  de  la 
 
figura) el capacitor ya adquirió su carga final  Q. Su 
 
autoenergía en esta situación es el trabajo necesario 
Capacitor sin carga (U = 0) 
para  colocar  esta  carga  en  sus  placas, 
 
transportándola  desde  la  placa  negativa  (que  es  el 
 
punto de referencia del potencial del capacitor). 
  Nos valdremos del siguiente artificio: 
  Vamos  a  irle  quitando  a  la  placa  inferior 
trozos  “dq”  de  carga  positiva,  distribuída  simétri‐
camente  en  un  plano,  la  cual  transportamos  y 
 
depositamos  en  la  placa  positiva  (Fig.  4.17,  abajo).   
Al  irle  quitando  cargas  positivas  “dq”  a  la  placa  Capacitor cargado (U ≠ 0) 
inferior motivaremos que esta placa tenga cada vez   
mayor  deficiencia  de  carga  positiva,  es  decir,  su   
carga  neta  total  se  irá  volviendo  cada  vez  más 
negativa.  Digamos  que  “–  q”  sea  el  valor  en  una 
etapa  arbitraria  del  proceso;  entonces  la  carga  ya 
depositada en la placa superior será “q”. 
  El campo eléctrico del capacitor en una etapa   
en que su carga es “q” es igual a   
 
Capacitor cargado con carga q 
σ q
    E= =    
ε0 Aε0
Fig. 28 
 

de  modo  que  el  trabajo  necesario  para  llevar  el  plano  de  carga  “dq”  hasta  la  placa  positiva,  contra  la 
fuerza de este campo, es  dW = fuerza × distancia = carga × campo × distancia, o sea 
 

q
    dW = dq ⋅ E ⋅ d = dq ⋅ ⋅d  
Aε0
 
Integrando  dW desde  q = 0 hasta  q = Q obtenemos el trabajo total, que viene siendo la autoenergía del 
capacitor cargado: 
 
Q2d
 
d Q
(44)    U=W=
Aε0 0
qdq =∫2Aε0
  Autoenergía de un capacitor cargado con carga Q 

 
Note que en términos del campo eléctrico del capacitor, 
 
4‐43 

σ
    E=  
ε0
 

tenemos que la autoenergía por unidad de volumen del capacitor, denominada densidad de energía, es 
 
   
U 1
(45)    u = = ε0 E 2   Densidad de energía del capacitor de placas paralelas. 
V 2 Unidades físicas: julio/(metro)3 = J/m3 
 
 
  La energía que el capacitor cargado almacena la cede al exterior al descargarse. 
 
EJEMPLO  4.22.   Calcular  la  autoenergía  de  un  capacitor  cuya  densidad  de  carga  es  σ  =  20  μC,  cuyo 
volumen es V = 4 cm3, y cuya distancia entre placas es d = 0.8 cm. 
  Usando la fórmula (44) tenemos 
 

Q2d
    U=  
2Aε0
 

Multiplicando numerador y denominador por el área A tenemos 
 

Q 2 Ad σ2 ⋅ V
    U= =  
2A 2 ε0 2ε 0
 
Q
donde hemos usado las relaciones  σ =  y V = volumen = A d. Sustituyendo valores numéricos, 
A
 
2
⎛ −6 C ⎞ −6 3
⎜ 20(10 ) 2 ⎟ ⋅ 4(10 )m
σ2 ⋅ V ⎝ m ⎠
    U= = = 9.035(10 −6 ) J
2 ε0 2
2 ⋅ 8.854(10 −12 )
C
N ⋅ m2
-----------------------------
4‐44 

 
4.16. CONDUCTORES Y POTENCIAL ELÉCTRICO. 
  En la Fig. 4.18 se representa un conductor cargado (digamos positivamente) y sus líneas de campo 
eléctrico E. Sabemos ya que para un conductor: 
• La carga excedente se localiza en su superficie. 
• Las líneas de fuerza en los puntos de la superficie del conductor son perpendiculares a la superficie. 
• El campo eléctrico en el interior del conductor es nulo. 
 

 
Fig. 29 
 
    Debido  a  estas  propiedades,  el  conductor  es  un  cuerpo  cuyos  puntos  se 
(46)  encuentran todos al mismo potencial eléctrico. En particular, la  superficie del conductor 
es una superficie equipotencial. 
 
  Para demostrar la afirmación anterior, usemos la definición general de potencial eléctrico. 
  Sea  A  un  punto  arbitrario  de  la  superficie  del  conductor.  Tomando  el  punto  de  referencia  en  el 
infinito, tenemos que el potencial en A viene dado por la integral (12), a saber, 
 
A
(47)         φ(A) = − ∫
Γ ∞
E • dr  
 

donde Γ es una trayectoria de integración arbitraria desde el infinito hasta el punto A. 
  Ahora bien, el potencial en cualquier punto interno, como el C mostrado en la figura, se obtendría 
extendiendo la integral (47) hasta el punto C. Pero en la parte de  Γ que va de A a C la integral sería nula, 
ya que el campo E es cero en el interior del conductor. Por tanto, el potencial en C es el mismo que en A. 
  Si extendemos ahora la trayectoria Γ desde A hasta un punto cualquiera B de la superficie, añadién‐
dole un tramo contenido en dicha superficie, obtendríamos análogamente que  φ(B) = φ(A), puesto que 
sobre la superficie el campo E es perpendicular al vector “dr” y E • dr = 0. 
4‐45 

 
4.18. PROBLEMAS. 
1. Calcular el potencial de un disco de radio “a” y carga uniforme Q, en cualquier punto P sobre su eje de 
simetría perpendicular: 
(a) Usando la integral (29) de la página 4‐21, junto 
con la expresión del campo  E del disco sobre su eje 
de simetría perpendicular. 
(b) Usando la integral (33) de la página 4‐24. 
 
σ ⎛ 2 2 ⎞
Resp.    φ( z ) = ⎜ a + z − z ⎟ + const 
2εo ⎝ ⎠

 
2. Calcular el potencial debido al cuadrupolo mostrado en la figura, en los puntos de su eje longitudinal. 
La distancia entre carga y carga es “a”. 

 
 

Resp. 
 
3. Un cascarón esférico posee radio R y carga uniforme Q. Calcular el potencial debido al cascarón en 
todo  el  espacio.  Refiera  el  potencial  al   
infinito. 
  ¿A  qué  potencial  estará  la  superficie 
del cascarón cuando aloja una carga eléctrica 
de 10−8 C? Tome R = 100 mm. 
Resp. 
kQ
  φ=   r ≥ R 
r
kQ
  φ=   r ≤ R 
R
  900 V 
 

4. Una gota esférica de agua porta una carga eléctrica de  3 x 10−6 C y el potencial en su superficie es de 
500V. ¿Cuál es el radio de la gota?  
  Si dos de estas gotas, de la misma carga y radio, se combinan para formar una sóla gota esférica, 
¿cuál es el potencial en la superficie de la gota así formada? 
Resp.  
 

5. Si la superficie de la Tierra poseyera una carga neta equivalente a la de un electrón por metro cuadrado, 
¿cuál  sería  el  potencial  eléctrico  en  la  superficie?  ¿Cuánto  valdría  el  campo  eléctrico  de  la  Tierra  en  su 
superficie? 
Resp. −0.12V; 1.8 x 10−8 V/m. 
4‐46 

 
6.  En  condiciones  ambientales  normales  el  campo  eléctrico  de  la  Tierra  varía  con  la  altura  z  sobre  la 
superficie terrestre de acuerdo con la fórmula empírica 
   
  E = −E 0 a e(−α z + be −β z
)
k   
 
 
donde  k es el vector unitario cartesiano, y  E0, 
a, α, b y β son constantes con los valores 
 
V
E0 = 130 m ; a = 0.69; b = 0.31; 
 
 
1 1
α = 3.5 km ; β = 0.23 km  
 
Calcular el potencial φ del campo eléctrico de la Tierra refiriéndolo a su superficie. Calcular los valores del 
potencial para alturas de 0, 1 y 10 km. 
Resp. 
 
7.  Una  placa  infinita  de  grosor  “w”   
porta densidad de carga uniforme ρ. 
Calcular el potencial eléctrico en todo 
el  espacio.  Use  el  plano  medio  de  la 
placa como referencia del potencial. 
(Use  las  expresiones  ya  encontradas 
para el campo eléctrico: 
 
ρz ρw
  E= ; E= )   
ε0 2ε 0
). 
ρz 2 ρw
Resp.     φ(z) = − φ(z) = (w − 4z)  
2ε0 8ε0
 
8. Calcular el potencial eléctrico en el centro de un   
cascarón  semiesférico que  porta  carga  uniforme  de 
10−8 C y cuyo radio es de 0.4 m. 
(Sugerencia.  Use  el  hecho  de  que  el  potencial  se 
superpone  escalarmente.  Considere  la  integral  (33) 
de la página 4‐24, que resulta trivial en este caso).   
Resp. 225 V. 
4‐47 

 
 
10. Dos hilos rectos infinitos paralelos y separados 
una distancia L portan densidades de carga lineal 
constantes iguales a λ y −λ.  
Tomando  los  ejes  como  se  propone  en  la  figura, 
calcular  el  potencial  eléctrico  de  esta  distribución 
en  los  puntos  del  plano  XY  entre  los  dos  hilos. 
Tomar el punto de referencia en el punto x = x0. 
Resp. 
 

λ ⎛ x(L − x) ⎞
  φ(x, y) = − ln ⎜ ⎟ 
2 πε0 ⎝ x0 (L − x0 ) ⎠
(x ≠ 0; x ≠ L) 

 
11. La medición del potencial entre   
los  electrodos  de  cierto  tubo  de 
descarga  arroja  el  resultado  que  se 
muestra en el gráfico como función 
de la distancia al electrodo positivo. 
Usando  esta  curva,  obtener  los 
valores  del  campo  eléctrico  en  los 
puntos  a  distancias  de  4,  10  y  12.2 
cm. 
Resp.  En  el  punto  a  4  cm  del 
electrodo positivo se tiene 
E ≈ 33 V/cm. 

 
12.  La  figura  muestra  algunas  superficies 
equipotenciales  de  un  campo  eléctrico  en 
escala real de longitud. 
Bosqueje  las  líneas  de  fuerza  del  campo 
eléctrico que pasan por los puntos P y S. Use 
una  regla  y  calcule  aproximadamente  el 
campo eléctrico en estos dos puntos. 
 
Resp. 
 

 
4‐48 

 
12.. El campo eléctrico debido a un cilindro sólido infinito de radio “a” y densidad de carga uniforme  ρ 
viene dado por 
   
⎧ ρr Para r ≤ a 
⎪ 2ε
⎪ 0
  Er (r) = ⎨ 2  
⎪ ρa Para r ≥ a 
⎪ 2ε 0 r

 
Este campo está graficado a la derecha. 
  Calcular  el  potencial  eléctrico  en  un 
punto P dentro del cilindro, a distancia  rP del 
eje. Tome el punto de referencia  P0 en la línea 
de fuerza que pasa por P, y a una distancia  r0 
 
del eje.   
Resp.   
 

ρ ⎛ 2 ⎛a ⎞ r2 a2 ⎞
  φ(r) = − ⎜ a ln ⎜ ⎟+ − ⎟ 
2ε0 ⎜⎝ ⎝ r0 ⎠ 2 2 ⎟⎠
 

o bien, quitando términos constantes, 
 

ρ ⎛ r2 ⎞
  φ(r) = − ⎜ ⎟      (φ(0) = 0)   
2ε0 ⎜⎝ 2 ⎟
⎠  
 
 
4‐49 

 
4.13.  Calcular  la  autoenergía  de  una  esfera  sólida  de  radio  R  cargada  uniformemente  con  carga  total  Q 
mediante  el  siguiente  procedimiento,  análogo  al  que  utilizamos  en  la  Sección  4.17  para  calcular  la 
autoenergía del capacitor de placas paralelas: 
  Supongamos  que  ya  hemos  formado  una  esfera  sólida  de  radio  r  <  R  y  carga  q  distribuída 
uniformemente (figura a la derecha). Esta es una etapa intermedia del proceso. 
  Añadamos  a  dicha  esfera  una  carga  “dq”,  traída  desde  el  infinito  y  distribuída  en  un  cascarón 
esférico  muy  delgado  (de  grosor  “dr”).  Dada  la  simetría  esférica  del  campo  de  la  esfera  de  radio  r,  el 
trabajo necesario para depositar la carga dq sobre esta esfera es 
   
  dW = (carga dq) × (potencial eléctrico 
    debido a la esfera de radio r en 
    su superficie) 
 
Encuentre  una  relación  entre  las  variables  “r”  y 
“q”,  usando  la  condición  de  igualdad  de  las 
densidades  de  carga  de  las  esferas  de  radios  “r”  y 
“R”, y obtenga de allí “dq” en términos de “dr”. 
Integre dW desde r = 0 hasta r = R. 
Resp. 
 
3kQ2
    U=  
5R
 
 
5‐1 

  CAPÍTULO 5 
   
  CAPACITORES Y CAPACITANCIA 
 
 
 
5.1. INTRODUCCIÓN. 
  El capacitor es un componente muy importante en los circuitos de corriente eléctrica. Su capacidad 
de almacenar y suministrar energía eléctrica los vuelve muy útiles. Existen muchos tipos de capacitores, 
incluyendo los que se usan en los circuitos integrados de pequeño tamaño. Sus características eléctricas y 
las leyes que los rigen son muy simples. 
  En  este  capítulo  introduciremos  los  conceptos  generales  de  capacitor  (antiguamente  llamado 
condensador)  y  capacitancia  (antiguamente  llamada  capacidad).  Lo  haremos  solamente  para  los  tipos  de 
capacitores más simples. 
 
5.2. DEFINICIÓN DE CAPACITOR Y CAPACITANCIA. 
  Un  capacitor  es  un  dispositivo  compuesto  de  dos  conductores,  uno  de  los  cuales  está  cargado 
positivamente (con carga total +Q), y el otro negativamente (carga total –Q). 
  Este par de conductores producen en el espacio circundante un campo eléctrico  E. En la Fig. 1 se 
representa un capacitor arbitrario y su campo eléctrico. 
 

 
Fig. 1 
 
  El  campo  eléctrico  E  tiene  un  campo  de  potencial  eléctrico  φ  asociado.  En  el  caso  de  la  Fig.  1, 
tomemos  el  punto  de  referencia  del  potencial  en  el  infinito,  y  sean  φ1  y  φ2  los  potenciales  de  los 
conductores  negativo  y  positivo,  respectivamente.  El  voltaje  V  entre  ambos  conductores  (o  voltaje  a 
través del capacitor) es igual a 
5‐2 

    V = φ2 – φ1 
 

  En términos de este voltaje y de la carga del conductor positivo se define la capacitancia: 
 
   
Q La capacitancia  C del capacitor se define como el cociente de la carga 
(1)    C=  
V del capacitor (Q) y el voltaje a través del mismo (V). 
 
 
  De (1) sacamos las unidades S.I. de capacitancia, a saber 
 
coulombio
    ≡ faradio = F  
voltio
 
  En los circuitos eléctricos, el símbolo gráfico del capacitor es el siguiente (Fig. 2): 
 

 
Fig. 2 
 
  Se  denomina  placa  del  capacitor  a  cualquiera  de  los  dos  conductores  que  lo  forman, 
independientemente  de  su  forma.  Si  se  conectan  las  placas  de  un  capacitor  sin  carga  a  una  fuente  de 
voltaje, el capacitor se carga. La fuente suministra carga +Q al conductor conectado a su polo positivo, y 
carga –Q al otro conductor (Fig. 3). 
 

 
Fig. 3 
 
  Los conductores que forman el capacitor son superficies equipotenciales: todos los puntos de cada 
conductor,  así  como  los  del  alambre  que  lo  conecta  con  la  fuente  de  voltaje,  se  encuentran  a  un  mismo 
potencial. 
5‐3 

 
5.3. EL CAPACITOR DE PLACAS PARALELAS. 
  Uno de los capacitores de más simple estructura es el capacitor de placas paralelas. Consta de dos 
placas  metálicas  de  áreas  iguales,  “A”,  separadas  una  distancia  “d”  (Fig.  4).  Por  simplicidad  se  supone 
que las dimensiones de las placas son mucho mayores que la distancia entre ellas, de tal manera que con 
buena aproximación el campo se puede considerar constante dentro del capacitor y nulo fuera de él. 
 

 
Fig. 4 
 

  Calculemos la capacitancia de este capacitor. Usaremos las relaciones aplicables al mismo: 
 
σ Q
    V = Ed    E=     y    σ=  
ε0 A
 

donde V es el voltaje a través de las placas y σ es la densidad de carga del capacitor. 
  Encontramos 
 
Q Q Qε0 QAε0 Aε0
    C= = = = =  
V Ed σd Qd d
 
 
 
Aε 0 Capacitancia de un capacitor de placas paralelas de área de placas “A” 
(2)    C=   y distancia entre placas “d”. 
d
 
 

  Notemos  que  la  capacitancia  no  depende  de  la  carga  del  capacitor,  sino  de  puros  factores 
geométricos (el área de las placas y la distancia de separación entre ellas). Lo mismo ocurrirá para otros 
tipos de capacitores, como veremos. 
 
EJEMPLO 5.1. Un capacitor de placas paralelas tiene un área de 5 cm2 y una distancia entre placas de 
0.5 cm. Calcular su capacitancia. 
 
N ⋅ m2
5(10 −4 )m 2 ⋅ 8.854(10 −12 )
Aε 0 C2
    C= = = 8.85(10 −13 )F  
d 0.5(10 −2 )m
5‐4 

 
A  la  vista  de  este  resultado  vemos  que  el  faradio  es  una  unidad  de  capacitancia  muy  grande.  Es  más 
común usar submúltiplos como 
 

    1 microfaradio = 1 μF = 10–6 F    1 picofaradio = 1 pF = 10–12 F 
 
La capacitancia calculada arriba se expresaría como 
 
    C = 0.885 pF 
 
EJEMPLO 5.2. El campo eléctrico de un capacitor de placas paralelas vale E = 2000 V/m, y su carga es 
Q = 4 (10–10) C. Calcular la distancia entre las placas sabiendo que la capacitancia vale C = 60 pF. 
   De la fórmula de la capacitancia del capacitor de placas paralelas, 
 
Aε 0
    C=  
d
 
despejamos la cantidad pedida: 
 
Aε 0
    d=  
C
 
Para calcular el área A usamos las relación 
 
σ Q Q
    E= = ⇒ A=  
ε 0 Aε 0 Eε 0
 
Entonces, en unidades S.I., 
 
Aε 0 Q ε0 Q 4(10 −10 )
    d= = ⋅ = = =  0.033 = 3.3             (cm) 
C Eε0 C EC 200 ⋅ 6(10 −11 )
 
 
5‐5 

5.4. EL CAPACITOR ESFÉRICO. 
  El capacitor esférico consta de dos cascarones esféricos concéntricos de radios menor “a” y mayor 
“b”. Se representa en la Fig. 5. Esta configuración ya la estudiamos en el Capítulo 4, Sección 4.10, página 
4‐26. 
 

 
Fig. 5 
 
  El cascarón interior puede ser también una esfera metálica sólida, cuya carga eléctrica residiría en 
su superficie. 
  El voltaje a través del capacitor ya fue calculado (Ec. (32) del Capítulo 4): 
 

⎛1 1⎞
    V = kQ ⎜ − ⎟  
⎝a b⎠
 

de tal manera que la capacitancia de este capacitor es 
 
Q Q
    C= =  
V ⎛ 1 1⎞
kQ ⎜ − ⎟
⎝a b⎠
 
   
ab Capacitancia de un capacitor esférico de radios menor “a” y 
(3)    C=  
k(b − a) mayor “b”. “k” es la constante de la ley de Coulomb (= 1/4πε0) 
 
 
De nuevo, esta capacitancia depende solamente de factores geométricos: los radios de los cascarones. 
  Como ejemplo, un capacitor esférico de radios a = 5 cm y b = 10 cm tendría una capacidad de 
 
0.05 ⋅ 0.10
    C= = 11.1 (pF)  
9(109 )(0.10 − 0.05)
5‐6 

5.5. EL CAPACITOR CILÍNDRICO. 
  El  capacitor  cilíndrico  consta  de  dos   
cascarones cilíndricos concéntricos de radios menor 
R1  y  mayor  R2,  como  se  muestra  en  la  Fig.  6.  El 
cilindro  interior  puede  ser  un  cilindro  metálico 
sólido cuya carga reside en su superficie. 
  El  campo  eléctrico  en  el  espacio  entre  los 
cascarones  apunta  ortoaxialmente  hacia  fuera.  En 
un  punto  arbitrario  cuya  distancia  al  eje 
longitudinal central del cascarón interior sea “r”, el 
campo vale (Fig. 7) 
 
λ
    Er =  
2 πε0 r
   
(Se  supone  aquí  que  la  altura  “h”  del  capacitor  es  Fig. 6 
mucho mayor que los radios R1 y R2).   
 
  Recordemos  que  las  líneas  de  fuerza  de  este 
campo  son  rectas  perpendiculares  al  eje 
longitudinal,  y  contenidas  en  familias  de  planos 
perpendiculares a este eje. 
  Tomando el punto de referencia del potencial 
en  r  =  R2,  tenemos  para  el  potencial  (aplicando  la 
fórmula (29) del Capítulo 4, página 4.24): 
 
r λ r dr
    φ(r) = −
R2∫Er dr = −
2πε0 R 2 r ∫  

λ ⎛ r ⎞  
      =− ln ⎜ ⎟ 
2 πε0 ⎝ R 2 ⎠ Fig. 7 
 

  Por tanto, el voltaje entre los cascarones es 
 

λ ⎛R ⎞ Q ⎛R ⎞
    V = φ(R1 ) − φ(R 2 ) = ln ⎜ 2 ⎟ = ln ⎜ 2 ⎟  
2 πε0 ⎝ R1 ⎠ 2 πε0 h ⎝ R1 ⎠
 

 donde hemos puesto  λ = Q/h. Obtenemos así la capacitancia C = Q/V: 
 

   
2 πε0 h Capacitancia del capacitor cilíndrico de radios R1 y R2 y 
(4)    C=  
⎛ R2 ⎞ altura “h”. 
ln ⎜ ⎟
⎝ R1 ⎠
 
 
5‐7 

5.6. CAPACITORES EN SERIE O EN PARALELO. 
  Cuando dos capacitores  C1 y  C2 se conectan de modo que tengan la misma carga, la conexión se 
denomina “en  serie”  (Fig.  8).  Si  la  conexión  es  tal  que  el  voltaje a  través  de  cada  capacitor  es  el  mismo, 
entonces se trata de una conexión “en paralelo” (Fig. 9). 
   

   
   
Fig. 8  Fig. 9 
 
  En  la  Fig.  8,  el  polo  positivo  de  una  batería  que  suministra  un  voltaje  V  se  conecta  mediante  un 
alambre metálico a la placa superior del capacitor  C1. El polo negativo se conecta a la placa inferior del 
capacitor  C2. Esta acción suministra carga positiva a la placa superior de  C1, e igual carga negativa a la 
placa inferior de C2. 
  Tomemos  nuestra  referencia  de  potencial  (φ  =  0)  en  el  polo  negativo  de  la  batería.  Entonces  el 
potencial del polo positivo será “V”, al igual que el potencial de cualquier punto del alambre conductor 
que  conecta  dicho  polo  con  la  placa  superior  de  C1.  Todo  punto  de  esta  placa,  de  carga  positiva  Q, 
también está al mismo potencial V. La razón es que el polo positivo de la batería, junto con el alambre que 
lo conecta con la placa superior de  C1, así como dicha placa, forman un solo cuerpo metálico que, como 
sabemos, es un cuerpo equipotencial. 
    Análogamente, el polo negativo de la batería, el trozo 
de  alambre  que  lo  conecta  con  la  placa  inferior  de  C2,  y 
dicha  placa,  forman  un  solo  cuerpo  metálico  y  se 
encuentran al mismo potencial φ = 0. 
  Por  otra  parte,  el  trozo  de  alambre  que  conecta  la 
placa  inferior  de  C1  con  la  placa  superior  de  C2,  incluídas 
dichas  placas,  forman  un  solo  cuerpo  metálico  cuyo 
potencial supondremos que es algún valor“φN”. 
  Por lo tanto, el voltaje a través del capacitor C1 es  
    V1 = V – φN, 
 
  y a través del capacitor C2 es 
Fig. 10      V2 = φN – 0 = φN. 
  Dado que ambos capacitores tienen la misma carga Q 
tenemos, por la definición de capacitancia, 
 
5‐8 

Q Q
(5)    C1 = y C2 =  
V1 V2
 

  Ahora  bien,  deseamos  sustituir  los  dos   


capacitores  C1 y  C2 por uno sólo, como se muestra 
en  la  Fig.  11,  de  tal  manera  que  la  carga 
suministrada a él por la batería sea la misma que la 
suministrada a los capacitores  C1 y  C2, o sea “Q”. 
¿Cuánto  debe  valer  la  capacitancia  Ceq  de  este 
único capacitor? Veamos.   
Fig. 11 
  Dado que el voltaje a través de este capacitor 
único es V, y su carga es Q, podemos poner 
 
Q
(6)    Ceq =  
V
 

Luego, de las relaciones ya dadas  V1 = V – φN  y  V2 = φN.  tenemos que 
 

(7)    V = V1 + V2 
 

Combinando (7) con (5) y (6) deducimos la relación 
 

  Para  capacitores  en  serie,  el  recíproco  de  la  capacidad 


1 1 1 equivalente  es  la  suma  de  los  recíprocos  de  las  capacitancias 
(8)    = +  
Ceq C1 C 2 individuales. Este resultado se puede generalizar a más de dos 
  capacitores. 
 

  Pasemos ahora a investigar la conexión en paralelo de capacitores (Fig. 12). Primeramente notemos 
que en esta clase de conexión el voltaje a través de cada capacitor es el mismo, V. La carga que 
adquieren, sin embargo, es distinta en general.   
  Son válidas las relaciones 
 
Q1 Q2
(9)    C1 = y C2 =  
V V
 

y  la  carga  suministrada  por  la  batería  es  en  este 


caso 
 

(10)    Q = Q1 + Q2 
 

Formulamos  ahora  la  misma  pregunta  que  en  el 


caso  de  capacitores  en  serie.  Si  C1  y  C2  se 
 
sustituyen  por  un  único  capacitor  de  capacitancia  Fig. 12 
Ceq, ¿Cuánto debe de valer ésta para que la batería  
siga suministrando la misma carga Q? Note que esta situación es la misma que la de la Fig. 11. 
5‐9 

  Tendríamos entonces la condición 
 
Q
(11)    Ceq =  
V
 
  Combinando las relaciones (9), (10) y (11) hallamos 
 
  Para capacitores en paralelo, la capacidad equivalente es la 
(12)    Ceq = C1 + C2  suma  de  las  capacitancias  individuales.  Este  resultado  se 
puede generalizar a más de dos capacitores. 
 
 
EJEMPLO  5.3.   Se  conectan  en  serie  dos  capacitores  de  capacitancias  C1  =  8  pF  y  C2  =  2  pF.  ¿Cuál  es  la 
capacitancia equivalente de esta combinación? 
  De la Ec. (8) tenemos (en unidades de “pF”) 
 
1 1 1 1 1 5
    = + = + =     (1/pF) 
Ceq C1 C 2 8 2 8
 
de donde 
 
8
    Ceq = pF = 1.6 pF  
5
 
La  capacitancia  equivalente  en  paralelo  siempre  es  menor  que  cualquiera  de  las  capacitancias 
individuales. 
  Si estos capacitores se hubiesen conectado en paralelo, la capacitancia equivalente sería 
 
    Ceq = C1 + C2 = 8 pF + 2 pF = 10 pF 
 
La capacitancia en paralelo siempre es mayor que cualquiera de las capacitancias individuales. 
‐‐‐‐‐‐‐‐‐‐‐‐‐‐‐‐‐‐‐‐‐‐‐‐‐‐‐‐‐‐‐‐‐‐‐‐‐‐ 
5‐10 

EJEMPLO 5.4.  Cinco capacitores están conectados a una batería de 120 V como se muestra en la Fig. 13. Se 
desea sustituir estos 5 capacitores por uno sólo (Fig. 14), tal que la batería siga suministrando a este único 
capacitor la misma carga que suministra en la conexión mostrada. 
(a) ¿Cuál es la capacitancia equivalente de los 5 capacitores mostrados? 
(b) ¿Cuál es la carga suministrada por la batería? 

   

Fig. 13  Fig. 14 
 
(a)  Los capacitores de 3 pF y 6 pF están conectados en serie. Lo mismo ocurre separadamente con los 
capacitores de 4 pF y 12 pF. La capacitancia equivalente de los dos primeros es 
 
3 ⋅ 6 18
    Ceq1 = = =2    (pF) 
3+6 9
 
y la de los dos últimos es 
 
4 ⋅ 12 48
    Ceq2 = = =3    (pF) 
4 + 12 16
 
Sustituyendo  capacitores,  la  conexión  queda  ahora 
como se muestra en la Fig. 15. 
  Vemos  ahora  que  el  voltaje  a  través  de  los 
capacitores de 3 pF y 2 pF es el mismo, de modo que se 
tiene  una  conexión  en  paralelo  de  ambos.  Se  pueden 
sustituir  por  un  solo  capacitor  de  capacitancia  igual  a  
 
Ceq3  =  3  pF  +  2  pF  =  5  pF.  Este  último  capacitor 
Fig. 15 
quedará luego conectado en serie con el de 7 pF. 
  El capacitor equivalente a los 5 dados tiene entonces capacitancia igual a 
 
7 ⋅ 5 35
    Ceq = = =  2.9  (pF) 
7 + 5 12
 
  Las Figs. 16 y 17 muestran las dos últimas reducciones. 
5‐11 

 
 

 
   
 
Fig. 16  Fig. 17 
 
(b)  La carga suministrada por la batería se calcula de la relación 
 
Q
    Ceq =  
V
 
y resulta igual a 
 
    Q = Ceq V = 2.9 pF ⋅ 120 V = 348 pC = 3.48 (10–10) C 
 
EJEMPLO 5.5.  Demostrar que la energía eléctrica almacenada en un capacitor de placas paralelas, dada 
por la Ec. (44) de la página 4‐42, a saber, 
 

Q2d
    U=  
2Aε0
 
se puede poner también en cualquiera de estas dos formas: 
 
1 1 Q2 Energía  potencial  eléctrica  almacenada  en  un 
(13)    U = CV 2       U=  
2 2 C capacitor de placas paralelas. 
 
‐‐‐‐‐‐‐‐‐‐‐‐‐‐‐‐‐‐‐‐‐‐‐‐‐‐‐‐‐‐‐‐‐‐‐‐‐‐‐‐‐‐‐‐‐‐‐‐‐‐‐‐‐‐‐ 
  Usando las relaciones válidas para el capacitor de placas paralelas: 
 
Q σ Aε 0
    σ= , E= , V = Ed, C=  
A ε0 d
 
junto con la definición de capacitancia, 
 
Q
    C=  
V
 

se llega fácilmente a las relaciones (13). 
5‐12 

 
EJEMPLO 6.5.  Un capacitor de 5  μF se carga conectándolo   
a  una  fuente  de  voltaje  de  120  V.  Seguidamente  se 
desconecta  de  la  fuente.  Separadamente  se  hace  lo  mismo 
con un capacitor de 3  μF, y se tiene la situación de la Fig. 
18,  donde  los  interruptores  S1  y  S2  están  inicialmente 
abiertos.  Ahora  se  cierran  ambos  interruptores.  ¿Cuál  es 
entonces el voltaje a través de los capacitores? 
‐‐‐‐‐‐‐‐‐‐‐‐‐‐‐‐‐‐‐‐‐‐‐‐‐‐‐‐‐‐‐ 
  Primeramente  calculemos  las  cargas  de  los 
capacitores  en  la  situación  inicial  de  la  Fig.  18.  Usando  la 
definición  de  capacitancia,  C  =  q/V,  tenemos  para  el 
capacitor superior,   
Fig. 18 
    q1 = C1 V = 5 μF ⋅ 120 V = 600 μC 
 

Para el capacitor inferior, 
 

    q2 = C2 V = 3 μF ⋅ 120 V = 360 μC 
 

(Observe que la placa izquierda del capacitor superior tiene más carga negativa que carga positiva tiene la 
placa izquierda del capacitor inferior.) 
  Entonces la carga total de las placas izquierdas de ambos capacitores es inicialmente 
 

    – q1 + q2 = – 500 μC + 360 μC = – 140 μC 
 

  Al cerrar los interruptores, la carga total de las placas 
izquierdas  (y  derechas)  debe  conservarse,  es  decir, 
poniendo  q’1  y  q’2  iguales  a  las  cargas  finales  de  estas 
placas, tendremos (Véase la Fig. 19) 
 

    q’1 + q’2 = 140 μC 
 

Sea V’ la diferencia de potencial entre los puntos “a” y “b”, 
la  cual  existe  también  a  través  de  cada  capacitor.  De  las 
relaciones 
 

    q’1 = C1 V’      y       q’2 = C2 V’ 
 
tenemos 
  Fig. 19 
     q’1 + q’2 = (C1 + C2) V’ 
 

    140 μC =  (5 μF + 3 μF) V’ 
 

de donde 
 

    V’ =140 μC/8 μF = 17.5 V 
5‐13 

 
EJEMPLO 6.5.  Se tiene el arreglo de capacitores conectados a una fuente de voltaje de 160 V, tal como se 
muestra en la Fig. 20. (a) Calcular la carga suministrada por la fuente. (b) Calcular el voltaje a través de los 
capacitores C1 y C2. 
 

 
 

Fig. 20  Fig. 21 
 
(a)  Vamos a reducir el sistema a lo que vemos en la Fig. 21: una sola capacitancia equivalente Cequ. Los 
capacitores  C4  y  C5  están  en  serie;  luego,  su  capacitancia  equivalente  C45  está  en  paralelo  con  C3,  y 
finalmente, el correspondiente capacitor equivalente C453 está en serie con C1 y C2. Tenemos 
 
1 1 1 1 1 3 10
    = + = + = C 45 = pF  
C 45 C 4 C 5 10 5 10 3
 
10 22
    C 453 = C 45 + C 3 = +4=  
3 3
 
1 1 1 1 1 1 3
    = + + = + + = 0.428 Cequ = 2.34 pF  
Cequ C1 C 2 C453 6 8 22
 

  La carga suministrada es 
 
    Q = Cequ V = 2.34 pF ⋅ 160 V = 374.4 pC  
 

(b)  En la Fig. 20 vemos que la batería suministra la misma carga Q a los capacitores C1 y C2, es decir, 
 
    Q1 = 374.4 pC       Q2 = 374.4 pC 
 
Aplicando ahora la relación C = Q/V a estos dos capacitores obtenemos su voltaje: 
 
    V1 = Q/C1 = 374.4 pC/6 pF = 62.4 V 
 
    V2 = Q/C2 = 374.4 pF/8 pF = 46.8 V 
 
 
5‐14 

 
5.7. CAPACITOR CON DIELÉCTRICO. 
  Hasta  ahora  hemos  supuesto  que  no  existe  materia  física  entre  las  placas  del  capacitor.  Las 
capacitancias que hemos calculado valen “para el vacío”. 
  Al  introducir  un  material  no  conductor  que  llene  completamente  el  espacio  entre  las  placas,  se 
altera  la  capacitancia  del  capacitor.  Tales  materiales,  llamados  dieléctricos,  poseen  una  propiedad  que 
mide este efecto, denominada  constante dieléctrica, denotada con “κ”. Sea C0 la capacitancia en el vacío, 
y Cd la misma con el dieléctrico; la constante dieléctrica se define por 
 
 
Constante  dieléctrica.  C0  es  la  capacitancia  para  el 
Cd
(14)    κ=  
vacío, Cd la capacitancia con dieléctrico. 
C0
 

Se trata de una constante adimensional (es un número puro). 
  Si  el  dieléctrico  está  sometido  a  un  campo  eléctrico  muy  intenso  dentro  del  capacitor,  puede  volverse 
conductor y producir una descarga del capacitor. Este campo se denomina la resistencia dieléctrica del material. En la 
tabla siguiente se muestran valores de la constante y la resistencia dieléctrica de algunos materiales. 
 
Dieléctrico  Constante  Resistencia 
dieléctrica κ  dieléctrica (106 V/m) 
Aire  1.00059  3 
Mylar  3.2  7 
Nylon  3.4  14 
Papel  3.7  16 
Poliestireno  2.56  24 
Porcelana  6  12 
Vidrio pirex  5.6  14 
Teflón  2.1  60 
 
  Note que la constante dieléctrica del aire (κ = 1.00059) es para muchos efectos prácticos igual a la del vacío 
(κ = 1).  
 
EJEMPLO 5.6. Un capacitor de placas paralelas está construído con placas de área 5 cm2, separadas 0.5 mm, y lleno 
con porcelana. ¿Cuál es el voltaje máximo que puede soportar este capacitor? 
  Calculemos primeramente la capacitancia. De la fórmula (14) tenemos, en unidades S.I., 
 

Aε 0 5(10 −4 ) ⋅ 8.85(10 −12 )


    Cd = κC0 = κ =6 = 53 (pF)  
d 0.5(10 −3 )
 

Usando el valor dado en la tabla anterior para la resistencia dieléctrica para la porcelana obtenemos 
 
    Vmáx = E máx ⋅ d = 12(10 6 ) ⋅ 5(10 −3 ) = 6 000 (V)  
‐‐‐‐‐‐‐‐‐‐‐‐‐‐‐‐‐‐‐‐‐‐‐‐‐‐‐‐‐‐‐‐‐‐‐‐‐‐ 
5‐15 

 
  Existe  un  modelo  atómico  muy  simple  para  explicar  el  efecto  del  dieléctrico.  Supongamos  que  los  átomos  o 
moléculas del dieléctrico poseen cierto momento dipolar, esto es, que sus centros de carga positivos y negativos no 
coinciden. Al introducir el dieléctrico en el capacitor, los momentos dipolares tienden a orientarse en la dirección del 
campo eléctrico. La Fig. 18 muestra esquemáticamente lo que ocurre: los óvalos mostrados allí representan moléculas 
con sus momentos dipolares orientados en la dirección del campo eléctrico E del capacitor (de hecho esta orientación 
no es perfecta, debido al movimiento térmico de las moléculas, que tiende a desorientarlas). 
  Si las moléculas del dieléctrico no poseen momento dipolar, de todas maneras el campo eléctrico produce una 
separación  o  polarización  de  sus  centros  de  carga  positivos  y  negativos,  y  la  molécula  adquiere  cierto  momento 
dipolar,  de  tal  manera  que  el  efecto  que  describiremos  a  continuación  está  presente  para  molécular  polares  o  no 
polares. 
 

 
Fig. 22 
 
  La carga eléctrica neta contenida en cualquier elemento de volumen tomado dentro del capacitor es 
nula  (en  la  Fig.  22,  la  carga  neta  dentro  de  la  región  limitada  por  la  línea  a  trazos,  es  igual  a  cero).  Sin 
embargo, en las orillas del dieléctrico hay carga no equilibrada. Existe en ellas una distribución superficial 
de carga negativa (llamada  carga inducida) junto a la placa positiva del capacitor, y una distribución de 
carga  positiva  inducida  junto  a  la  placa  negativa,  como  vemos  en  dicha  figura.  Denotemos  con  –σind  y 
σind las densidades de carga de estas distribuciones. 
  ¿Qué sucede entonces con el voltaje a través de las placas, con el campo eléctrico, y con la capacidad 
original C0? Distinguiremos dos situaciones. 
 
(a)  Situación  1  (carga  constante).  La  carga  libre  del  capacitor  (es  decir,  la  carga  de  las  placas  antes  de 
introducir el dieléctrico) no se altera. Se supone pues que no existe un camino (por ejemplo un alambre 
conductor) que conecte las placas con ningún otro dispositivo. 
  Las  Figs.  23  y  24  corresponden  a  esta  situación.  En  la  Fig.  23  el  dieléctrico  todavía  no  se  ha 
introducido. Las placas poseen carga libre “q0” y densidad de carga libre σ0. Se genera un campo eléctrico 
inicial dado por E0 = σ0/ε0. El voltaje entre las placas es V0 = E0 d, donde d es la distancia entre placas. 
5‐16 

 
  Al introducir el dieléctrico, se induce una carga negativa –σind junto a la placa positiva, y una carga 
positiva  σind junto a la placa negativa (Fig. 24). Estas cargas inducidas, por sí solas, producen un campo 
eléctrico Eind que se opone al campo inicial, y cuyo valor es  Eind = σind/ε0. 
  El campo eléctrico total con dieléctrico, Ed, será ahora 
 
σ0 − σind
    Ed = E 0 − E ind =  
ε0
   
 

 
 
 
Fig. 23  Fig. 24 
 
  En virtud de que el voltaje entre las placas es el producto del campo total y la distancia, tendremos 
que la introducción del dieléctrico motiva que el voltaje entre las placas disminuya a 
 

( σ0 − σind ) d
(15)    Vd = Ed d = (E 0 − E ind )d =      (menor que V0) 
ε0
 

Por otra parte, la capacitancia, que inicialmente era C0 = q0/V0, cambia ahora a 
 

q0
(16)    Cd =  
Vd
 

la cual es mayor que la inicial, puesto que Vd < V0. De acuerdo con la definición (1), podemos poner 
 

q0 q
(17)    Cd = = κC0 = κ 0
Vd V0  
 
que nos da 
 
V0 E0
(18)    Vd = y Ed =
κ κ  
 

  Combinando  las  relaciones  anteriores  podemos  llegar  tambien  a  esta  expresión  para  la  carga 
inducida: 
5‐17 

 
⎛ 1⎞ ⎛ 1⎞
(19)    σind = ⎜ 1 − ⎟ σ0 y q ind = ⎜ 1 − ⎟ q 0  
⎝ κ⎠ ⎝ κ⎠
 
Como vemos en (18), la carga inducida siempre es menor que la carga libre. 
  Resumiendo:  al  introducir  el  dieléctrico,  el  voltaje  y  el  campo  eléctrico  disminuyen  en  un  factor 
“1/κ”. La capacitancia aumenta en un factor “κ”. 
 
Situación 2 (voltaje constante). El voltaje entre las placas del capacitor no se altera (es decir, las placas del 
capacitor están conectadas a una fuente de voltaje que mantiene un voltaje constante V entre ellas). 
  Las  Figs.  25  y  26  corresponden  a  esta  situación.  En  la  Fig.  25  el  dieléctrico  todavía  no  se  ha 
introducido. Existen las relaciones  E0 = σ0/ε0  y  V = E0 d. En este caso se sigue cumpliendo que Cd = κC0. 
   

Fig. 25  Fig. 26 
 

  Al introducir el dieléctrico, se induce una carga superficial σind en los bordes del dieléctrico, junto a 
las  placas.  Al  igual  que  en  el  caso  anterior,  estas  cargas  inducidas  producen  un  campo  eléctrico  que  se 
opone  al  campo  original  sin  dieléctrico.  Pero  dado  que  el  voltaje  entre  las  placas  es  constante,  y  que  el 
campo es proporcional al voltaje, tenemos que el campo neto dentro del capacitor no puede variar; debe 
mantener su valor original  E0 = V/d. Se deduce que la fuente de voltaje debe suministrar una carga libre 
adicional  σind a las placas (que cancele el efecto de las cargas inducidas en el dieléctrico), aumentando su 
densidad  de  carga  de  σ  a  σd,  de  tal  manera  que  el  campo  en  el  espacio  dentro  del  capacitor  continúe 
siendo igual a E0: 
 
σ0 (σd − σind )
(18)    E0 = =  
ε0 ε0
 
  Calculemos la carga inducida. Las capacitancias con y sin dieléctrico son 
5‐18 

 
q0 qd
    C0 = Cd = = κC0  
V V
 
De aquí tenemos que la carga del capacitor con dieléctrico es 
 
(19)    qd = κ q0 
 
y de (18) hallamos 
 
(20)     qind = qd – q0 = (κ – 1) q0 
 
EJEMPLO 5.7.  Se tiene un capacitor de placas paralelas con los siguientes datos: área de placa A = 30 cm2; 
distancia  entre  placas  d  =  2  cm.  El  capacitor  se  conecta  a  una  fuente  de  voltaje  de  400  V  y  luego  se 
desconecta de la misma. Luego se introduce en él material Mylar (constante dieléctrica  κ = 3.2; resistencia 
dieléctrica  7  (106)  V/m),  que  llena  completamente  el  espacio  interior.  (a)  Calcular  la  carga  del  capacitor 
después de introducir el dieléctrico, (b) El voltaje del capacitor con dieléctrico, y (c) la carga inducida en el 
dieléctrico. 
‐‐‐‐‐‐‐‐‐‐‐‐‐‐‐‐‐‐‐‐‐‐‐‐‐‐‐ 
  Al conectar el capacitor a la fuente de voltaje, las placas adquieren su carga libre q0, que podemos 
determinar de la relación C = q0/V0. En unidades S.I., 
 
Aε 0 30(10 −4 ) ⋅ 8.85(10 −12 )
    q 0 = CV0 = V0 = −2
⋅ 400 = 532(10 −12 ) (C)  
d 2(10 )
 
(a)  Dado que la carga se mantiene constante al desconectar el capacitor de la fuente de voltaje, tenemos 
que la carga con dieléctrico es la misma que la carga libre: 
 
    qd = q0 = 532 pC 
 
(b)  De la fórmula (18‐p14) tenemos 
 
V0 400V
    Vd = = = 125V =400/3.2 = 125 
κ 3.2
 
(c)  De la fórmula (19), 
 
⎛ 1⎞ ⎛ 1 ⎞
    q ind = ⎜ 1 − ⎟ q 0 = ⎜ 1 − ⎟ 532 pC = 366 pC
⎝ κ⎠ ⎝ 3.2 ⎠  
5‐19 

 
EJEMPLO 5.8.  La capacitancia de un capacitor de placas paralelas en aire es de 40 pF. El área de la placa 
es A = 20 cm2. Se conecta el capacitor a una fuente de voltaje de 30 V y, mientras la fuente sigue conectada, 
se inserta una placa de vidrio pirex (κ = 5.6, resistencia dieléctrica 14 (106) V/m) que ocupa totalmente el 
espacio entre placas. (a) ¿Cuánto vale la carga libre del capacitor después de la inserción del dieléctrico) 
(b) ¿Cuánto vale la carga inducida en el dieléctrico? (c) ¿Y la carga adicional suministrada por la fuente de 
voltaje a las placas al introducir el dieléctrico? (d) ¿Cuánto vale la capacitancia con dieléctrico? (e) ¿Cuánto 
vale el campo eléctrico con y sin dieléctrico? (f) Demostrar que la carga inducida se puede poner también 
en la forma  qind = (1 – 1/κ) qd. 
‐‐‐‐‐‐‐‐‐‐‐‐‐‐‐‐‐‐‐‐‐‐‐‐‐‐‐‐‐ 
(a)  De la fórmula (19‐p16) tenemos que la carga libre del capacitor con dieléctrico es 
 
    q d = κq 0 = κ ⋅ C0 V0 = 5.6 ⋅ 40pF ⋅ 30V = 6.72nC  
 
donde C0 y V0 son la capacitancia y voltaje del capacitor en aire, respectivamente. 
 
(b)   De la fórmula (20‐p16) obtenemos 
 
    qind = (κ – 1) q0 = (κ – 1) C0V0 = (5.6 – 1) 40 pF 30 V = 4.6*40*30 = 5.52 pC 
 
(c)  La carga adicional suministrada por la fuente es igual a la carga inducida en el dieléctrico ya que, 
como el voltaje y el campo total se mantienen constantes, esta carga adicional debe cancelar exactamente 
el efecto de la carga inducida. 
 
    qadic = qind = 5.52 pC 
 
(d)  La capacitancia con dieléctrico es 
 
    Cd = κC0 = 5.6 ⋅ 40pF = 124pF  
 
(e)  El  campo  eléctrico  antes  y  después  de  introducir  el  dieléctrico  es  el  mismo,  ya  que  el  voltaje  del 
capacitor se mantiene constante y el campo es el cociente de voltaje y distancia entre placas. 
 
V0 V0 C0 30 ⋅ 40(10 −12 ) ⎛V⎞
    Ed = E 0 = = = −4 −12
= 67.8(10 3 ) ⎜ ⎟  
d Aε 0 20(10 ) ⋅ 8.85(10 ) ⎝m⎠
 
(Se usó C0 = Aε0/d). 
(f) Usando las relaciones  qd = κ q0  y  qind = (κ – 1) q0 se demuestra fácilmente la relación. 
5‐20 

 
EJEMPLO  5.9.   Entre  las  placas  de  un  capacitor  de  placas  paralelas  se  introduce  una  placa  metálica  de 
grosor “d – a – b”, quedando un espacio libre de dimensión “a” en la parte de arriba, y otro de dimensión 
“b” en la parte de abajo, como se muestra en la figura 27. ¿Cómo se altera la capacitancia? 
 

 
Fig. 27 
‐‐‐‐‐‐‐‐‐‐‐‐‐‐‐‐‐‐‐‐‐‐‐‐‐‐‐‐‐‐‐‐‐‐‐‐‐‐‐‐‐‐‐ 
  Las cargas del capacitor original inducen cargas superficiales en la placa metálica conductora. Dado 
que  el  campo  dentro  del  conductor  debe  ser  nulo,  se  deduce  que  la  densidad  de  carga  inducida  en  el 
metal debe ser la misma que la de las placas, como se muestra en la Fig. 28. 
 

 
Fig. 28 
 

  El voltaje a través de las placas del capacitor es 
 
σ
    V = Ea + E b = (a + b)  
ε0
 

  La capacitancia es 
 

q0 σA Aε 0
    C= = =  
σ
V (a + b) a + b
ε0
 

Notemos  que  la  capacitancia  no  depende  de  la  posición  de  la  placa  dentro  del  capacitor,  puesto  que  la 
suma “a + b” es independiente de la misma. Asimismo, la configuración de la Fig. 24 se puede considerar 
como dos capacitores de distancias entre placas “a” y “b”, conectados en serie (la fórmula (8‐p8) conduce 
a la misma expresión para la capacitancia hallada en este ejemplo). 
5‐21 

 
5.8. PROBLEMAS. 
 
3.   Las siguientes cantidades se refieren a un capacitor de placas paralelas: 
A = área de cada placa; d = distancia entre placas; Q = carga del capacitor (esto es, de su placa positiva), 
V = voltaje entre las placas; E = campo eléctrico; σ = densidad de carga de las placas; C = capacitancia. 
Dadas las siguientes 3 cantidades, calcular las restantes: 
(a) Q, σ y E 
(b) V, d y A 
(c) E, d y Q 
(d) C, σ y d 
 
4.  Calcular  la  capacitancia  de  un  capacitor  cilíndrico  de  altura  21  cm,  radio  interior  R1  =  8  cm  y  radio 
exterior R2 = 12 cm. ¿Qué carga almacena este capacitor si se le aplica un voltaje de 240 V? 
 
1.  Calcular la capacitancia equivalente de las siguientes combinaciones de capacitores entre los puntos “a” 
y “b”. 
(a) 

     
 
Resp. 4.71 pF 
 
(b) 
 

     
 
Resp. 146.66 μF. 
5‐22 

 
2.  Calcular la carga eléctrica suministrada por la batería a la combinación de 4 capacitores mostrada. 
 

     
 
Resp. 3840 pC. 
 
9.  Utilizando las fórmulas dadas en el Ejemplo 5.5, página 5‐11, demostrar que la densidad de energía “u” 
del capacitor  (energía almacenada en el capacitor por unidad de volumen del mismo) es 
 
1
    u= ε0 E 2  
2
 
donde E es el campo eléctrico del capacitor. 
 
6. Un capacitor de placas paralelas está construído con placas de área 20 cm2, separadas 12 mm, y lleno con teflón. 
¿Cuál es el voltaje máximo que puede soportar este capacitor? 
 
8.  Se tiene un capacitor de placas paralelas con los siguientes datos: área de placa A = 20 cm2; distancia 
entre placas d = 1.2 cm. El capacitor se conecta a una fuente de voltaje de 220 V y luego se desconecta de la 
misma. Luego se introduce en él un material de constante dieléctrica  κ = 2.8; resistencia dieléctrica 9 (106) 
V/m),  que  llena  completamente  el  espacio  interior.  (a)  Calcular  la  carga  del  capacitor  después  de 
introducir el dieléctrico, (b) El voltaje del capacitor con dieléctrico, y (c) la carga inducida en el dieléctrico. 
 
9. La capacitancia de un capacitor de placas paralelas en aire es de 50 pF. El área de la placa es A = 40 cm2. 
Se conecta el capacitor a una fuente de voltaje de 60 V y, mientras la fuente sigue conectada, se inserta una 
placa de teflón (κ = 2.1, resistencia dieléctrica 60 (106) V/m) que ocupa totalmente el espacio entre placas. 
(a) ¿Cuánto vale la carga libre del capacitor después de la inserción del dieléctrico) 
(b) ¿Cuánto vale la carga inducida en el dieléctrico? (c) ¿Y la carga adicional suministrada por la fuente de 
voltaje a las placas al introducir el dieléctrico? (d) ¿Cuánto vale la capacitancia con dieléctrico? (e) ¿Cuánto 
vale el campo eléctrico con y sin dieléctrico? 
 
 
6‐1 

  CAPÍTULO 6 
   
  CORRIENTE ELÉCTRICA Y RESISTIVIDAD 
 
 
 

6.1. DEFINICIÓN DE CORRIENTE ELÉCTRICA. 
  El movimiento de cargas eléctricas constituye lo que se denomina una corriente eléctrica. 
  Ejemplos: 
• El movimiento de los electrones libres (carga negativa) dentro de un alambre conductor. 
• El movimiento de un haz de protones (carga positiva) generado en un acelerador de partículas. 
• El movimiento de iones en una batería de automóviles o un plasma. 
• El movimiento tanto de cargas positivas como negativas en gases y electrolitos. 
• El  movimiento  de  vacancias  (ausencia  de  electrones)  o  huecos  en  un  material  semiconductor.  Las 
vacancias se comportan como cargas positivas en movimiento. 
  La carga móvil puede ser positiva o negativa. Por convención, la dirección de la corriente eléctrica 
se  toma  como  aquella  hacia  la  que  se  mueve  la  carga  positiva,  de  tal  manera  que  si  la  corriente  está 
formada  puramente  por  electrones  u  otra  clase  de  portadores  de  carga  negativa,  la  dirección  de  la 
corriente  es  contraria  a  la  del  movimiento  de  los  mismos.  Los  efectos  producidos  por  las  corrientes 
eléctricas  (generación  de  campos  magnéticos,  fenómenos  electrolíticos,  calentamiento,  etc.)  son 
independientes de esta convención relativa a la dirección de la corriente. 
  Podemos  visualizar  la  corriente  como  un  flujo  de  carga  a  través  de  una  superficie  abierta. 
Cuantitativamente,  la  corriente  eléctrica  a  través  de  una  superficie  es  igual  a  la  carga  eléctrica  que 
atraviesa  la  superficie  por  unidad  de  tiempo.  Dado  que  la  corriente  puede  ser  una  función  del  tiempo, 
para calcularla habría que considerar un lapso de tiempo muy corto “dt”; si la carga que pasa durante tal 
lapso es “dq”, entonces, denotando la corriente con “i”, podemos poner 
 
 
Definición  de  la  corriente  eléctrica  a  través  de  una 
dq
(1)    i=   superficie.  “dq”  es  la  carga  que  atraviesa  la  superficie 
dt
considerada durante un lapso de tiempo “dt”. 
 

La unidad física de la corriente es el amperio. En el capítulo 7 daremos la definición exacta. Por lo pronto 
definiremos el amperio así: 
 
(2)    amperio = A = coulombio / segundo = C / s
 
 

6.2. CORRIENTE ELÉCTRICA EN UN CONDUCTOR. 
  Cuando un cuerpo conductor neutro se coloca dentro de un campo eléctrico estático  E, se induce 
una  distribución  de  carga  eléctrica  en  su  superficie.  Estas  cargas  inducidas  se  acomodan  de  tal  manera 
que, en todo punto en el interior del conductor, generan un campo eléctrico que cancela el campo eléctrico 
E.  Esto  ocurre  durante  un  espacio  de  tiempo  muy  corto,  durante  el  cual  hay  movimiento  de  electrones 
libres (es decir, una corriente eléctrica) desde el interior del conductor hasta su superficie. Los electrones 
son  impulsados  allí  por  el  campo  eléctrico  neto  que  existe  en  el  interior  del  conductor  durante  el 
brevísimo tiempo que las cargas se reacomodan y se llega a una situación electrostática. La corriente cesa 
cuando se establece una situación estática (cuando E = 0 dentro del conductor). 
6‐2 

  He aquí otra situación en la que se tiene dentro de un conductor una corriente eléctrica que dura 
también  un  tiempo  muy  corto:  consideremos  dos  placas  conductoras  cargadas,  una  negativamente  y  la 
otra positivamente, como se muestra en la Fig. 1. En el espacio entre placas existe un campo eléctrico.  
   

Fig. 1  Fig. 2 
 

  Coloquemos ahora una placa conductora entre las dos placas, haciendo contacto con ellas (Fig. 2). 
(Se  dice  que  estamos  poniendo  las  placas  en  cortocircuito,  lo  cual  no  es  recomendable  hacer  en  el 
laboratorio en muchas circunstancias). Los electrones de la placa cargada negativamente fluirán a través 
de  la  barra,  llenando  el  déficit  de  electrones  de  la  placa  cargada  positivamente.  Las  placas  se  vuelven 
neutras  y  su  campo  eléctrico  desaparece.  Esto  ocurre  durante  una  pequeñísima  fracción  de  segundo, 
durante la cual existe una corriente eléctrica a través de la barra. Los electrones en tránsito entre ambas 
placas  son  impulsados  por  el  campo  eléctrico  generado  por  las  cargas  en  las  placas  (la  fuerza  eléctrica 
sobre los electrones está dirigida en la Fig. 2 hacia la derecha). 
  Para lograr mantener una corriente eléctrica estable y duradera dentro de un conductor es necesario 
un  mecanismo  para  mantener  un  campo  eléctrico  estable  y  sostenido  en  el  interior  del  conductor.  Este 
campo impulsaría los electrones libres del conductor, aplicándoles una fuerza eléctrica que los mantendría 
en movimiento, constituyendo así una corriente eléctrica. 
  Esta situación no está en desacuerdo con lo que sabemos de la electrostática (que el campo eléctrico 
dentro  de  un  conductor  es  nulo),  puesto  que  no  se  trata  aquí  de  una  situación  estática  (existen  cargas 
eléctricas en movimiento dentro del conductor). 
  El mecanismo más común para producir una corriente eléctrica estable en un conductor consiste en: 
• Hacer a este conductor parte de un circuito eléctrico, es decir, un camino cerrado por el que puedan 
circular  continuamente  sus  electrones  libres.  Supongamos  que  el  conductor  es  en  este  caso 
simplemente un alambre metálico de sección transversal de área A y densidad de electrones libres 
“n”. 
• Utilizar un dispositivo (denominado fuente de fuerza electromotriz (o brevemente fuente de fem)) 
que  impulse  a  los  electrones  libres  para  que  puedan  realizar  su  viaje  alrededor  del  circuito.  La 
fuente de fem comunica continuamente a los electrones la energía necesaria para mantenerse en 
movimiento. Hablemos un poco más sobre esta clase de dispositivo. 
6‐3 

  Antes recordemos aquí un resultado obtenido anteriormente. 
  Consideremos  una  carga  puntual  positiva  q 
dentro  de  un  campo  eléctrico  E  (Fig.  3). 
Supongamos  que  la  carga  se  mueve  entre  dos 
puntos  entre  los  cuales  existe  una  diferencia  de 
potencial  V,  haciéndolo  de  menor  a  mayor 
potencial  (desde  φ  hasta  φ  +  V,  con  V  >  0). 
Sabemos  que  el  campo  eléctrico  entre  dos  puntos 
está dirigido de mayor a menor potencial, es decir, 
la  carga  se  estaría  moviendo  en  este  caso  contra  la 
fuerza eléctrica que le ejerce el campo E. 
  Al  trasladarse  de  esta  manera,  la  carga  q   
aumenta su energía potencial eléctrica en el valor  Fig. 3 
U = qV. Esto puede suceder ya sea porque la carga  
disminuye su energía cinética en la transición, o porque existe un dispositivo que le comunica la energía 
necesaria, o por ambas razones.  
 
(3)  Una carga puntual positiva q que   se desplaza contra un campo eléctrico entre dos 
  puntos cuya diferencia de potencial (o voltaje) es V requiere de una energía “qV” 
  para realizar esta transición. 
 
  Existen  dispositivos  eléctricos  que  realizan  precisamente  esta  acción,  es  decir,  que  suministran 
energía a las cargas eléctricas (digamos positivas), que les permita vencer un campo eléctrico y moverse 
(dentro del dispositivo) hacia puntos a mayor potencial eléctrico. Un dispositivo así se denomina fuente de 
fuerza electromotriz (abreviado fuente de fem o simplemente fem). 
  Típicamente,  una  fuente  de  fuerza  electromotriz  se  utiliza 
en un circuito eléctrico (un camino cerrado por el que circula una 
corriente eléctrica) para mantener en él una corriente eléctrica. La 
fuente  de  fem  posee  una  terminal  positiva  y  una  negativa,  entre 
las  cuales  mantiene  idealmente  una  diferencia  de  potencial 
constante  “E”.  La  Fig.  4  muestra  esquemáticamente  una  pila 
eléctrica  ordinaria,  la  cual  es  una  fuente  de  fem  que  idealmente 
mantiene  una  diferencia  de  potencial  de  1.5  voltios  entre  sus 
terminales. 
  Durante  su  operación,  la  pila  genera  un  campo  eléctrico 
tanto en el exterior como en el interior, el cual hemos denotado en 
la figura con E (afuera) y E’  (adentro) respectivamente. En ambas 
regiones, el campo eléctrico se dirige siempre de mayor a menor   
potencial.  Sus  líneas  de  fuerza  nacen  en  las  cargas  positivas  y  Fig. 4 
mueren en las negativas. 
  Conectemos  ambas  terminales  de  la  pila  mediante  un  alambre  conductor,  formando  un  circuito 
eléctrico (Fig. 5). 
6‐4 

  En la Fig. 5, la fuente de fem se ha representado por un rectángulo cuya terminal positiva es su lado 
superior  y  la  negativa  su  lado  inferior.  El  resto  del  circuito  mostrado  en  la  Fig.  5  consta  de  un  alambre 
conductor doblado en forma de un rectángulo. 
 

 
Fig. 5 
 

  Ahora bien, la fuente de fem produce tanto en el espacio vacío circundante, como en el interior del 
alambre conductor, como también en el interior de la propia fuente, un campo eléctrico  E cuya dirección 
es  la  que  se  muestra  en  la  Fig.  5.  Con  este  campo  eléctrico  tenemos  asociado  un  potencial  eléctrico  φ. 
Tomando  como  referencia  del  potencial  el  de  la  terminal  positiva  (φ  =  0),  entonces  el  potencial  de  la 
terminal positiva será φ = E. 
  Recordemos  que  el  campo  eléctrico  está  dirigido  en  todo  punto  de  mayor  a  menor  potencial 
eléctrico. Entonces, en el interior del alambre conductor el campo  E se dirige en el sentido indicado en la 
Fig. 5, y en el interior de la fuente el campo E’ se dirige de la terminal positiva a la negativa. 
  Por facilidad, supongamos que las cargas libres del conductor son positivas (sabemos de hecho que 
son  negativas,  puesto  que  son  electrones).  El  campo  eléctrico  de  la  fuente  dentro  del  alambre  actuaría 
sobre  estas  cargas  libres  haciendo  que  se  desplazaran  en  la  dirección  del  campo  E,  es  decir,  desde  la 
terminal  positiva  de  la  fuente  hasta  la  terminal  negativa,  a  lo  largo  del  alambre.  Dentro  de  la  fuente,  el 
campo eléctrico E’ está en dirección contraria al flujo de las cargas positivas libres (o equivalentemente de 
la corriente). Para lograr desplazarse desde la terminal negativa hasta la positiva y continuar circulando, 
las cargas positivas deben adquirir una energía potencial que les permita vencer el campo eléctrico dentro 
de la fuente. 
  Precisamente esta es la función de la fuente de fem. En la pila ordinaria, la fuente de fem utiliza un 
proceso químico para comunicar a cada carga “q” una (fuerza o) energía potencial igual a “q E”, que le 
permita superar la fuerza eléctrica del campo interior de la fuente. 
  Dentro  de  la  pila  hay  una  transformación  de  energía  química  a  energía  potencial  eléctrica.  La 
corriente en el circuito considerado sigue fluyendo hasta que la pila agota su energía química. 
Nota. El análisis efectuado aquí sería esencialmente el mismo si hubiésemos partido desde el principio del 
hecho de que las cargas móviles dentro del alambre son electrones. En este caso el movimiento de estas 
cargas dentro del alambre sería en el sentido contrario al de las manecillas del reloj y, dentro de la pila, de 
la terminal positiva a la negativa. 
6‐5 

  ¿Cuál es el valor de la corriente eléctrica que circula por el alambre conductor de la Fig. 5? 
  Empecemos  con  el  campo  eléctrico  en  el  conductor.  Sea  LT  la  longitud  total  del  alambre,  de 
terminal a terminal, y E, como dijimos, el voltaje constante mantenido por la fuente. Entonces el campo 
dentro del conductor es en magnitud 
 
E Campo  eléctrico  dentro  del  alambre  conductor.  LT 
(4)    E=  
LT es la longitud total del alambre. 
 
y está dirigido a lo largo del alambre en todo punto. 
  Definamos la densidad de corriente J del conductor en la forma 
 
i Densidad de corriente en el alambre. A es el área de 
(5)    J=  
A la sección transversal del alambre. 
 
  Se  encuentra  experimentalmente  que  para  ciertos  materiales,  denominados  óhmicos,  existe  la 
siguiente relación entre el campo E y la densidad de corriente J: 
 
(6)    J = σc E  Ley de Ohm 
 
donde  σc es una constante material denominada conductividad eléctrica. Para los materiales óhmicos, sc 
es una constante independiente de E, pero dependiente de la temperatura. 
  Es de notar que la relación (6) no es válida para todos los materiales, sino para una clase especial, 
los materiales óhmicos. Para ellos, esta relación se denomina Ley de Ohm. 
  De  las  relaciones  (4),  (5)  y (6)  podemos  derivar  la  siguiente  relación  entre  la  corriente  y  el  voltaje 
que suministra la fuente de fem: 
 
σc A
    i= E 
LT
 
  Finalmente, definiendo la resistividad eléctrica del conductor en la forma 
 
1 La  resistividad  eléctrica  es  el  recíproco  de  la 
(7)    ρ=  
σc conductividad eléctrica. 
 
y la resistencia eléctrica del alambre de longitud LT y sección tranversal de área A en la forma 
 
ρL T Resistencia  eléctrica  de  un  alambre  conductor  de 
(8)    R=  
A longitud LT y área transversal A. 
 
se llega a 
 
 
Ley  de  Ohm  (materiales  óhmicos).  El  voltaje  es  el 
(9)    E = R i 
producto de la resistencia y la corriente. 
6‐6 

  Tanto (6) como (9) se suelen denominar Ley de Ohm, pues son equivalentes. 
  De (6) sacamos las unidades físicas de la conductividad eléctrica: 
 

amperio / metro 2 amperio 1 1


    = = =  
voltio / metro voltio ⋅ metro ohmio ⋅ metro Ω ⋅ m
 

donde hemos definido la unidad 
 
voltio V
    1 ohmio = 1 Ω = 1 =1  
amperio A
 

De (7), la unidad física de la resistividad es 
 

    1 ohmio ⋅ metro = 1 Ω ⋅ m 
 

y de (8) la de la resistencia eléctrica es 
 

    1 ohmio = 1 Ω 
 

  La resistividad es una de las propiedades materiales con más amplio rango de variación a través de 
los materiales, como se puede apreciar en la siguiente tabla (valores a 20 °C). 
 
Material  Resistividad ρ (ohmio ⋅ m)  Coeficiente de temperatura α (1/°C) 
Cobre  1.69 (10–8)  4.3 (10–3) 
Plata  1.62 (10–8)  4.1 (10–3) 
Hierro  9.68 (10–8)  6.5 (10–3) 
Carbono  3.5 (10–5)  –0.5 (10–3) 
Silicio puro  2.5 (103)  –70 (103) 
Vidrio  1010 ‐‐‐‐ 1014   
Poliestireno  > 1014   
Cuarzo fundido  75 (1016)   
 
No existe un conductor perfecto, ni un aislador perfecto.  
  En  un  rango  limitado  de  temperatura,  la  resistividad  de  un  conductor  varía  linealmente  con  la 
temperatura: 
 
(10)    ρ = ρ0 [1 + α (T – T0)]  Variación de la resistividad con la temperatura 
 
Aquí, T está en grados Celsius (°C),  ρ0 es la resitividad a cierta temperatura de referencia T0 (usualmente 
20°C). El coeficiente α se denomina coeficiente de temperatura de resistividad. 
  Dado  que,  por  (8),  la  resistencia  es  proporcional  a  la  resistividad  (con  A  y  L  constantes)  se  tiene 
también 
 
(11)    R = R0 [1 + α (T – T0)]  Variación de la resistencia con la temperatura 
6‐7 

  Para metales como el cobre, la resistividad es prácticamente proporcional a la temperatura, como se 
muestra en la Fig. 6, excepto para muy bajas temperaturas, donde la resistividad se acerca a un valor finito 
ρ0 cuando la temperatura se acerca al cero absoluto. 
   

 
Fig. 6  Fig. 7 
 
  Observe  que  el  valor  de  α  para  el  silicio  puro  es  negativo.  Para  este  material  (y  otros  con  α 
negativa) la resistividad se reduce al elevarse la temperatura (Fig. 7). Se trata en este caso de materiales 
semiconductores. 
 
EJEMPLO 6.1. Un cilindro sólido recto de aluminio tiene una sección circular de diámetro 0.5 cm y una 
longitud  de  20  cm.  ¿Cuál  es  la  resistencia  eléctrica  del  cilindro  entre  sus  extremos?  La  resistividad  del 
aluminio a temperatura ambiente es de 2.75 (10–8) Ω m. 
‐‐‐‐‐‐‐‐‐‐‐‐‐‐‐‐‐‐‐‐‐‐‐‐‐‐‐‐‐‐‐‐‐‐ 
  El área de la sección transversal del cilindro es 
 
πd 2 3.14(0.5 ⋅ 10 −2 m)2
    A= = = 1.96(10 −5 )m 2  
4 4
 
La resistencia es 
 
L 20(10 −2 )m
    R =ρ = 2.75(10 −8 )Ω ⋅ m −5 2
= 28(10 −4 )Ω = 0.0028 Ω
A 1.96(10 )m  
 
Una diferencia de potencial de E = 1.5 voltios entre los extremos de este cilindro produciría teóricamente 
una corriente de 
 
    i = E/R = 1.5 V/0.0028 Ω = 535.7 A   (!!!!) 

     
lo cual indica que la unidad “ohmio” es muy pequeña. En la práctica se construyen trozos de materiales 
llamados resistores, cuyas resistencias son mucho mayores, desde decenas de ohmios hasta millones de 
ohmios. Son útiles los múltiplos 
 
    1 kΩ = 1 kilohmio = 1000 Ω         1 MΩ = 1 megohmio = 1000 000 Ω 
6‐8 

  En  los  diagramas  de  circuitos  eléctricos  muchas  veces  se  desprecia  la  resistencia  de  los  alambres 
conductores. Por otra parte, el símbolo gráfico de un resistor es el siguiente 
 

 
Fig. 8 
 
6.3. VISIÓN MICROSCÓPICA DE LA CORRIENTE EN UN ALAMBRE CONDUCTOR. 
  Consideremos  un  alambre  conductor  metálico  de  sección  transversal  de  área  A  y  densidad  de 
electrones libres “n”. En un modelo simple, el metal consiste en una retícula rígida de átomos con carga 
neta  positiva,  rodeada  de  un  “gas”  de  electrones  libres.  Estos  electrones,  libres  de  vagar  por  todo  el 
material, efectúan un movimiento aletorio similar al de las moléculas de un gas (la rapidez media de estos 
electrones  es  del  orden  de  ¡106  m/s!).  En  su  camino,  los  electrones  libres  sufren  innumerables  colisiones 
con los átomos de la retícula. Entre colisiones, los electrones prácticamente no interaccionan con la retícula 
y se mueven en trayectorias rectas. 
  En ausencia de un campo eléctrico dentro del conductor, la velocidad promedio de los electrones 
libres es nula (Fig. 9), pues no hay desplazamiento neto en ninguna dirección. 
   No sucede lo mismo si existe un campo eléctrico  E: los electrones libres sufren la fuerza eléctrica 
(opuesta a  E), con el resultado de que realizan en promedio unos desplazamientos en la dirección opuesta 
al  campo  eléctrico.  Los  electrones  libres  adquieren  así  una  velocidad  neta,  denominada  velocidad  de 
arrastre  o  velocidad  de  deriva,  denotada  con  “vd”  (Fig.  10).  Tal  flujo  neto  de  electrones  (en  la  dirección 
opuesta al campo) constituye una corriente eléctrica. 
 

   
Fig. 9  Fig. 10 
 
  La  corriente  eléctrica  “i”  que  circula  por  el  alambre  conductor  se  puede  relacionar  con  ciertas 
propiedades atómicas de los portadores de carga eléctrica dentro del conductor. 
6‐9 

 
Fig. 11 
 
  Sea  “i”  la  corriente  eléctrica  en  el  conductor.  La  Fig.  11  muestra  una  sección  longitudinal  del 
alambre.  Se  representan  allí  los  portadores  de  carga  (electrones  libres)  mediante  pequeños  círculos 
sombreados. Se supone que todos ellos tienen la misma carga “e” y se desplazan con la misma velocidad 
promedio, la velocidad de deriva “vd”. 
  Tracemos una sección transversal S en cualquier lugar del alambre. Sea A el área de esta sección. 
  La corriente eléctrica es igual a la carga eléctrica que atraviesa la sección transversal S por unidad 
de  tiempo.  Para  calcular  esta  carga  marquemos  desde  la  sección  S  una  distancia  “vd  Δt”  hacia  atrás,  la 
cual es igual a la distancia que recorre cada portador de carga (o sea cada electrón libre) en un tiempo Δt.  
Entonces  todos  los  electrones  libres  contenidos  en  el  cilindro  de  generatriz “vdΔt”  y área  transversal  A 
lograrán  atravesar  dicha  sección  S  en  el  tiempo  Δt.  Para  obtener  el  número  ΔN  de  tales  electrones 
multiplicamos la densidad “n” por el volumen de este cilindro: 
 
    ΔN = n ⋅ (A vd Δt) 
 
Esto implica que la carga eléctrica ΔQ que atraviesa la sección S en un tiempo Δt es 
 
    ΔQ = n A vd Δt e 
 
Dividiendo ΔQ por Δt obtenemos la carga que atraviesa por unidad de tiempo, o sea la corriente “i”: 
 
ΔQ nAvd eΔt
    i= =
Δt Δt  
 
  Corriente eléctrica en un alambre conductor. 
 
n = densidad de electrones libres; A = área de la sección 
(12)    i = n A vd e 
transversal del alambre; vd = velocidad de deriva de los 
electrones libres; e = carga del electrón. 
 
Llamaremos a (12) la “fórmula de la nave”. 
6‐10 

EJEMPLO 6.2. Un alambre de cobre de sección circular de diámetro d = 1 cm transporta una corriente de 
20 A. Calcular la velocidad de deriva vd de los electrones. La densidad de electrones libres del cobre es 
n = 8.5 (1028) / m3. 
‐‐‐‐‐‐‐‐‐‐‐‐‐‐‐‐‐‐‐‐‐‐‐‐‐‐‐‐‐‐‐‐ 
  Despejando vd de la Ec. (12), 
 
i
    vd =  
nAe
 
El área de la sección transversal es 
 

πd2 3.14(10 −2 )2 m 2
    A= = = 0.785(10−4 ) m 2  
4 4
 

Entonces, 
 
20A m
    vd = = 1.88(10 −5 )  
1 −4 −19 s
8.5(10 ) 3 ⋅ 0.78(10 )m ⋅ 1.6(10 )C
28 2
m
 

O bien 
 
mm
    vd ≈ 2   
s
 

Como vemos, la velocidad de deriva es muy pequeña. 
 
6.4. CORRIENTE Y RESISTENCIA EN UN CIRCUITO SIMPLE. 
  Volvamos  a  la  situación  representada  en  la  Fig.  5  de  la  página  6‐4,  que  reproducimos  a 
continuación: 

 
(Fig. 5) 
 
6‐11 

  El campo eléctrico dentro del alambre ya fue calculado (Ec. (4)). Resulta igual a 
 
E
[(4)]    E=  
LT
 

Sea “V” la diferencia de potencial entre los extremos de un pedazo de alambre de longitud L (Fig. 12), y 
sea “R” la resistencia de este pedazo, dada por la Ec. (8‐p5) en la forma 
 
ρL
(13)    R=  
A
 

 
Fig. 12 
 

Dado que el campo eléctrico es constante, podemos poner también, en lugar de (4), 
 
V
(14)    E=  
L
 

Se sigue también la relación 
 

(15)    V = R i 
 

donde “i” es la corriente en el alambre. 
  Note que las relaciones (13), (14) y (15) son completamente análogas a (8), (4) y (9), respectivamente. 
La diferencia es que estas últimas se aplican a todo el alambre (longitud LT), y las primeras se aplican a un 
trozo de alambre de longitud L. Para efectos de calcular la corriente “i”, el diagrama de la Fig. 5 se puede 
simplificar al de la Fig. 13, donde la resistencia total del alambre se ha concentrado en un resistor: 
 

 
Fig. 13 
 
  En el circuito simple de la Fig. 13 se tiene, como ya sabemos, i = E / R. La caída de potencial a través 
del resistor R (o sea la diferencia de potencial o voltaje entre sus extremos) es  V = R i = E. 
6‐12 

  El símbolo gráfico de la fuente de fem en los circuitos es el siguiente: 
 

 
 
Fig. 14 
 

de modo que el circuito de la Fig. 13 también se puede representar así: 
 

 
Fig. 15 
 

6.5. RESISTENCIA EQUIVALENTE DE VARIOS RESISTORES. LEYES DE KIRCHHOFF. 
  Dos resistores están conectados en serie si la corriente eléctrica es la misma a través de cada uno. La 
Fig. 16 muestra una situación tal. 
  Se define la resistencia equivalente del par de 
resistores  R1  y  R2  como  la  resistencia  de  un  solo 
resistor por el que circula la misma corriente “i” que 
en la conexión de la Fig. 16. Denotemos la resistencia 
equivalente con Requ y calculémosla.  E 
  De la Ec. (15) aplicada a cada resistor tenemos 
 
    V1 = R1 i    V2 = R2 i 
 
  Fig. 16 
Ahora bien, dado que el potencial en todo punto del   
circuito  es  una  función  unívoca  de  la  posición,  se  sigue  que  las  caídas  de  potencial  a  través  de  ambos 
resistores, sumadas, deben ser iguales a la diferencia de potencial suministrada por la fuente de fem, es 
decir, 
 

(16)    E = V1 + V2 
 

o sea 
 

    E = R1 i + R2 i = (R1 + R2) i 
 

Se deduce que la resistencia equivalente de los dos resistores es 
 
   

(17)    Requ = R1 + R2  Resistencia equivalente de dos resistores en serie 


6‐13 

 
  Así pues, en cuanto al cálculo de la corriente 
suministrada por la fuente de fem, el circuito de la 
Fig.  16  es  equivalente  al  de  la  Fig.  17.  En  él  se  ve 
inmediatamente que 
 
E E
    i= =   E 
R equ R1 + R 2

 
Fig. 17 
 
  Volviendo  a  la  Fig.  16,  los  signos  “+”  y  “–“  junto  a  los  extremos  de  cada  resistor  indican  la 
polaridad  de  dichos  extremos.  Se  entiende  por  ello  que  el  extremo  etiquetado  con  el  signo  “+”  está  a 
mayor potencial que el extremo con “–“. Entonces, si recorremos el circuito de la Fig. 16 en la dirección de 
la corriente eléctrica, tenemos sendas caídas (o disminuciones) de potencial en ambos resistores. Por otra 
parte,  recorriendo  la  fuente  de  fem  en  el  sentido  de  la  corriente  tenemos  que  el  borne  negativo  está  a 
menor potencial que el positivo, de modo que en este caso tenemos una elevación de potencial al recorrer 
desde el borde negativo al positivo. En estos términos, la ecuación (16), escrita en la forma 
 
(18)    E – V1 – V2 = 0 
 
se puede leer así: 
 
(19)  Si  se  recorre  un  camino  cerrado  a  lo  largo  de  un   
circuito con fuentes de fem y resistores, la suma algebraica de   
las  variaciones  de  potencial  para  todos  los  elementos  del  Segunda Ley de Kirchhoff. 
circuito  es  igual  a  cero.  En  un  resistor  existe  una  caída  de  (Una  caída  de  potencial  se  considera  una 
variación  negativa  del  mismo,  una 
potencial  (o  una  elevación)  si  el  mismo  se  recorre  en  el 
elevación un cambio positivo). 
sentido  de  la  corriente  (o  en  el  sentido  opuesto).  En  una 
fuente  de  fem  hay  una  elevación  de  potencial  (o  caída)  si  la 
fuente se recorre del borne negativo al positivo (o al revés)  
 

  Dos resistores están en paralelo si la diferencia de potencial a través de cada uno es la misma. La 
Fig. 18 muestra tal situación. 
 

 
Fig. 18 
 

Cualquier camino cerrado a lo largo del circuito compuesto de la Fig. 18 se denomina una malla. Existen 
6‐14 

en él por lo tanto tres mallas, a saber, 
• Desde  el  borne  positivo  de  la  fuente  de  fem,  pasando  por  el  nodo  “a”,  luego  hacia  abajo  por  el 
resistor R1, luego por el nodo “b”, y finalmente llegando a la fuente de fem por el borne negativo. 
(Nota: un nodo es un punto del circuito donde confluyen más de dos segmentos de alambre por 
los que circulan en general distintas corrientes). 
• Desde el borne positivo de la fuente de fem, pasando por el nodo “a”, luego por el punto “c”, por el 
resistor  R2,  por  el  punto  “d”,  por  el  nodo  “b”,  y  finalmente  llegando  al  borne  negativo  de  la 
fuente. 
• Desde  el  nodo  “a”,  por  el  punto  “c”,  por  el  resistor  R2,  por  el  punto  “d”,  por  el  nodo  “b”  y 
finalmente por el resistor de nuevo hasta el nodo inicial “a”. 
  El potencial eléctrico es el mismo por un lado en los puntos “a” y “c”, y por otro en los puntos “b” y 
“d”, puesto que ambas parejas de puntos pertenecen al mismo conductor equipotencial. Esto significa que 
el voltaje a través de cada resistor es el mismo: 
 
    φa = φc       φb = φd 
 
    Vab = φb – φa = φd – φc = Vcd 
 
Ambos voltajes son iguales a E: 
 
    Vab = Vcd = E 
 
Aplicando (15), la corriente a través de R1 es 
 
Vab E
(20)    i1 = =  
R1 R1
 

y a través de R2 es 
 
Vcd E
(21)    i2 = =  
R2 R2
 

  La corriente “i” suministrada por la fuente de fem se bifurca en el nodo “a”, dividiéndose en dos 
corrientes “i1” e “i2”. Existe la relación 
 
(22)    i = i1 + i2 
 
la  cual  significa  físicamente  que  la  carga  eléctrica  que  fluye  por  las  diversas  ramas  del  circuito  no  se 
acumula en ningún punto; esto es, la corriente que “entra” al nodo “a” (o sea “i”) es igual a la corriente 
total que “sale” de dicho nodo (o sea “i1 + i2”). 
 
  Primera Ley de Kirchhoff. 
(23)    i = i1 + i2  La corriente total que entra a un nodo es igual a la 
corriente total que sale de él. 
 
6‐15 

  De (20) y (21) obtenemos la siguiente relación: 
 
 
La corriente a través de cada uno de dos resistores 
i1 R 2
(24)    =   en  paralelo  es  inversamente  proporcional  a  sus 
i2 R1 resistencias. 
 
 
Sustituyendo ahora (20) y (21) en (22) tenemos 
 
E E E
    i= + =  
R1 R 2 R1 + R 2
 
y comparando con la relación definitoria de la resistencia equivalente, a saber, 
 
E
    i=  
R equ
 
deducimos la relación 
 
1 1 1 Resistencia  equivalente  de  dos  resistores 
(25)    = +  
R equ R1 R 2 conectados en paralelo. 
 
Otra forma de (25) es 
 
R 1R 2 Resistencia  equivalente  de  dos  resistores 
(26)    R equ =  
R1 + R 2 conectados en paralelo. 
 
  Las  fórmulas  encontradas  para  la  resistencia  equivalente  de  resistores  en  serie  y  en  paralelo  son 
generalizables a más de dos resistores. Así, la resistencia equivalente de N resistores conectados en serie 
es 
 
    Requ = R1 + R2 + … + RN 
 
y la de N resistores conectados en paralelo es 
 
1 1 1 1
    = + + ... +  
R equ R1 R 2 RN
 
  La resistencia equivalente de resistores en serie siempre es mayor que cualquiera de las resistencias 
individuales.  En  cambio,  la  de  los  resistores  en  paralelo  siempre  es  menor  que  cualquiera  de  las 
resistencias individuales. 
6‐16 

EJEMPLO 6.3. Se tiene un circuito con una fuente de fem de 120 V y tres resistores de 80  Ω, 50  Ω y 30  Ω 


conectados como se muestra en la Fig. 19. (a) Calcular la corriente eléctrica suministrada por la fuente de 
fem.  Haga  el  cálculo  reduciendo  el  circuito  a  una  sola  resistencia  equivalente.  (b)  Calcular  la  corriente 
eléctrica a través de cada resistencia. (c) Calcular la caída de potencial a través de cada resistencia. 
 

 
Fig. 19 
‐‐‐‐‐‐‐‐‐‐‐‐‐‐‐‐‐‐‐‐‐‐‐‐‐‐‐‐‐‐‐‐‐‐‐‐‐‐‐‐‐‐‐‐‐‐‐‐‐‐ 
 

Fig. 20  Fig. 21
 
(a)  Primeramente  reducimos  las  dos  resistencias  en  serie  (las  de  50  W  y  30  W)  a  una  sola  resistencia 
equivalente igual a  Requ = 50 Ω + 30 Ω = 80 Ω. El circuito queda entonces como en la Fig. 20. 
  Luego tenemos dos resistores de 80 W en paralelo. Su resistencia equivalente es 
 
80Ω ⋅ 80Ω
    = 40Ω  
80Ω + 80Ω
 
Llegamos así al circuito equivalente de la Fig. 21. Se calcula inmediatamente la corriente “i”: 
 
120V
    i= = 3A  
40Ω
 
(b) De la Fig. 20 y de la Ec. (24) se deduce que la corriente es la misma a través del resistor de 80  Ω y a 
través de los resistores de 50Ω y 30Ω, o sea 1.5 A. 
(c) La caída de potencial a través de cada resistor es el producto de su resistencia y de la corriente que lo 
atraviesa. Entonces, 
 
  A través del resistor de 80Ω:    caída de potencial = 80 Ω × 1.5 A = 120 V 
 

  A través del resistor de 50Ω:    caída de potencial = 50 Ω × 1.5 A = 75 V 
 

  A través del resistor de 50Ω:    caída de potencial = 30 Ω × 1.5 A = 45 V 
6‐17 

6.6. CONSIDERACIONES DE ENERGÍA EN CIRCUITOS CON FUENTES DE FEM Y RESISTORES. 
  Consideremos el circuito simple de la Fig. 24. Por simplicidad, supongamos que las cargas móviles 
en este circuito son positivas.   
  Cuando  una  carga  móvil  positiva  “dq”  pasa 
por  la  fuente  de  fem  del  borne  negativo  al  positivo, 
su energía potencial aumenta en el valor 
  E 
    dU = q E 
 
donde  E  es  el  voltaje  suministrado  por  la  fuente 
(Véase el cuadro (3) en la página 3). Si el tránsito de   
Fig. 24 
dicha carga ocurre durante un tiempo “dt”, entonces 
la potencia suministrada por la fuente es 
 
dU dq
    P= = E 
dt dt
 
Pero dq/dt es la corriente por el circuito, de modo que la relación anterior se vuelve 
 
   
(27)    P = E i  Potencia suministrada por la fuente de fem 
 
 
  Por otra parte, cuando la carga “dq” pasa por el resistor, moviéndose en el sentido de la corriente, 
su energía potencial disminuye en 
 
    dU = – dq V 
 
donde V es la caída de potencial a través del resistor. La potencia disipada en el resistor es entonces 
 
dU dq
    P= =− V 
dt dt
 
    P = −i V  
 
Usando la relación  V = R i, tenemos entonces 
 
   
(28)    P = – R i2  Potencia disipada en un resistor 
 
 
  La energía eléctrica que se pierde en el resistor se transforma en calor: el resistor se calienta. Se le 
llama calor de Joule. 
  Dado que  E = V  en el circuito de la Fig. 24, la potencia suministrada por la fuente de fem, Ec. (27), 
es numéricamente igual a la potencia disipada en el resistor, Ec. (18). Esto es conservación de energía. 
  La combinación de unidades amperio × voltio (A V) es igual al vatio (W), la unidad de potencia. 
6‐18 

EJEMPLO 6.5. Para el circuito de la Fig. 25, calcular la potencia suministrada por las fuentes de fem, así 
como la potencia disipada por cada resistor. 

 
Fig. 25 
 
  Resolveremos el circuito empleando las 2 leyes de Kirchhoff. El método es el siguiente: 
• Introducir símbolos matemáticos para la corriente a través de cada elemento del circuito. 
• Aplicar la 1ª. ley de Kirchhoff a un nodo del circuito (repetir si es necesario). 
• Divida el circuito en mallas independientes y aplicar la ley de Kirchhoff a cada malla. 
  Observe la Fig. 26. Hemos denotado con “i” la corriente por la fuente de fem de 40 V, y con “i1” e 
 
“i2”  las  corrientes  por  los  demás  elementos, 
tal como vemos en la figura. 
  En el nodo “a” entra la corriente “i” y 
salen  las  corrientes  “i1”  e  “i2”,  de  tal  modo 
que la 1ª. ley de Kirchhoff da 
 

  corriente entrante = corriente saliente 
 

o sea 
 

    i = i1 + i2   
Fig. 26 
 

El nodo “b” es equivalente al nodo “a” y no proporciona una ecuación distinta. 
  Distinguimos dos mallas en el circuito. La primera malla incluye la fuente de 40 V y el resistor de 
20  Ω. Recorramos esta malla empezando del borne negativo de la fuente, en sentido horario. La 2ª. ley de 
Kirchhoff da 
 

    40 – 20 i2 = 0 
 

  La  segunda  malla  incluye  la  fuente  de  20  V  y  todos  los  resistores.  La  ecuación  de  la  2ª.  ley  de 
Kirchhoff es ahora (recorriendo la malla en sentido horario desde el borne negativo de la fuente de 20V): 
 
    20 – 50 i1 – 10 i1 + 20 i2 = 0 
 
Note  que  en  el  resistor  de  20  Ω  hay  una  elevación  de  potencial,  ya  que  lo  estamos  recorriendo  en  el 
sentido opuesto de la corriente i2. 
  Hemos encontrado así tres ecuaciones para las incógnitas i, i1 e i2. Resolviendo tenemos 
 
    i = 3 A,    i1 = 1 A,   i2 = 2 A 
6‐19 

  La potencia dada por la fuente de 20 V es el producto de este voltaje y la corriente “i” a través de 
ella: 
 

    P1 = 40 V ⋅ 3 A = 120 W 
 

La potencia por la fuente de 20 V es 
 

    P2 = 20 V ⋅ 1 A = 20 W 
 

  La potencia suministrada por ambas fuentes es 
 

    120 W + 20 W = 140 W 
 

  La potencia disipada en los resistores es, de acuerdo con la fórmula P = –R i2, 
 

En el resistor de 20Ω:    – (20 Ω) (2 A)2 = – 80 W 
 

En el resistor de 50Ω:    – (50 Ω) (1 A)2 = – 50 W 
 

En el resistor de 10Ω:    – (10 Ω) (1 A)2 = – 10 W 
 

La potencia total disipada en los resistores es 
 

    –80 W –50 W –10 W = –140 W 
 
6.7. FUENTES DE FEM REALES. 
  Hemos definido una fuente de fuerza electromotriz ideal como aquella que mantiene una diferencia 
de  potencial  constante  E  entre  sus  terminales  o  bornes.  Sin  embargo,  en  la  realidad  las  fuentes  de  fem 
poseen  una  resistencia  interna  que  aunque  pequeña,  motiva  que  esta  diferencia  de  potencial  no  sea 
constante. 
  Un modelo apropiado de una fuente de fem real consiste en una fuente de fem ideal en serie con un 
resistor  interno  cuya  resistencia  denotaremos  con  “r”.  La  Fig.  27  muestra  un  circuito  simple  con  una 
fuente de fem real enmarcada en el rectángulo con líneas punteadas. 
 
  Dado  que  las  resistencias  R  y  “r”  están  en  serie,  la 
corriente suministrada por la fuente real es 
 
E
    i=  
r+R
 

Esta  es  menor  que  la  corriente  en  el  caso  de  que  la  fuente 
de fem fuera ideal, o sea  i = E/R; Una fuente ideal de fem 
tendría una resistencia interna r = 0.   
  Entre  los  bornes  de  la  fuente  de  fem  real,  Fig. 27 
representados por los puntos “a” y “b” en la Fig. 27, existe  
un voltaje igual a 
⎛ ⎞
E ⎜ 1 ⎟
    Vab = E − r i = E − r =E ⎜ ⎟
r+R ⎜ 1+
r

⎝ R ⎠ 
 

Este es el voltaje suministrado por la fuente de fem real. Notemos que depende de la resistencia interna y 
de la resistencia R en este circuito. 
6‐20 

 
6.8. PROBLEMAS. 
 
1. Calcular la corriente eléctrica como dq/dt. 
 
2. ¿Qué longitud debería tener un alambre conductor de hierro, cuya área transversal es de 3 mm2, para 
que su resistencia eléctrica fuese de 1 W? 
 
3.  Un  alambre  conductor  de  plata,  que  forma  parte  de  un  circuito,  transporta  una  corriente  de  5  mA. 
¿Cuál es la caída de potencial a través de un pedazo de 10 cm de longitud de este alambre? El área de la 
sección transversal del alambre es de 3 mm2. 
 
4. Un cilindro sólido recto de aluminio tiene una sección circular de diámetro 0.8 cm. ¿Cuál es la longitud 
del  cilindro  si  la  resistencia  eléctrica  entre  sus  extremos  es  de  3  Ω?  La  resistividad  del  aluminio  a 
temperatura ambiente es de 2.75 (10–8) Ω m. 
 
5. Un alambre de cobre de sección circular de radio r = 2 mm  transporta una corriente de 8 A. Calcular la 
velocidad de deriva vd de los electrones. La densidad de electrones libres del cobre es 
n = 8.5 (1028) / m3. 
 
6. Calcular la resistencia equivalente entre los puntos “a” y “b” para los siguientes arreglos de resistores. 
(a) 
 
(b) 
 
(c) 
6‐21 

 
7.  Emplee  las  leyes  de  Kirchhoff  para  calcular  la  corriente  y  la  caída  de  potencial  a  través  de  cada 
resistencia.  Calcule  también  la  potencia  suministrada  (o  recibida)  por  las  fuentes  de  fem,  y  la  potencia 
disipada en cada resistencia. 
(a) 

     
 
(b) 
 

     
(c) 
 

     
 
7‐1 

|||||||||| 
CAPÍTULO 7 
   
  CAMPO MAGNÉTICO 
 
 
 
7.1. FENÓMENOS MAGNÉTICOS. 
  Los  fenómenos  magnéticos  más  directos  con  los  que  tenemos  contacto  en  nuestras  experiencias 
cotidianas son los relacionados con los imanes permanentes. Los imanes son capaces de atraer limaduras 
de  hierro  y  otros  materiales  magnéticos.  Al  esparcir  limaduras  de  hierro  en  la  vecindad  de  un  imán 
permanente,  las  limaduras  forman  un  patrón  característico  como  vemos  en  la  Fig.  1,  formando  curvas 
ovoidales cerradas. Si en lugar de limaduras colocamos una aguja imantada o brújula en diversos puntos 
alrededor del imán, la aguja se orienta en cada punto tangencialmente a las líneas mostradas en la Fig. 1. 
Estas  líneas  se  denominan  lineas   
de campo magnético. 
  El  efecto  magnético  del 
imán  sobre  los  objetos  a  su 
alrededor  se  describe  mediante 
un  campo  vectorial  denominado 
campo  magnético,  denotado 
usualmente con el símbolo “B”. 
  También  se  producen 
efectos  magnéticos  sobre 
limaduras  o  agujas  magnéticas 
cuando  se  colocan  en  la 
proximidad  de  un  alambre  que   
conduce una corriente eléctrica.  Fig. 1 
El alambre produce un campo magnético cuyas líneas de campo son círculos con centro en el alambre y 
planos perpendiculares al mismo (Fig. 2). 
  Los  imanes,  limaduras,  agujas  magnética  y  alambres   
son  cuerpos  macroscópicos.  Sus  interacciones  magnéticas  se 
deben, en un nivel fundamental, a las partículas atómicas que 
los constituyen. 
  De  hecho,  los  campos  magnéticos  son  generados  por 
cargas  eléctricas  en  movimiento,  que  ejercen  fuerzas 
magnéticas  sobre  otras  cargas  eléctricas,  también  en 
movimiento.  Otra  propiedad  de  las  partículas  atómicas,  el 
espín,  también  está  asociada  con  la  generación  de  campos 
magnéticos. 
  En  la  siguiente  sección  empezaremos  nuestro 
tratamiento  del  magnetismo  discutiendo  la  interacción   
magnética entre cargas puntuales.  Fig. 2 
 
 
7‐2 

 
7.2. INTERACCIÓN ENTRE CARGAS ELÉCTRICAS PUNTUALES. 
  Sean  q1 y  q2 dos cargas eléctricas puntuales en movimiento,  R el vector separación entre ellas, y  v1 
y v2 sus respectivas velocidades (Fig. 3). 
  Existen  dos  clases  de  interacciones  entre  las   
cargas, una eléctrica y otra magnética. 
  La  fuerza  eléctrica  entre  ambas  cargas  no 
depende  de  las  velocidades  de  las  mismas.  De 
acuerdo  con  la  ley  de  Coulomb,  la  fuerza  eléctrica 
que ejerce q1 sobre q2 es 
 
1 q1q 2 ˆ
(1)    Fe = ⋅ R 
4 πε0 R 2  
   
Fig. 3 
donde “1/4πε0” es una constante y  R̂  es el vector 
unitario en la misma dirección que R. 
  Como vemos, la fuerza eléctrica disminuye con el cuadrado de la distancia entre las cargas. Si en (1) 
intercambiamos los índices “1” y “2” y además cambiamos “ R̂  “ por “–  R̂  “ obtenemos “– F”, es decir, la 
fuerza  eléctrica  que  ejerce  q1  sobre  q2  es  igual  y  opuesta  a  la  que  ejerce  q2  sobre  q1.  La  interacción 
eléctrica, pues, obedece la tercera ley de Newton. 
  La  fuerza  magnética  entre  ambas  cargas  es  más  compleja  que  la  eléctrica.  Tiene  en  común  con  la 
fuerza eléctrica el hecho de que depende también del producto de las cargas, “q1q2” y disminuye con el 
cuadrado de la distancia, pero sus propiedades direccionales no son tan simples. 
  La fuerza magnética que q1 ejerce sobre q2 viene dada por la expresión 
 
μ q q v × ( v1 × R )
(2)    Fm = 0 ⋅ 1 2 2  
4π R3
 
donde “μ0/4π” es una constante cuyo valor, como veremos, proviene de la definición del amperio, unidad 

electromagnética básica en el Sistema Internacional. Resulta ser μ0 = 4π (10–7) N‐s2/C2. 
  La  fuerza  (2)  es  perpendicular  tanto  a  v2  como  al  plano  determinado  por  v1  y  r.  Esta  fuerza  no 
obedece la tercera ley de Newton, pues si en (2) intercambiamos los índices “1” y “2” y cambiamos “ R̂  “ 
por “–  R̂ “, no obtenemos “– Fm ” para la fuerza de q2 sobre q1. 
  Por otra parte, las expresiones (1) y (2) son válidas solamente en el caso en que las velocidades de 
las cargas son pequeñas en comparación con la velocidad de la luz,  c ≈ 300 000 km/s. En caso negativo 
las fuerzas son más complejas. 
  Notemos que la fuerza magnética es nula cuando una o ambas cargas están en reposo (v1 = v2 = 0). 
  La cosa no para allí. Si las velocidades varían con el tiempo, entonces las cargas emiten radiación 
electromagnética:  generan  campos  eléctricos  y  magnéticos  que  dependen  de  la  posición  y  del  tiempo,  y 
que  se  propagan  con  velocidad  “c”.  Estos  campos  decaen  con  el  inverso  de  la  distancia  “R”,  o  sea  más 
lentamente que la variación implicada en (1) y (2). 
7‐3 

 
  Ahora  bien,  hay  una  circunstancia  que  nos  permite  separar  los  fenómenos  eléctricos  de  los 
magnéticos en muchos escenarios especiales, y es ésta: 
 

  La fuerza magnética entre dos cargas es mucho menor que la fuerza eléctrica. 
 

Usemos las expresiones (1) y (2) para obtener el cociente  |Fm| / |Fe|. Sea  v1  ≈  v2  =  v,  y olvidémonos de 


los  productos  vectoriales.  Notemos  que  las  cargas  se  cancelan,  lo  mismo  que  el  factor  1/R2,  común  a 
ambas fuerzas. Resulta 
 
μ0 q1q 2 ⋅ v ⋅ v
Fm 4π R2
    = = ε0 μ0 ⋅ v 2  
F e 1 q q
1 2
4 πε0 R 2
 
  Ahora  bien,  la  luz  es  un  fenómeno  electromagnético  y  la  velocidad  de  la  luz  se  relaciona  con  las 
constantes ε0 y μ0 mediante 
 
1
    c=  
ε0 μ0
 
de modo que 
 
Fm v2
(3)    =  
Fe c2
 
  Ahora bien, las velocidades de deriva de los portadores de carga en los conductores son del orden 
de  1  mm/s.  Para  estas  velocidades  el  cociente  (3)  es  del  orden  de  10–  23,  así  que  las  fuerzas  magnéticas 
entre  cargas  pueden  despreciarse  en  muchos  casos  prácticos.  Si  las  velocidades  son  próximas  a  la  de  la 
luz, ambos tipos de fuerzas son comparables. 
  Por  esta  razón,  el  estudio  del  electromagnetismo  suele  empezar  con  la  electrostática,  en  la  que  se 
supone  que  las  cargas  están  en  reposo  o  bien,  si  se  trata  de  cargas  libremente  móviles  en  el  espacio 
tridimensional, poseen velocidades muy pequeñas en comparación con la de la luz. 
  Aunque la fuerza magnética es pequeña, puede ser apreciable en una situación en la que un enorme 
número  de  cargas  móviles  influencian  a  otra  carga  móvil.  Esto  es  lo  que  ocurre  en  un  conductor  que 
transporta  corriente  eléctrica.  El  número  de  electrones  libres  en  un  conductor  de  cobre  es  del  orden  de 
1028  por  metro  cúbico.  Si  el  minúsculo  efecto  magnético  producido  por  cada  uno  de  estos  electrones  se 
multiplica por  1028, se obtiene un efecto fácilmente medible en el laboratorio. Esto no es otra cosa que la 
fuerza magnética que experimenta una carga móvil cerca de un conductor por el que circula una corriente 
eléctrica. Estos fenómenos se estudian en la magnetostática, en la que se supone que las corrientes eléctricas 
no varían con el tiempo. 
7‐4 

7.3. DEFINICIÓN DEL CAMPO MAGNÉTICO. 
  Así  como  la  interacción  eléctrica  entre  dos  cargas  puntuales  se  describe  convenientemente  en 
términos  del  concepto  de  vector  de  intensidad  eléctrica  E  (o  brevemente,  campo  eléctrico),  la  interacción 
magnética también se describe en términos del llamado  vector de inducción magnética  B o  vector densidad de 
flujo magnético (se suele llamarle brevemente: campo magnético). 
  Escribamos  de  nuevo  la  expresión  de  la  fuerza  magnética  que  una  carga  puntual  q1  ejerce  sobre 
otra carga puntual q2 separada de aquella por el vector R: 
 

μ q q v × ( v1 × R )
    F= 0⋅ 1 2 2
4π R3  
 

Pongámosla en la forma 
 

⎛ μ q v × R) ⎞
(4)    F = q2v2 × ⎜ 0 ⋅ 1 1 ⎟
⎝ 4π R3 ⎠ 
 

  La cantidad en paréntesis se define como el campo magnético producido por la carga q1 en el punto 
a separación R de la misma, donde se encuentra q2. 
 
  Campo  magnético  producido  por  una  carga 
μ qv×R puntual “q” que se mueve con velocidad “v”, en un 
(5)    B= 0  
4π R 3 punto a separación R de la carga. 
 
 
Usando (5), la relación (4) se escribe como 
 
    F = q2 v2 × B1 
 
donde  hemos  designado  ahora  con  “B1”  el  campo  magnético  producido  por  la  carga  q1.  Abstrayendo 
ahora ambos índices “1” y “2” tenemos 
 
  Fuerza  magnética  que  experimenta  una  carga 
(6)    F = q v × B  puntual  “q”  que  se  mueve  con  velocidad  “v” 
  dentro de un campo magnético “B”. 
 
  Como vemos, esta fuerza es perpendicular tanto al vector velocidad v como al campo magnético B. 
De  hecho,  la  relación  (6)  es  la  base  de  la  definición  más  general  del  campo  magnético.  Es  decir,  (6)  no 
solamente es aplicable a la interacción magnética entre dos cargas puntuales, sino en general también a la 
interacción  entre  magnética  entre  una  carga  puntual  q  y  un  campo  magnético  B  generado  por  otras 
fuentes distintas a la carga puntual. Pongámoslo así: 
 
(7)  Se define en general el campo magnético  B de tal manera que la fuerza magnética sobre una carga 
puntual q que se mueva con velocidad v en el seno del campo venga dada por la expresión (6). 
7‐5 

 
  Otros nombres para B son: vector de inducción magnética o vector densidad de flujo magnético. 
  De la Ec. (6) sacamos las unidades físicas del campo magnético, a saber, 
 
N N
    = = T = tesla  
m A⋅m
C⋅
s
 
Otra unidad muy usada es el gauss, definido por 
 
    1 gauss = 10–4 tesla    1 tesla = 10 000 gauss 
 
EJEMPLO 7.1.  En la Fig. 4 se han trazado unas pocas líneas de campo del campo magnético generado por 
una carga eléctrica que se mueve con velocidad constante horizontal v. 
  Este campo vale 
 
μ0 q v × R
    B=
4π R 3  
 
donde R es el vector separación entre la carga y el punto campo considerado. 
 

 
Fig. 4 
 
  Las líneas de campo son círculos concéntricos, cuyos centros se encuentran en la trayectoria recta de 
la carga, y cuyos planos son perpendiculares a esta recta. Note que la carga no produce campo magnético 
en los puntos de esta recta (donde  v  × R = 0), y que el sentido de giro del campo y el sentido del vector 
velocidad cumplen la regla de la mano derecha. 
7‐6 

 
EJEMPLO  7.2.   En  un  momento  dado,  dos  cargas  del  mismo  signo  (digamos  positivas)  se  mueven  en 
trayectorias rectas paralelas con velocidades iguales y opuestas. ¿Qué interacción magnética existe entre 
ellas cuando pasan una frente a la otra? 
  Coloquemos unos ejes X, Y y Z como se 
muestra  en  la  Fig.  B.  El  campo  magnético  B 
producido  por  la  carga  inferior  en  el  punto 
donde  se  halla  la  carga  superior  apunta  en  la 
dirección  +X.  Por  lo  tanto,  usando  la  Ec.  (6), 
tenemos que la fuerza sobre la carga superior 
apunta en la dirección +Z. 
  Haciendo un análisis similar obtenemos 
que la fuerza magnética sobre la carga inferior, 
debida  a  la  superior,  apunta  en  la  dirección 
negativa del eje Z.  
  Se concluye que las cargas se repelen. Si 
las  velocidades  estuvieran  en  el  mismo   
Fig. 5 
sentido, las cargas se atraerían. 
 
EJEMPLO 7.3.  En una región del espacio existe un campo magnético uniforme B = 0.35 T dirigido en el 
sentido del Eje Y. En un momento dado, una carga puntual  q = 8  μC pasa por el origen de coordenadas 
con  una  velocidad  v  cuyas  proyecciones  sobre  el  plano  XY  y  el  eje  Z  tienen  los  valores  indicados  en  la 
figura A. Calcular la fuerza magnética sobre esta carga en esta situación. 
‐‐‐‐‐‐‐‐‐‐‐‐‐‐‐‐‐‐‐‐‐‐‐‐‐‐‐‐‐‐‐‐‐‐‐‐ 
  Debemos expresar los vectores  B y   
v vectorialmente en la base cartesiana 
{i,  j,  k}  y  luego  hacer  el  producto 
vectorial 
 
    F = q v × B 
 
El vector B, en unidades S.I., es 
 
    B = 0.35 j 
   
Por otra parte, las componentes X y Y del  Fig. 6 
vector v son, en unidades S.I., 
 
v x = 3 ⋅ 10 3 sen 37° = 1.8 ⋅ 10 3
     
v y = 3 ⋅ 10 3 cos 37° = 2.4 ⋅ 10 3
 
La componente Z es inmediata, igual a 
 
     m/s 
7‐7 

 
con lo que 
 
    v = (1.8 i + 2.4 j + 5 k) 103    (m/s) 
 
  Tenemos entonces 
 
    F = q v × B = (8 10–6) (1.8 i + 2.4 j + 5 k) 103 × (0.35 j)      (N) 
 
Usando las relaciones  i × j = k,  j × j = 0  y  k × j = – i tenemos 
 
    F = (8 10–6) (1.8 ⋅ 0.35 k – 5 ⋅ 0.35 i)    (N) 
 
    F = ( – 14 i + 5.04 k ) 10–6 N 
 
EJEMPLO 7.4.  Una carga  q = 5 μC  se desplaza a lo largo del Eje de las X, en el sentido positivo, con una 
velocidad constante de  4 (106) m/s. En el momento en que la carga pasa por el origen de coordenadas, 
¿Cuál es el campo magnético que produce en el punto (0.5 m, 1.2 m, 0)? 
 
  El  campo  magnético  de  la  carga  viene  dado   
por la Ec. (5‐p4), o sea 
 
μ0 q v × R
B=
    4π R 3  
 
donde R es el vector del punto considerado relativo 
a la carga (Véase la Fig. 22). 
  Se tienen las expresiones vectoriales 
 
    v = 4 (106) i 
 
    R = 0.5 i + 1.2 j    R = 1.3   
Fig. 22 
 
Además,    v × R = 4.8 (106) k. Entonces, usando μ0/4π = 10–7 (unidades S.I.), 
 
10 −7 ⋅ 5(10 −6 )(4.8(106 ))
    B = Bz k = k 
1.33
 
    B = 10.92 (10–7) k   tesla 
 
 
7‐8 

 
7.4. MOVIMIENTO DE CARGAS DENTRO DE UN CAMPO MAGNÉTICO. FUERZA DE LORENTZ. 
  De la expresión de la fuerza que sufre una carga puntual dentro de un campo magnético  o sea 
“F  =  q  v  ×  B”,  se  deduce  que  tal  fuerza  es  perpendicular  tanto  al  vector  velocidad  v  como  al  campo 
magnético  B. Por ser perpendicular a  v, la fuerza magnética no puede alterar la rapidez de la carga, esto 
es, el campo magnético solamente puede  desviar a la carga, sin producir ningún aumento o disminución 
en la magnitud de su velocidad. Dentro de un campo magnético, las cargas no sufren ninguna variación 
de su energía cinética. 
  Si  la  carga  se  encuentra  en  una  región  donde  existen  tanto  un  campo  eléctrico  E  como  uno 
magnético B, la fuerza que experimenta es la suma vectorial de las fuerzas eléctrica y magnética, es decir,  
 
(88)    F = qE + q v × B 
 
Esta se denomina Fuerza de Lorentz. 
  Uno puede usar una combinación de un campo eléctrico constante y un campo magnético constante 
para obtener un  selector de velocidades, que es un dispositivo simple para obtener un haz de partículas 
cargadas que tengan una velocidad bien determinada. Veamos. 
 

 
Fig. 8 
 
  Se  calienta  un  filamento  para  desprender  electrones  de  él.  Estos  electrones  se  aceleran  hacia  la 
derecha al establecer una diferencia de potencial “V” entre el filamento y un primer filtro colocado justo 
antes  de  un  capacitor  de  placas  paralelas.  En  el  espacio  dentro  del  capacitor  se  establece  un  campo 
eléctrico  E dirigido hacia abajo, y también un campo magnético  B dirigido en la Fig. 7.7 hacia dentro del 
papel. 
  El  electrón  entra  a  la  región  interna  del  capacitor  con  cierta  velocidad  horizontal.  Allí  sufre  una 
fuerza  eléctrica  hacia  arriba  de  magnitud  “eE”,  y  una  fuerza  magnética  perpendicular  a  su  velocidad 
horizontal y al campo magnético, en la dirección hacia abajo, de magnitud “e v B”. 
  Para  que  el  electrón  prosiga  en  línea  recta,  sin  ser  desviado  por  estos  campos,  debemos  tener 
igualdad de ambas fuerzas, o sea 
 
     
 
de donde se obtiene la velocidad de los electrones que no son desviados, en la forma 
 
E
(89)    v=  
B
7‐9 

 
7.5. EL EFECTO HALL. 
  Consideremos el siguiente arreglo experimental: 
  Una cinta plana de anchura “s” y espesor “t” se coloca dentro de un campo magnético uniforme  B, 
con su plano perpendicular al campo, como se muestra en la Fig. 7.8. La cinta está hecha de un material 
conductor cuyos portadores de carga son, digamos, electrones. 
  Se manda una corriente “i” a lo largo de la cinta. Dado que el área de la sección transversal de la 
cinta es A = s t, tendremos por la fórmula de la nave 
 

(77)    i = n A v e = n s t v e 
 

donde “n” es el número de electrones libres por unidad de volumen, “v” es la velocidad de arrastre de los 
electrones libres, y “e” es la carga del electrón. 
 

 
Fig. 9 
 

  Los  electrones  libres  sufren  dentro  del  campo  magnético  una  fuerza  magnética  dirigida  (según  la 
fórmula F = q v × B) hacia arriba. Tal fuerza motiva que se vaya formando una acumulación de electrones 
en el borde superior de la cinta, y al mismo tiempo una deficiencia de electrones en el borde inferior, como 
vemos en la Fig. 7.9. Mientras esto ocurre, se va formando   
también un campo eléctrico  E dirigido hacia arriba, hasta 
que  se  llega  a  una  situación  de  equilibrio  en  la  que  el 
campo eléctrico ya no permite más acumulación de carga 
en  los  bordes.  Esto  último  ocurre  cuando  la  fuerza 
eléctrica  (hacia  abajo)  se  iguala  con  la  fuerza  magnética 
 
(hacia arriba), o sea cuando  
Fig. 10 
(89)    e E = e v B    o     E = v B 
 

El campo eléctrico se puede obtener midiendo la diferencia de potencial “V” entre los bordes de la cinta, y 
dividiendo ésta por la distancia “s”, en la forma  “E = V/s”. Por otra parte, despejando la velocidad de 
(88), tenemos “v = i/nste”. Sustituyendo ahora lo anterior en (88) y despejando “n” se llega a 
 
iB
(89)    n=  
Vte
 
Tenemos así un modo de medir la propiedad “n” con ayuda de este efecto, denominado Efecto Hall. 
7‐10 

 
EJEMPLO 7.4.  Espectrómetro de masas. 
  Un modo de determinar la masa de una partícula ionizada es mediante un dispositivo denominado 
el  espectrómetro  de  masas.  Consta  esencialmente  de  un  canal  por  donde  se  introducen  los  iones  a  una 
región donde existe un campo magnético constante  B (Fig. A). La velocidad  v de los iones al entrar a esta 
región es conocida, al igual que su carga eléctrica q. Relacionar la masa del ión con los demás parámetros. 
 

 
Fig. 11 
‐‐‐‐‐‐‐‐‐‐‐‐‐‐‐‐‐‐‐‐‐‐‐‐‐‐‐‐‐‐‐‐‐‐‐‐‐ 
  En  la  Fig.  A,  el  ión  entra  al  campo  magnético  por  la  parte  inferior  izquierda,  con  velocidad  v.  El 
campo  B  apunta  hacia  fuera  del  papel,  perpendicularmente.  Una  vez  en  el  seno  del  campo,  sufre  una 
fuerza magnética dada por 
 
    F = q v × B 
 
Dado que esta fuerza es perpendicular tanto a  v como a  B, tenemos que en el punto de entrada del ión la 
fuerza apunta horizontalmente hacia la derecha (hacia el punto C en la figura). 
  La fuerza es siempre perpendicular a la velocidad  v, la cual no cambia su magnitud, de tal manera 
que apunta siempre hacia un mismo punto C. Se trata aquí de una fuerza centrípeta, y el movimiento del 
ión  es  precisamente  un  movimiento  circular  uniforme.  De  la  segunda  ley  de  Newton  tenemos  en  la 
dirección radial la ecuación 
 
v2
    Fr = ma r qvB = m  
r
 
donde “r” es el radio de la trayectoria circular del ión. 
  Despejando la masa, 
 
qB
    m=  
vr
 
Vemos que  los  radios  de las  trayectorias son  tanto menores  cuanto  mayores son  las  masas  de los iones. 
Esta expresión nos permite calcular la masa a partir de la medición de q, B, v y r. 
7‐11 

 
7.6. LA LEY DE BIOT‐SAVART. 
  Como ya hemos dicho, los campos magnéticos son producidos por cargas eléctricas en movimiento. 
El campo magnético que produce una sola carga “q”, en un punto P localizado por el vector separación R 
relativo a la carga, es 
 
μ q v×R  
[(5)]       B= 0  
4π R 3
 
donde  v es el vector velocidad de la carga. Este campo 
es perpendicular tanto a v como a R. 
  Un  haz  o  enjambre  de  partículas  cargadas  en 
movimiento  produce  un  campo  magnético  que  es  la 
suma vectorial de los campos magnéticos individuales 
producidos por cada carga en el haz o enjambre. 
  En  particular,  si  tal  enjambre  de  partículas 
 
cargadas móviles está confinado dentro de un alambre  Fig. 12 
conductor,  a  manera  de  constituir  una  corriente 
eléctrica en éste, existirá un campo magnético asociado. 
  La  ley  de  Biot‐Savart  es  una  expresión  matemática  para  el  campo  magnético  producido  por  un 
alambre  conductor  por  el  que  circula  una  corriente  eléctrica.  Procederemos  a  deducir  esta  expresión 
usando la ec. (5) y la fórmula de la nave (ec. (12), página 6‐9). 
 

 
Fig. 13 
 
  Observemos  la  Fig.  7.3.  Se  representa  un  alambre  conductor  que  transporta  una  corriente  “i”.  Un 
punto  cualquiera  de  este  alambre,  como  el  punto  P’  mostrado,  se  denomina  punto  fuente.(su  vector  de 
posición  se  denota  con  “  r’  ”  Por  otra  parte,  el  punto  donde  deseamos  evaluar  el  campo  magnético  del 
alambre  (punto  P  en  la  figura)  se  denomina  punto  campo  (con  vector  de  posición  “  r  “  (Estas 
designaciones son análogas a las que usamos en relación con los campos eléctricos). 
7‐12 

  Marquemos  en  el  punto  P’  un  elemento  de  arco  vectorial  dr’  (que  llamaremos  elemento  de 
corriente).  Calcularemos  el  campo  dB  producido  en  P  por  este  elemento  y  luego  sumaremos 
vectorialmente  los  campos  diferenciales  de  todos  los  elementos  que  componen  el  alambre,  es  decir, 
integraremos sobre todo el alambre. 
 

  La  Fig.  7.4  es  una  vista  ampliada   


del elemento de corriente. Por el alambre 
circulan  electrones  libres,  cada  uno  con 
carga  eléctrica  “e”  y  velocidad  “v”.  El 
número  de  electrones  libres  contenidos 
en el pedazo de alambre de longitud 
|dr’| =  ds’ y área transversal A es 
 
    ΔN = n A ds’ 
 
donde  “n”  es  la  densidad  de  electrones 
libres. 
  Cada  uno  de  estos  electrones 
 
produce en P un campo magnético dado  
Fig. 14 
por la Ec. (5) en la forma 
 
μ0 e v × R
     
4π R 3
 
(Estamos  suponiendo  que  el  elemento  ds’  es  tan  pequeño  que  todos  los  electrones  se  encuentran 
distanciados por el mismo vector, R, del punto P). 
  Entonces  el  campo  magnético  producido  por  todos  los  electrones  contenidos  en  el  elemento  de 
corriente dr’ es 
 
μ e v×R
    dB = nAds′ ⋅ 0  
4π R 3
 
Dado que dr’ y v tienen la misma dirección, de tal manera que  
 
    ds’ v = dr’ v 
 
podemos escribir dB como 
 
μ e dr ′ × R
    dB = nAv ⋅ 0  
4π R 3
 
  Usando ahora la fórmula de la nave (o sea  i = nAve), tenemos 
 
μ i dr ′ × R
    dB = 0  
4π R 3
7‐13 

  Finalmente, el campo magnético producido por todo el alambre es 
 
  Ley de Biot‐Savart 
μ i dr ′ × R

(8)    B= 0   Campo magnético producido por un alambre 
4π R3 que transporta una corriente “i” 
 
 
7.7. CAMPO MAGNÉTICO GENERADO POR UN HILO RECTO INFINITO DE CORRIENTE. 
  Como aplicación de la Ley de Biot y Savart, calcularemos el campo magnético producido por un 
alambre recto conductor, de longitud infinita, en un   
punto P arbitrario del espacio. Supondremos que el 
alambre transporta corriente eléctrica “i”. 
  Coloquemos  el  alambre  como  se  muestra  en 
la Fig. 7.5, a lo largo del Eje Y. Deseamos calcular el 
campo B en un punto cualquiera P(x, 0) del Eje X. 
  Dado que el campo magnético posee simetría 
rotacional alrededor del Eje Y, el campo B tendrá la 
misma  magnitud  en  todo  punto  de  un  círculo  de 
radio “x” y centro en el origen  O. El plano de este 
círculo es perpendicular al alambre. 
  En  la Fig.  7.5  hemos  indicado  las  cantidades 
implicadas  en  el  cálculo:  el  elemento  de  corriente 
dr’, la coordenada (variable)  y’ de dicho elemento, 
y  el  vector  separación  R  que  va  del  elemento  de 
 
corriente al punto campo P.  Fig. 15 
  Para calcular la integral 
 

μ i dr ′ × R
    B= 0
4π R3
∫  
 

expresaremos el integrando vectorial en términos de la base vectorial cartesiana {i, j, k}. 
  La longitud del elemento dr’ es “ dy’ ”, y su dirección es la del unitario j, así que 
 

    dr’ = dy’ j 
 

Por otra parte, 
 

    R = x i – y’ j 
 

Tenemos entonces 
 

    dr′ × R = (dy′ j) × (x i − y′ j) = −xdy′ k  


 

Por tanto, el campo magnético diferencial “dB” producido por el elemento de corriente en P es 
 
μ i dr ′ × R μ i xdy′
    dB = 0 ⋅ =− 0 ⋅ k 
4π 4π
( )
R 3 3/2
x 2 + y ′2
7‐14 

  Hemos usado aquí  R = x 2 + y′2 . 
  Vemos que el campo diferencial  dB tiene la dirección del vector “–  k”, es decir, está en la dirección 
negativa del Eje Z. En la Fig. 7.5, tal dirección es la perpendicular al papel, hacia dentro. 
  El campo magnético del alambre es 
 
μ ix dy′
    ∫
B = dB = −k 0 ⋅ ∫  

( )
3/2
x 2 + y ′2
 

Los límites de integración deben abarcar todo el alambre: desde y’ = –∞ hasta y’ = ∞. 
 
a
⎡ ⎤ ⎛ ⎞
μ ix y′ μ i 2a μ i
    B = −k 0 ⋅ lím ⎢ ⎥ = −k 0 ⋅ lím ⎜ ⎟ = −k 0 ⋅ 2  
4 π a →∞ ⎢ x 2 x 2 + y′2 ⎥ 4 π x a →∞ ⎜ x 2 + a 2 ⎟ 4π x
⎣ ⎦ −a ⎝ ⎠
 

  Obtenemos el resultado   
 
μ i
(9)    B = − 0 ⋅k  
2π x
 

Veamos  la  Fig.  7.5  de  nuevo.  El  campo  B  es 


perpendicular tanto al alambre como a la recta que une 
el  alambre  y  el  punto  campo  P  (o  sea  el  Eje  X  en  esa 
figura). La magnitud de  B decrece con el inverso de la 
distancia “x” del alambre al punto P. 
  Conviene  hacer  aquí  un  cambio  de  símbolo: 
denotaremos  con  “r”  la  distancia  del  punto  campo 
considerado  al  alambre  Por  otra  parte,  en  lo  sucesivo 
manejaremos  la  Ec.  (9)  casi  siempre  en  forma  escalar 
(claro  está,  teniendo  siempre  presente  tanto  que  el 
campo  magnético  es  un  vector,  como  también  la 
 
dirección del mismo, circundando al alambre).  Fig. 16 
  Con ello tendremos: 
 
  Campo magnético de un alambre recto infinito que transporta una corriente 
 
eléctrica “i”, en un punto a una distancia “r” del alambre. La dirección del 
μ0 i
(10)        B =   campo es tangente al círculo horizontal que pasa por el punto campo y tiene 
2π r
centro en el alambre. El campo “gira” alrededor del alambre obedeciendo la 
  regla de la mano derecha con respecto a la dirección de la corriente eléctrica. 
 
  Las  líneas  de  fuerza  de  este  campo  magnético  son  círculos  centrados  en  el  alambre  y  situados  en 
planos perpendiculares al mismo (Fig. 7.6). 
  La  expresión  (10)  es  de  suma  importancia  pues,  entre  otras  cosas,  se  usa  en  la  definición  de  la 
unidad S.I. de corriente eléctrica, como veremos más adelante. 
7‐15 

 
EJEMPLO 7.6.  Un alambre recto transporta una corriente i = 8.5 A. El alambre corre perpendicularmente 
a un plano cartesiano XY, como se muestra en la    
Fig.  A,  con  la  corriente  dirigida  hacia  el  lector.  El 
lado  de  la  cuadrícula  mide  un  metro.  Calcular  el 
campo  magnético  del  alambre  en  el  punto  cuyas 
coordenadas  son  (9,  3)  m.  Suponer  el  alambre 
infinito. 
‐‐‐‐‐‐‐‐‐‐‐‐‐‐‐‐‐‐‐‐‐‐‐‐‐‐‐‐‐‐‐‐‐‐ 
  El  problema  pide  el  vector  B.  Sabemos  que 
este vector es tangente al círculo cuyo centro está en 
el  alambre  y  que  pasa  por  el  punto  campo 
considerado.  Sea  “u”  el  vector  unitario  en  la 
dirección de B  (Fig. A). 
  El  campo  se  puede  expresar  vectorialmente 
en la forma   
   
μ i Fig. 17 
(E1)    B = Bu = 0 u  
2 πr
 
Un modo conveniente de obtener el vector unitario u es el siguiente: 
• Primeramente obtenemos el vector unitario en el sentido de la corriente (en este caso será el vector 
unitario básico k, puesto que la corriente va en la dirección positiva del Eje Z). 
• Luego definimos en el plano XY el vector separación “r”, que va desde el alambre hasta el punto 
campo considerado. 
• Hacemos  el  producto  vectorial    “k  ×  r”,  cuya  dirección  es  perpendicular  tanto  a  k  como  a  r,  es 
decir, precisamente en la dirección de u ó B. 
• Dividimos el vector “k × r” por su magnitud “|k × r| =  r” para hacerlo unitario: 
 
k×r
(E2)    u=  
r
 
Poniendo (E2) en (E1) obtenemos 
 
μ0 i
(E3)    B= (k × r )
2 πr 2    
 
Apliquemos esta expresión al caso presente. Se tiene 
 
  r = 6 i – 3 j        r = 6 2 + 32 = 6.71     k × r  = k × (6 i – 3 j) = 6 j + 3 i 
 
con lo que 
 
μ0 i 8.5
    B= ( k × r ) = 2(10 −7 ) (3i + 6 j) = (1.13i + 2.26 j) ⋅ 10 −7 (tesla)
2 πr 2
6.712  
 
7‐16 

 
EJEMPLO 7.7.  Tres alambres rectos paralelos portan corrientes de 4 A, 5 A y 10 A. Los alambres corren 
perpendicularmente  a  un  plano  XY,  y  los  sentidos  de  sus  corrientes  son  los  indicados  en  la  Fig.  A. 
Suponer que el lado de la cuadrícula mide un metro. Calcular el campo magnético total en el origen de 
coordenadas, debido a los tres alambres. Suponer que los alambres son de extensión infinita. 
‐‐‐‐‐‐‐‐‐‐‐‐‐‐‐‐‐‐‐‐‐‐‐‐‐‐‐‐‐‐‐‐‐‐‐‐‐‐‐‐‐‐‐‐ 
  Hay  que superponer  vectorialmente los   
campos  magnéticos  de  los  tres  alambres. 
Designémoslos con los números 1, 2 y 3 como 
se muestra en la Fig. A. 
  Para  expresar  cada  campo  vectorial‐
mente usaremos la expresión 
 
μ0 i
B= ( ±k × r )
    2 πr 2  
 
obtenida  en  el  ejemplo  anterior.  En  ella,  el 
vector  básico  k  se  usa  si  la  corriente  va  en  la 
dirección  positiva  del  Eje Z;  en  caso contrario 
debe usarse el vector “– k”. 
  Para el alambre 1, en unidades S.I., 
 
  r = 7i  –8j    r = 10.63 
  Fig. 18 
   – k × r = – 7 j – 8 i   
 
μ0 i 4
    B1 = ( −8i − 7 j) = 2(10 −7 ) ( −8i − 7 j) = ( −0.56i − 0.49 j)10 −7   (tesla) 
2 πr 2
10.63 2

 
Procediendo análogamente con el campo B2 tenemos 
 
    r = – 8 i – 6 j    r = 10   – k × r = – k × (– 8 i – 6 j) = 8 j – 6 i 
 
μ0 i 5
    B2 = ( −6i + 8 j) = 2(10 −7 ) 2 ( −6i + 8 j) = ( −0.6i + 0.8 j) 10 −7  
2 πr 2
10
 

Finalmente, es obvio de la Fig. A que el unitario en la dirección de B3 es u = (–1, 0), de tal manera que 
 
μ0 i 10
    B3 = u = 2(10 −7 ) ( −i ) = ( −5 i ) 10 −7  
2 πr 4
 

El campo magnético total en el origen es 
 

    B = B1 + B2 + B3 = [(0.56 i –0.49 j) + (–0.6 i + 0.8 j) + (–5 i)] 10–7 
 

⇒     B = (–5.04 i+  0.31 j)  10–7 T 
7‐17 

 
7.8. CAMPO MAGNÉTICO DEBIDO A UNA ESPIRA CIRCULAR. 
  Como  segundo  ejemplo  de  aplicación  de  la  Ley  de  Biot‐Savart,  calcularemos  ahora  el  campo 
magnético producido por una espira circular de corriente “i” y radio “a”, en un punto arbitrario sobre su 
eje de simetría perpendicular. 
  Coloquemos  la  espira  con  su   
plano coincidente con el plano XY, y su 
centro O en el origen, como se muestra 
en la Fig. 7.7.  
  Sea  P(0,  0,  z)  el  punto  donde 
deseamos calcular el campo  B (o sea el 
“punto  campo”).  Sea  dr’  el  elemento 
de  corriente,  ubicado  en  un  punto 
arbitrario de la espira (este es el “punto 
fuente”). 
  Usaremos  coordenadas  polares 
para ubicar el punto fuente general. De 
hecho,  solamente  necesitamos  la 
coordenada  polar  θ’,  pues  la 
coordenada radial es constante:  a.   
  Tracemos el vector separación  R   
desde  el  punto  fuente  hasta  el  punto  Fig. 19 
campo. 
  De acuerdo con la Ley de Biot‐Savart, el campo diferencial dB producido en P por el elemento de 
corriente es 
 
μ i dr ′ × R
[(8)]    B= 0
4π R3
∫  

 
Vamos a expresar  dB en la base vectorial cartesiana  {i, j, k}, expresando las componentes de los diversos 
vectores en términos de la variable θ’. 
  Las  coordenadas  cartesianas  del 
punto fuente son (Véase la Fig. 7.8) 
 
  (a cos θ’, a sen θ’, 0) 
 
y las del punto campo son 
 
  (0, 0, z) 
 
de  tal  manera  que  las  componentes  del 
 
vector R son 
Fig. 20 
 

(11)    R = (–a cos θ’, – a sen θ’, z) 
7‐18 

  Por otra parte, el vector dr’ tiene la dirección tangente a la espira que, como se muestra en la 
Fig. 7.8, es la dirección del vector unitario “(– sen θ’ i + cos θ’ j)”. La magnitud de  dr’ es el elemento de 
arco “a dθ’’”, de tal manera que 
 
(12)    dr’ = a dθ’ (– sen θ’ i + cos θ’ j) 
 
  Finalmente, la magnitud del vector R es 
 
(13)    R = a 2 + z 2 = cons tan te  
 
  Ahora bien, la espira posee simetría rotacional alrededor del Eje Z. Los campos diferenciales “dB” 
de  sus  elementos  de  corriente  “giran”  alrededor  de  dicho  eje,  de  tal  manera  que  sus  componentes 
horizontales  se  cancelan  (Véase  la  Fig.  7.8).  Así  pues,  solamente  hay  que  calcular  la  componente  Z  del 
campo B. Tenemos 
 
μ i (dr ′ × R )z
    dBz = 0 ⋅  
4π R3
 
Usando la relación válida para cualesquiera vectores A y B: 
 
    ( A × B)z = A x By − A y Bx  
 
además de las expresiones (11) y (12), obtenemos 
 
    (dr’ × R)z = – a sen θ’ dθ’ ⋅ (– a sen θ’) – a cos θ’ dθ’ (– a cos θ’) = a2 dθ’ 
 
μ i 2 π a 2dθ′ μ0 i ⋅ a 2 2 π μ i ⋅ a2
⇒     Bz = 0 ∫
4π 0 R 3
=
4πR 3

0 ∫
dθ′ = 0
4πR 3
⋅ 2π  

 
(Se usó que “R” es constante, dado por (13)). 
  Vectorialmente, 
 
  Campo magnético de una espira circular de radio “a” que 
μ0 ia 2 transporta una corriente “i”, en un punto arbitrario de su 
(14)    B= k  eje  de  simetría  perpendicular,  a  distancia  “z”  del  centro 
2(a 2 + z 2 )3 / 2
de la espira. 
 
 
  Notemos que el sentido de rotación de la corriente eléctrica y la dirección del campo  B obedecen la 
regla de la mano derecha. 
  En la línea recta que pasa por el centro de la espira, perpendicularmente al plano de la misma, el 
campo  magnético  varía  como  se  muestra  en  las  Figs.  7.9  y  7.10:  es  máximo  en  el  centro  de  la  espira  y 
disminuye con la distancia al mismo. 
7‐19 

  

 
Fig. 21 
 
  Note que la línea de campo a través 
del centro de la espira apunta siempre en la 
misma dirección. 
  En  la  Fig.  22  se  muestra  un  gráfico 
del campo de la espira a lo largo de su eje 
de simetría perpendicular. 
  El campo en el centro de la espira es 
máximo y vale 
 
μ0 i
    B=  
2a
 
 

 
Fig. 22 
 
 
7‐20 

7.9. CAMPO MAGNÉTICO DE UN SOLENOIDE. 
  Un solenoide es un alambre conductor que ha sido enrollado en forma de hélice. La distancia entre 
espiras  (o  longitud  de  paso  del  solenoide)  se  supone  pequeña.  En  muchos  casos  las  espiras  tienen  un 
recubrimiento aislante y están muy bien empaquetadas, tocándose unas con otras. 
  En la Fig. 7.11 se representa un solenoide, por el que se manda una corriente constante “i”. Visto 
desde el extremo izquierdo, la corriente del solenoide circula las espiras en el sentido de las manecillas del 
reloj. 
 

 
Fig. 23 
 
  Vamos  a  calcular  el  campo  magnético  producido  por  esta  corriente  eléctrica,  en  un  punto 
cualquiera del eje longitudinal central de la espira (recta a través de los centros de las espiras). 
  Visualizamos  el  solenoide  como  un  conjunto  muy  numeroso  de  espiras  paralelas  muy 
empaquetadas.  El  empaquetamiento  de  las  espiras  se  describe  mediante  la  densidad  de  espiras  “n”, 
definida como el número “N” de espiras por unidad de longitud del solenoide, es decir 
 
N
    n=     (L = longitud del solenoide) 
L
 
  Véase  la  Fig.  7.12.  El  método  consistirá  en  calcular  el  campo  “dB”  producido  por  un  manojo  de 
espiras dispuestas a lo largo de un pequeño tramo “  dx’ “, localizado en el punto variable “  x’ “, y luego 
integrar dicho campo diferencial para obtener el de todo el solenoide. 
 

 
Fig. 24 
7‐21 

  Advierta que todos los campos diferenciales dB, independientemente de la posición de las espiras 
que los generan,  tienen la dirección del Eje X (hacia la derecha en la Fig. 7.12), de tal manera que la suma 
vectorial de estos campos se convierte en una suma algebraica. 
  Para obtener el campo de las espiras confinadas al tramo  dx’, utilizaremos la expresión del campo 
de una espira dada en la Ec. (14), a saber, 
 

μ0 ia 2
    B=  
2(a 2 + z 2 )3 / 2
 

donde “a” es el radio de las espiras. Hay que adaptar esta expresión al caso presente. En primer lugar, la 
distancia  del  punto  campo  al  centro  de  las  espiras  en  dx’  no  es  “z”  sino  “x  –  x’  ”.  Por  otra  parte,  el 
número de espiras contenidas en dx’ es “n dx’ “, y cada una de ellas transporta una corriente “i”, como si 
tuviéramos pues una sola espira por la que circula una corriente “  n  dx’  i “. Finalmente cambiaremos  B 
por dB. Con estos cambios tenemos 
 

μ0 ⋅ n idx′ ⋅ a 2
    dB =  
2(a 2 + (x − x′)2 )3 / 2
 
  El campo del solenoide en el punto x es entonces 
 

μ n ia 2 L dx′
    B= 0
2

0 ∫
(a + (x − x′)2 )3 / 2
2
 

L
μ n ia 2 ⎡ ⎤ ⎡ ⎤
⎢ x − x′ ⎥ = μ0 n i ⎢ L−x x ⎥ 
    B= 0 ⋅ − ⋅ +
2 ⎢ 2 2 2 ⎥ 2 ⎢ 2 2 a 2 + x2 ⎥
⎣ a a + (x − x′) ⎦0 ⎣ a + (x − L) ⎦
 

 
Fig. 25 
 

  El campo se puede escribir en la forma más compacta 
 

1
(15)    B =  μ0 n i (cos αL + cos α0)     
2
 
 

donde  αL y  α0 son los ángulos que forman con el Eje X las rectas que van desde el punto campo P hasta 


los extremos superiores del solenoide, como vemos en la Fig. 7.13. 
7‐22 

  Las  líneas  del  campo  magnético  del  solenoide  son  curvas  cerradas.  Fuera  del  solenoide  el  campo 
decrece rápidamente con la distancia (Véase la Fig. 7.14). Este campo es semejante al campo magnético de 
un  imán  de  barra.  La  región  de  donde  “salen”  las  líneas  de  fuerza  se  denomina  el  polo  norte  del 
solenoide. La región opuesta, por donde “entran” las líneas al solenoide, es el polo sur del mismo. 
 

 
Fig. 26 
 

  Para muchos fenómenos, el solenoide se comporta   
como  un  imán  de  barra.  Por  ejemplo,  si  se  colocan  dos 
solenoides con sus polos como se muestra en la Fig. 7.15, 
se  produce  una  fuerza  de  atracción  entre  ambos.  La   
fuerza  es  de  repulsión  si  los  solenoides  se  colocan  con  Fig. 27 
polos de la misma denominación uno cerca del otro. 
 
7.10. CAMPO MAGNÉTICO DE UN SOLENOIDE INFINITO. 
  Observemos  el  solenoide  de  nuevo  en  la  Fig.  7.13  de  la  sección  anterior.  Cuando  la  longitud  del 
solenoide  tiende  a  infinito  (manteniéndose  constante  la  densidad  de  espiras  “n”),  los  ángulos  αL  y  α0 
mostrados  allí  tienden  a  cero.  Entonces  sus  cosenos  tienden    a  “1”  y  el  campo  magnético  (15)  tiende  al 
valor: 
 
  Campo magnético de un solenoide infinito. 
(16)    B = μ0 n i  “n” es la densidad de espiras e “i” es la corriente 
que circula por el solenoide. 
 
 

 
Fig. 28 
 
  Por otra parte, si el solenoide es infinito (Fig. 7.16), el campo magnético es uniforme en el interior 
del solenoide, y es nulo fuera de él. Sus líneas de fuerza del campo son rectas. 
  El  solenoide  constituye  un  dispositivo  muy  útil,  pues  en  la  práctica  se  puede  producir  con  un 
solenoide muy largo un campo magnético más o menos uniforme en la parte central del mismo. 
7‐23 

 
EJEMPLO 7.4.  Se tiene un solenoide de longitud  L = 0.5 m, compuesto de  100 espiras, por el que circula 
una corriente i = 10 A. El radio de las espiras es R = 0.2 m. Calcular el campo magnético del solenoide en 
su punto central. 
‐‐‐‐‐‐‐‐‐‐‐‐‐‐‐‐‐‐‐‐‐‐‐‐‐‐‐‐‐‐‐‐‐‐‐‐‐‐‐‐‐‐‐‐‐ 
  Emplearemos la fórmula (15). 
  En  el  punto  central  las  rectas  a 
los  extremos  del  solenoide  forman  el 
mismo ángulo α. Se tiene 
 
0.25
  cos α = = 
(0.2)2 + (0.25)2
 

    = 0.78 
   
La densidad de espiras es  Fig. 29 
 
100 1
  n= = 200  
0.5m m
 

Poniendo  αL = α0 = α  en (15) obtenemos (en unidades S.I.) 
 
1
    B =   μ0 n i (cos αL + cos α0) = μ0 n i (2 cos α) 
2
       = 0.5 × 4π (10–7) ×⋅ 200 × 10 × (2 × 0.78) = 0.0195      (T) 
o bien 
 

    B = 195 gauss 
‐‐‐‐‐‐‐‐‐‐‐‐‐‐‐‐‐‐‐‐‐‐‐‐‐‐‐‐‐‐‐‐‐‐‐‐‐‐‐‐‐ 
 
7‐24 

7.11. FUERZA MAGNÉTICA SOBRE UN ALAMBRE RECTO QUE TRANSPORTA CORRIENTE ELÉCTRICA 
  DENTRO DE UN CAMPO MAGNÉTICO CONSTANTE. 

  Un campo magnético produce fuerzas sobre cargas eléctricas en movimiento. Si las cargas se están 
moviendo dentro de un alambre conductor, constituyendo una corriente eléctrica que fluye en el seno del 
campo,  entonces  el  campo  magnético  produce  una  fuerza  sobre  el  conductor.  Vamos  a  obtener  una 
expresión matemática para esta fuerza. 
  La Fig. 7.17 representa un alambre conductor dentro de un campo magnético uniforme  B dirigido 
hacia dentro del papel. El alambre transporta una corriente “i”. Consideremos un segmento del alambre, 
de  longitud  “L”.  Definamos  el  vector  “L”,  cuya  magnitud  sea  la  longitud  de  este  segmento,  y  cuya 
dirección esté en el sentido de la corriente “i”. ¿Cuál es la fuerza magnética sobre este segmento? 
   

 
   
Fig. 30  Fig. 31 
 

  Observe  la  Fig.  7.18.  Cada  uno  de  los  portadores  de  carga  dentro  del  conductor  sufre  una  fuerza 
dada  por  “e  v  ×  B”,  donde  “e”  es  la  carga  del  portador  y  “v”  es  su  vector  velocidad.  Dentro  de  un 
segmento  de  longitud  L  del  alambre  existe  un  número  “n  A  L”  de  tales  portadores,  donde  “n”  es  la 
densidad de electrones libres y “A” es el área de la sección transversal del alambre. 
  Entonces la fuerza magnética sobre todos los portadores de carga contenidos en el segmento “L” es 
 

    (n A L) e v × B 
 

Ahora bien, los vectores “v” y “L” tienen la misma dirección, así que podemos hacer el cambio 
 

    L v = v L 
 

Finalmente, usando la fórmula de la nave 
 

    i = nAve 
 

obtenemos 
 

  Fuerza  magnética  sobre  un  segmento  recto  de 


(17)    F = i L × B  alambre  de  longitud  L  dentro  de  un  campo 
  magnético uniforme B. 
7‐25 

 
EJEMPLO 7.5.  Se propone medir el campo magnético uniforme entre los polos de un electroimán de la 
siguiente  manera:  se  suspende  de  un  dinamómetro  un  alambre  conductor  por  el  que  se  envía  una 
corriente  eléctrica  “i”.  Una  porción  de  longitud  L  del  alambre  corre  horizontalmente  dentro  del  campo 
magnético como se muestra en la Fig. A. Relacionar el campo magnético B con los parámetros pertinentes. 
   

 
 
 
 
 
Fig. 32  Fig. 33 
 
  La  Fig.  B  muestra  una  vista  frontal  del  dispositivo.  En  esta  figura,  el  segmento  de  alambre 
horizontal  dentro  del  campo  se  ve  como  un  punto,  y  la  corriente  eléctrica  en  él  va  en  el  sentido 
perpendicular  al  papel,  hacia  el  lector.  El  campo  magnético  del  electroimán  va  en  dirección  de  su  polo 
norte “N” a su polo sur “S”, hacia la izquierda. 
  Sobre el segmento de alambre mencionado existe una fuerza magnética dada por la fórmula 
 

    F = i L × B 
 

(Las  fuerzas  sobre  los  segmentos  verticales  de  alambre  se  cancelan  mutuamente  y  no  afectan 
sensiblemente el cálculo). 
En la Fig. B vemos que esta fuerza  F está dirigida hacia abajo y, dado que los vectores  L (hacia fuera del 
papel) y  B (hacia la izquierda) son perpendiculares, su magnitud es  F = i L B. Esta fuerza motiva que el 
dinamómetro en la parte superior sufra cierta elongación “δ”. 
  La fuerza del dinamómetro equilibra la fuerza magnética, de tal manera que 
 

    k δ = i L B 
 

Despejando B obtenemos 
 

    B=  
iL
 

  Por ejemplo, usando i = 20 A, L = 0.1 m, y k = 1000 N/m, se mide una elongación de δ = 1.5 mm. 
se sigue que (en unidades S.I.) 
 
1000 ⋅ 1.5(10 −3 )
    B= = 0.75    (tesla) 
20 ⋅ 0.1
7‐26 

 
EJEMPLO  7.6.   Una  varilla  recta  de  masa  m  puede  deslizarse  verticalmente  mediante  guías  sin  fricción 
haciendo contacto con dos rieles verticales lisos. La varilla, por la que circula una corriente eléctrica “i”, se 
suelta desde cierta posición y del reposo. Desde ese momento, la varilla entra en una región donde existe 
un campo magnético uniforme  B (hacia dentro del papel en la Fig. 44). Calcular la velocidad de la varilla 
después de haber caído una distancia “h” dentro del campo. 
 

 
Fig. 44 
 
  De acuerdo con la fórmula  F = i L × B, y dado que el vector  L en este caso es horizontal y hacia la 
izquierda,  en  el  sentido  de  la  corriente,  la  fuerza  magnética  sobre  la  varilla  apunta  verticalmente  hacia 
abajo. Entonces el diagrama de cuerpo libre de la varilla es el que se muestra en la Fig. 45. Consta del peso 
de la varilla, “mg”, y de la fuerza magnética “iLB”.   
Existen  además  las  reacciones  normales  de  los 
rieles  sobre  la  varilla,  pero  aquí  solamente  nos 
interesan las fuerzas verticales. 
  La varilla adquiere una aceleración constante 
 
vertical hacia abajo igual a 
Fig. 45 
 
mg + iLB
    a=  
m
 
  Recordando  la  fórmula  del  movimiento  con  aceleración  constante  que  expresa  la  velocidad  en 
términos de la distancia recorrida, o sea 
 
    v 2 = v 02 + 2a(x − x0 )     (v0 = velocidad en x = x0) 
 
obtenemos, con x0 = 0 y v0 = 0, v2 = 2ax. Poniendo x = h, 
 
⎛ iLB ⎞
    v2 = 2 ⎜ g + ⎟h  
⎝ m ⎠
7‐27 

 
EJEMPLO  7.7.   Una  espira  circular  de  radio  “r”,  por  la  que  circula  una  corriente  “I”,  se  encuentra  en  el 
campo  magnético  de  un  imán  permanente.  El  eje  longitudinal  de  la  espira  coincide  con  el  del  imán. 
Demostrar  que,  si  el  sentido  de  la  corriente  es  el  indicado  en  la  figura,  el  imán  ejerce  una  fuerza  de 
atracción sobre la espira. 
 

 
Fig. 44 
 
  La Fig. 44 muestra la espira vista en su propio plano. En la parte superior la corriente “I” va hacia 
dentro del papel, en la parte inferior hacia fuera. El campo magnético  B del imán forma, en los elementos 
de corriente de la espira, un ángulo θ con la vertical. 
  Tomando  los  ejes  X,  Y,  Z  como  se  muestra,  la  fuerza  sobre  el  elemento  de  corriente  en  la  parte 
superior de la espira es 
 
    dF = I dr × B = I (– ds k) × (– Bx i + By j) = I ds Bx j + I ds By i 
 
           = I ds B sen θ j + I ds B cos θ i 
 
donde  ds = |dr|.  Al hacer la suma de las fuerzas sobre todos los elementos de corriente de la espira, se 
cancelan las componentes Y y Z de la fuerza total. Resta solamente la componente X, igual a 
 

    Fx = I ( 2πr ) B cos θ 
 

  La fuerza de atracción entre el imán y la espira es análoga a la atracción entre dos imanes que tienen 
sus polos opuestos cerca uno del otro. Uno puede considerar que la espira tiene dos polos, norte y sur, tal 
como se indica en la Fig. 44. El polo norte de la espira estaría cerca de la misma, en la región por donde 
“salen” sus líneas de campo magnético. 
  Varios  fenómenos  relacionados  con  espiras  se  pueden  explicar  cualitativamente  de  manera 
conveniente considerando a la espira como si fuera un pequeño imán de barra permanente. 
7‐28 

 
7.12. DEFINICIÓN DEL AMPERIO, UNIDAD ELECTROMAGNÉTICA BÁSICA 
  EN EL SISTEMA INTERNACIONAL DE UNIDADES. 

  La  unidad  básica  electromagnética  en  el  Sistema  Internacional  de  Unidades  no  es  la  unidad  de 
carga eléctrica, el coulombio, sino la de corriente eléctrica, el amperio. 
  He aquí la definición del amperio: 
 
  Un  amperio  es  la  corriente  constante  que,  al  circular  por  dos  conductores  paralelos  de 
(21)  longitud  infinita  y  separados  por  una  distancia  de  un  metro  en  el  espacio  vacío,  motiva 
que cada conductor experimente una fuerza de exactamente 2 × 10– 7  N/m. 
 
  Analicemos esta definición. 
 

Fig. 34  Fig. 35 
 
  Consideremos  dos  alambres  conductores  rectos  por  los  que  circulan  corrientes  “i1”  e  “i2”,  como 
vemos  en  la  Fig.  7.22..  Sea  “x”  la  distancia  entre  ambos  alambres.  El  alambre  “1”  produce  en  un  punto 
cualquiera del alambre “2” un campo B1 dado por la Ec. (10): 
 
μ i
    B1 = 0 1  
2 πx
 
  Usando  ahora  la  Ec.  (17),  tenemos  que  la  fuerza  sobre  el  segmento  L2  mostrado  en  la  Fig.  7.22, 
debida al alambre “1”, es 
 
    F1 / 2 = i 2 L 2 × B1  
 
Tal fuerza apunta hacia el alambre “1”. Como L2 y B1 son perpendiculares, la magnitud de la fuerza es 
 
7‐29 

μ i i L
    F1/ 2 = i 2 L2 B1 = 0 1 2 2  
2πx
7‐30 

  Como se representa en la Fig. 7.23, sobre el alambre 2 actúa una distribución lineal de fuerza, cuya 
densidad de fuerza viene dada por 
 
F1/ 2μ i i
(22)    = 0 1 2  
L2 2 πx
 
Esta es la fuerza por unidad de longitud que el alambre 1 ejerce sobre el 2. Recíprocamente, el alambre 2 
ejerce sobre el 1 la misma distribución lineal de fuerza. Los alambres se atraen. 
  De acuerdo con la definición del amperio, si la distancia entre ambos alambres, “x”, es igual a un 
metro, esta densidad de fuerza debe valer 
 
F1/ 2 N
(23)    = 2(10 −7 )  
L2 m
 
  De aquí se deduce el valor de μ0. Igualando (22) y (23), 
 
μ0 i1 i 2 N
    = 2(10−7 )  
2πx m
 
2 πx N 2 π ⋅ 1m
⇒     μ0 = ⋅ 2(10 −7 ) = ⋅ 2(10 −7 )m  
i1 i 2 m 1A ⋅ 1A
 
  Usando la definición de la unidad “weber”, a saber,: 
 
N⋅m
    1 Wb = 1 , 
A
 
se obtiene 
 
Wb
    μ 0 = 4 π (10 −7 )  
A⋅m
 
  Como hemos demostrado, pues, este valor de  μ0, que hemos estado usando desde el principio del 
capítulo, proviene directamente de la definición del amperio. 
  El coulombio, unidad de carga eléctrica, se define ahora en términos del amperio en la forma 
 
    1 coulombio = 1 amperio × 1 segundo 
 
    1 C = 1 A ⋅ s 
7‐31 

 
7.13. ESPIRA RECTANGULAR DE CORRIENTE DENTRO DE UN CAMPO MAGNÉTICO UNIFORME. 
  Consideremos  una  espira  rectangular  de  lados  “a”  y  “b”,  por  la  que  circula  una  corriente  “i” 
constante. Supongamos que la espira esté situada dentro de un campo magnético uniforme “B”, dirigido 
en el sentido del Eje Y. Hemos colocado la espira de modo que sus lados inferior y superior, de longitud 
“a”, sean paralelos al Eje X, como se muestra en la Fig. 7.8. Además, el plano de la espira forma un ángulo 
θ con el plano XZ, como miramos en la Fig. 7.20. 
 

Fig. 36  Fig. 37 
 

  Sabemos que sobre cada lado de la espira existe una fuerza debida al campo magnético, dada por la 
expresión  
 
    F = i L × B 
 

donde  L  es un  segmento  recto  vectorial  de  alambre conductor,  tomado a  lo largo,  y  con la  longitud  de, 
cada lado en el sentido de la corriente. 
  Calculemos la fuerza total del campo  B sobre la espira. El campo magnético es  B = B  j, y los lados 
de la espira (los vectores “L” de la expresión (17)) son 
 

    Superior:    a i 
    Inferior:    – a i 
    Anterior:    – b sen θ j – b cos θ k 
    Posterior:    b sen θ j + b cos θ k 
 
  Evaluemos para cada lado el producto vectorial indicado en (17), o sea la fuerza sobre el mismo: 
 

    Superior:    i (a i) × B = i (a i) × (B j) = i a B k 
    Inferior:    i (– a i) × B = i (– a i) × (B j) = – i a B k 
    Anterior:    i (– b sen θ j – b cos θ k) × (B j) = i b B cos θ i 
    Posterior:    i ( b sen θ j + b cos θ k) × (B j) = – i b B cos θ i 
 
  Cada una de estas fuerzas actúa continuamente a lo largo de todo el lado, pero se puede considerar 
que actúa en el punto medio del lado. 
7‐32 

  Como  vemos,  la  suma  vectorial  de  las  cuatro  fuerzas  es  igual  a  cero.  Sin  embargo,  la  torca  que 
producen estas fuerzas es diferente de cero. Las dos fuerzas que actúan sobre los lados anterior y posterior 
forman  un  par  cuya  torca  es  nula.  Pero  el  par  de  fuerzas  que  actúa  sobre  los  lados  superior  e  inferior 
produce una torca igual a 
 
(18)    τ = i a b B sen θ i     (La primera “i” es la  corriente y la segunda el vector unitario “i”) 
 
ya que la distancia entre las líneas de acción de las fuerzas del par es  “i a B k” e “–i a B k” es “b cos θ”. 
  Se define el momento magnético de la espira, μ, como un vector cuya magnitud es “i A”, donde  
A = ab  es  el  área  de  la  espira  e “i”  es la  corriente que  circula  por  ella.  La  dirección  de  este  vector  es  la 
misma que la del vector área  A de la espira, tomado de tal manera que el sentido de la corriente eléctrica 
corresponda con el sentido de A conforme a la regla de la mano derecha. Tenemos 
 
 
Momento magnético de una espira de área 
(19)    μ = i A 
vectorial A por la que circula corriente “i”. 
 
 
  En  la  Fig.  7.20  vemos  que  el  área  vectorial  del  rectángulo  es  el  producto  vectorial  de  los  lados 
superior y anterior del rectángulo, o sea 
 
    A = (a i) × (– b sen θ j – b cos θ k) = – A sen θ k + A cos θ j 
 
Usando (19), se verifica la igualdad 
 
  Torca sobre una espira de momento magnético 
(20)    τ = μ × B  μ situada dentro de un campo magnético 
  uniforme B. 
 

(Puesto que 
 

    μ × B = (i A) × (B j) = i (– A sen θ k + A cos θ j) × (B j) = μ B sen θ i 
 
  Como  vemos  en  la Fig. 7.21,  el ángulo  que  forman los  vectores  μ  y  B es  θ, de  tal  modo que  este 
último resultado se puede poner como el producto vectorial de μ y B.) 
  En  la  Fig.  7.21,  la  dirección  de  la  torca  τ  es   
perpendicular  al  plano  YZ,  hacia  fuera  del  papel. 
Esta torca tiende a hacer rotar el plano de la espira 
en  el  sentido  antihorario  alrededor  del  Eje  X.  En 
cuanto  el  vector  μ  rebasa  la  posición  horizontal  la 
torca  cambia  de  sentido  y  ahora  produce  una 
rotación  horaria.  En  otras  palabras,  el  plano  de  la 
espira  tiende  a  conservarse  perpendicular al Eje  Y,   
o bien, el vector  μ tiende a alinearse con el campo   
magnético B.  Fig. 38 
 
7‐33 

 
7.14. LA LEY DE AMPÈRE. 
  En nuestro estudio de la electrostática utilizamos una de sus leyes fundamentales, la ley de Gauss, 
para calcular campos eléctricos de distribuciones de carga muy simétricas, como el alambre recto, el plano 
infinito,  la  esfera,  etc.  Existe  en  magnetostática  una  ley  similar  para  calcular  campos  magnéticos  de 
distribuciones de corriente de gran simetría, si bien la formulación matemática es muy distinta a aquella. 
Es la llamada Ley de Ampère. 
  Consideremos  un  conjunto  de  hilos  (alambres)  conductores,  digamos  N  de  ellos,  que  transportan 
sendas  corrientes  i1,  i2,  …,  iN  (Fig.  39).  Tracemos  una  curva  cerrada  arbitraria  C,  tal  que  algunos  de  los 
hilos de corriente la atraviesen. 
 

 
Fig. 39 
 
  La Ley de Ampère, que no demostraremos aquí, establece que 
 
  La  integral  de  línea  del  campo  magnético  a  lo  largo  de  una  curva 
  cerrada  arbitraria  C  es  igual  a  la  constante  μ0  multiplicada  por  la 
(21)    ∫C
o B • dr = μ0 iatr   corriente  total  “iatr”  que  atraviesa  dicha  curva.  Cada  corriente  se 
considera  positiva  (o  negativa)  según  su  dirección  y  el  sentido  de 
integración  a  lo  largo  de  C  obedezca  (o  no)  la  regla  de  la  mano 
derecha. 
 
  La  ley  de  Ampère  es  válida  no  solamente  para  hilos  conductores.  La  corriente  eléctrica  puede 
circular  a  lo  largo  de  un  cuerpo  sólido  arbitrario.  La  integral  que  aparece  en  (21)  se  denomina  la 
circulación del campo magnético a lo largo de la curva C, que se denomina curva de Ampère. 
  En la Fig. 39, la corriente  i1 no atraviesa la curva  C, de tal modo que no contribuye a  iatr. (Note la 
analogía con la ley de Gauss, en la que las cargas fuera de la superficie Gaussiana no contribuyen al flujo 
del campo a través de la superficie). 
7‐34 

 
EJEMPLO 7.6.  Utilizaremos la Ley de Ampère para calcular un campo magnético ya conocido: el de un 
alambre recto infinito. 
  Sea  “i”  la  corriente  que  transporta  el   
alambre,  dirigida  hacia  arriba  en  la  Fig.  40. 
Escojamos la curva de Ampère como un círculo de 
radio  “r”  con  su  plano  perpendicular  al  alambre  y 
centrado  en  el  mismo.  Integraremos  el  campo  a  lo 
largo de tal círculo, en un sentido de integración tal 
que  cumpla  la  regla  de  la  mano  derecha  con 
respecto a la dirección de la corriente. 
  Para  aplicar  la  ley  de  Ampère  es  necesario 
hacer  una  hipótesis  sobre  las  líneas  de  fuerza  del 
campo  magnético,  basándonos  en  que  tales  líneas 
son  siempre  curvas  cerradas  (o  líneas  infinitas). 
Pero  en  el  caso  simple  del  alambre  ya  las 
conocemos:  las  líneas  de  fuerza  son  círculos   
centrados en el alambre.  Fig. 40 
  La ley de Ampère es 
 
    ∫C
o B • dr = μ0 iatr  

 
  Ahora bien, como se advierte en la Fig. 40, el vector dr y el campo B son paralelos en todo punto de 
C, de tal manera que su producto escalar se reduce a 
 
    B • dr = B ds cos 0° = B ds 
 
donde  “ds”  es  la  magnitud  de  dr,  es  decir,  el  elemento  de  arco  del  círculo  de  Ampère.  Además,  por 
simetría, B es constante sobre C, de tal manera que la integral se reduce a 
 
    ∫C
o B • dr = ∫C Bds = B∫C ds = B ⋅ 2πr  
 
Por otra parte, la corriente que atraviesa la curva de Ampère es “i”, de tal modo que 
 
    B 2πr = μ0 i 
 
de donde 
 
μ0 i
    B=  
2π r
 
Esta es la expresión encontrada anteriormente para este campo. 
7‐35 

 
EJEMPLO  7.7.   Un  cilindro  sólido  conductor  de  radio  R,  recto  e  infinito,  transporta  una  corriente  “i” 
distribuída uniformemente a través de su sección transversal. Utilizar la ley de Ampère para calcular el 
campo magnético del cilindro dentro y fuera de él. 
  Calculemos  el  campo  B  dentro  del  cilindro.   
Suponemos que tanto dentro como fuera, las líneas 
de fuerza del campo son círculos centrados en el eje 
central de simetría longitudinal del cilindro. 
  Observe la curva de Ampère  C escogida para 
la región dentro del cilindro: es un círculo de radio 
“r”. Observemos que sobre  se cumple, como en el 
ejemplo anterior, que  B y  dr son paralelos en todo 
punto, y que B es constante. Entonces la integral de 
la ley de se reduce a 
 
  ∫C
o B • dr = ∫C Bds = B∫C ds = B ⋅ 2πr  
 
  Por  otra  parte,  la  corriente  iatr  que  atraviesa 
la  curva  de  Ampère  es  proporcional  a  su  área 
“πr2”.  La  podemos  obtener  haciendo  una 
proporción  entre  corriente  y  área,  del  siguiente 
Fig. 41 
modo: 
 
iatr πr 2
    =  
i πR 2
 
Se tiene entonces que   iatr = (r2/R2) i,   y la ley de Ampère nos da 
 
    B 2πr = μ0 (r2/R2) i 
 
de donde 
 
μ0 i
    B= r 
2πR 2
 

Como vemos, el campo varía linealmente con “r” dentro del cilindro. 
  Fuera del cilindro, de nuevo tomaríamos como curva de Ampère un círculo de radio r > R centrado 
en el eje longitudinal del cilindro. En este caso la corriente que atraviesa tal curva de Ampère es toda la 
corriente “i” que circula por el cilindro, de tal manera que (como puede comprobar fácilmente) la ley de 
Ampère da 
 
μ0 i
    B=  
2 πr
 

Así, fuera del cilindro el campo es el mismo que el de un hilo conductor recto e infinito. 
7‐36 

 
7.15. PROBLEMAS. 
 
1.   Dos  cargas  puntuales  q1  =  2  μC  y  q2  =  4  μC  se 
mueven  en  cierto  momento  paralelamente  en  las 
direcciones –X y +X con velocidades  v1 = 8 (106) m/s  
y    v2  =  3  (106)  m/s,  respectivamente,  cuando  su 
distancia  mutua  d  =  0.4  m  es  mínima.  Calcular  el 
campo  magnético  producido  por  ambas  cargas  en  el 
punto medio P.   
 
 
2.   Un  protón  viaja  con  una  velocidad  v  =  2  (105) 
m/s  que  forma  un  ángulo  θ  =  35°  con  el  Eje  X. 
Existe un campo magnético uniforme  B = 0.1 T en 
la dirección positiva del Eje Y. (a) Calcular la fuerza 
magnética  sobre  el  protón,  en  magnitud  y 
dirección. (b) ¿Cuál es la aceleración del protón? 
Datos. La masa y la carga eléctrica del protón son 
mp = 1.67 (10–27) kg 

e = 1.60 (10–19) C 
 
 
 
3.   Tres  cargas  puntuales  describen  trayectorias 
como  las  mostradas  en  la  figura,  dentro  de  un 
campo  magnético  constante  B.  ¿Qué  puede  Ud 
concluir sobre los signos de estas cargas? 

 
4.   Una  fuente  de  iones  produce  iones 
cada uno con carga positiva “e” (masa de 
cada ión  m = 6 mp, donde  mp es la masa 
del  protón).  Los  iones  son  acelerados  a 
través de una diferencia de potencial de  
9  kV,  pasando  luego  por  una  región 
donde  existen  campos  eléctricos  y 
magnéticos  cruzados,  constantes.  El   
campo magnético vale 0.9 T. Calcular el 
campo eléctrico para que los iones pasen sin desviarse por esta región. 
7‐37 

 
5.  Una carga negativa “q” entra con velocidad  v0 a   
una  región  donde  existe  un  campo  magnético 
constante  B.  Luego  de  recorrer  dentro  del  campo 
una  distancia  horizontal  “d”,  la  carga  sale  de  la 
región.  Calcular  el  ángulo  que  forma  su  velocidad 
de salida “v” con la horizontal. 
Sugerencia.  Calcular  el  radio  de  la  trayectoria  de 
“q”, y resolver un triángulo rectángulo simple. 

 
6.   Una  cinta  metálica  plana  con  las  siguientes   
dimensiones: 
  Largo = 8.5 cm 
  Ancho = 1 cm 
  Espesor = 0.8 mm 
se  mueve  con  velocidad  constante  v  dentro  de  un 
campo  magnético  constante  B  perpendicular  a  su 
plano,  como  se  muestra  en  la  figura.  Se  mide  una 
diferencia  de  potencial  de  2.6  μV  entre  los  puntos 
“a” y “b”. Calcular la velocidad de la cinta. 
Sugerencia. Aplique las ideas discutidas en relación 
con el Efecto Hall (Sección 7.5, página 7‐9). 

 
7.  Un ión positivo con una carga igual en magnitud a la del electrón se acelera desde el reposo a través de 
una  diferencia  de  potencial  de  1,800  V,  entrando  luego  a  un  campo  magnético  de  2  T  en  una  dirección 
perpendicular  a  la  del  campo  (Consulte  el  espectrómetro  de  masas  del  Ejemplo  7.4  en  la  página  7‐10). 
Calcular el radio de la trayectoria circular dentro del campo. La masa del ión es de 5 (10–25) kg. 
 
8.  En la Sección 7.5, en la página 7‐9,  se discutió el Efecto Hall, y se halló la relación 
 
iB
n=
    Vte  
 
donde “n” es la densidad de electrones libres, “i” la corriente,  B el campo magnético, “V” el voltaje Hall, 
“t” el espesor de la cinta metálica y “e” la carga del electrón. 
  Se desea medir el campo magnético de un electroimán utilizando el efecto Hall. Describa un arreglo 
experimental para hacerlo. 
 
7‐38 

 
9.  En la situación del problema 8 anterior, ¿Cuál es el valor mínimo que debe tener el campo magnético 
para  que  la  carga  “q”  no  alcance  a  salir  de  la  región  del  campo  (es  decir,  para  que  su  desplazamiento 
horizontal sea a lo más igual a “d”)? 
 
10 . Calcular el campo magnético de un alambre muy largo que transporta una corriente I = 20 A, en un 
punto a una distancia de 1.2 m del alambre. ¿Cómo se compara este campo con el campo magnético de la 
Tierra, de orden 1 gauss? 
 
11.  Dos alambres conductores infinitos transportan 
corrientes  de  5A  y  8A,  como  se  muestra  en  la 
figura. Calcular el campo magnético que producen 
estos alambres en el punto P mostrado. 
Sugerencia.  Defina  un  sistema  cartesiano  XYZ  y 
proceda  vectorialmente.  Consulte  antes  el  Ejemplo 
7.6 en la página 7‐15.   
 
12.  Un alambre conductor está compuesto de dos secciones rectas muy largas, y de una espira circular de 
radio “r”, como se muestra en la figura. Calcular el campo magnético en el centro de la espira. 
 

 
 
13.  Cuatro alambres conductores rectos muy largos   
transportan  cada  uno  la  misma  corriente  de  50  A,   
en  las  direcciones  mostradas  en  la  figura  (la 
corriente  va  hacia  dentro  del  papel  en  los 
conductores de la izquierda, y hacia fuera en los de 
la  derecha).  Calcular  el  campo  magnético  que 
producen  los  4  en  el  centro  de  la  configuración,  el 
punto  C.  ¿Cuánto  vale  el  campo  si  se  invierte  la   
dirección de la corriente en el alambre situado en la 
parte inferior derecha? 
 
14.   Diseñe  un  solenoide  de  longitud  L  =  15  cm  tal  que  el  campo  magnético  en  el  punto  central  del 
solenoide, sobre su eje longitudinal, sea de B = 100 gauss. 
 
 
 
7‐39 

 
15.   Una  varilla  conductora,  suspendida  por  dos   
alambres  conductores,  transporta  una  corriente 
eléctrica I = 40 A dentro de un campo magnético B 
uniforme dirigido hacia fuera del papel. La masa de 
la  varilla  es  m  =  0.1  kg.  ¿Cuánto  debe  valer  el 
campo  magnético  para  que  la  tensión  en  los 
alambres  que  sostienen  la  varilla  sea  nula.  ¿Cuál 
debe ser la dirección de la corriente I? 

 
16.   Un  aro  circular  cuya  circunferencia  es  de  1.2  m  mantiene  una  corriente  de  3.5  mA.  El  aro  se  halla 
dentro de un campo magnético B = 0.5 T cuya dirección es paralela al plano del aro. Calcular el momento 
magnético del aro, así como la torca que ejerce sobre él el campo magnético. 
 
 
17.  Una espira rectangular de largo “a” y ancho “b” 
que transporta corriente eléctrica “i” se halla dentro 
de un campo magnético uniforme  B, de tal manera 
que el plano de la espira es paralelo al campo. 
  Calcular  la  fuerza  y  la  torca  total  sobre  esta 
espira debida al campo B. 

 
18.   Cuatro  hilos  rectos  conductores  transportan  las  corrientes  indicadas  en  la  figura.  De  acuerdo  con  la 
Ley  de  Ampère,  ¿Cuánto  vale  la  circulación  del  campo  magnético  a  lo  largo  de  cada  una  de  las  curvas 
cerradas C1, C2 y C3? 
 

 
7‐40 

 
19.   Una  placa  plana  infinita  de  espesor  “t”  transporta  una  corriente  a  lo  largo  de  su  superficie.  La 
intensidad de la corriente se puede describir mediante una densidad lineal de corriente  Js, definida como 
la  corriente  que  atraviesa  la  sección  transversal  de  la  placa  por  unidad  de  longitud.  Así  por  ejemplo,  la 
corriente que pasa a través de la sección transversal de extensión longitudinal “L” mostrada en la figura 
abajo a la izquierda es “Js L”. 
   
 

 
Suponiendo que el campo magnético de la placa infinita es constante y tiene sus líneas de fuerza paralelas 
a la placa, como se muestra en la figura arriba a la derecha, calcular el campo magnético B usando la Ley 
de Ampère. 
Sugerencia.  Escoja  como  curva  de  Ampère  un  rectángulo  de  dimensiones  arbitrarias  (el  rectángulo 
punteado en la figura a la derecha), dispuesto simétricamente con respecto a la placa. 
1
Resp.  B = μ 0 Js . 
2
 

Anda mungkin juga menyukai